Listen to earn your patients’ trust

Article Type
Changed
Fri, 06/14/2024 - 09:07

Recently, I had an interesting conversation while getting my hair cut. It gave me a great deal of insight into some of the problems we have right now with how medical information is shared and some of the disconnect our patients may feel.

The young woman who was cutting my hair asked me what I did for an occupation. I said that I was a physician. She said, “Can I please ask you an important question?” She asked me what my thoughts were about the COVID vaccine. She prefaced it with “I am so confused on whether I should get the vaccine. I have seen a number of TikTok videos that talk about nano particles in the COVID vaccine that can be very dangerous.”

I discussed with her how the COVID vaccine actually works and shared with her the remarkable success of the vaccine. I asked her what side effects she was worried about from the vaccine and what her fears were. She said that she had heard that a lot of people had died from the vaccine. I told her that severe reactions from the vaccine were very uncommon.

Paauw_Doug_SEATTLE_2019_web.jpg
Dr. Douglas S. Paauw

She then made a very telling comment: “I wish I could talk to a doctor about my concerns. I have been going to the same health center for the last 5 years and every time I go I see a different person.” She added, “I rarely have more than 5-10 minutes with the person that I am seeing and I rarely get the opportunity to ask questions.”

She thanked me for the information and said that she would be getting the COVID vaccine in the future. She said it is so hard to know where to get information now and the very different things that she heard confused her. She told me that she thought her generation got most of its information from short sound bites or TikTok and Instagram videos.

Why did she trust me? I still think that the medical profession is respected. We are all pressured to do more with less time. Conversations where we can listen and then respond go a long way. We can always listen and learn what information people need and will appreciate. I was also struck by how alone this person felt in our health care system. She did not have a relationship with any one person whom she could trust and reach out to with questions. Relationships with our patients go a long way to establishing trust.
 

Pearl

It takes time to listen to and answer our patients’ questions. We need to do that to fight the waves of misinformation our patients face.

Dr. Paauw is professor of medicine in the division of general internal medicine at the University of Washington, Seattle, and he serves as third-year medical student clerkship director at the University of Washington. He is a member of the editorial advisory board of Internal Medicine News. Dr. Paauw has no conflicts to disclose. Contact him at imnews@mdedge.com.

Publications
Topics
Sections

Recently, I had an interesting conversation while getting my hair cut. It gave me a great deal of insight into some of the problems we have right now with how medical information is shared and some of the disconnect our patients may feel.

The young woman who was cutting my hair asked me what I did for an occupation. I said that I was a physician. She said, “Can I please ask you an important question?” She asked me what my thoughts were about the COVID vaccine. She prefaced it with “I am so confused on whether I should get the vaccine. I have seen a number of TikTok videos that talk about nano particles in the COVID vaccine that can be very dangerous.”

I discussed with her how the COVID vaccine actually works and shared with her the remarkable success of the vaccine. I asked her what side effects she was worried about from the vaccine and what her fears were. She said that she had heard that a lot of people had died from the vaccine. I told her that severe reactions from the vaccine were very uncommon.

Paauw_Doug_SEATTLE_2019_web.jpg
Dr. Douglas S. Paauw

She then made a very telling comment: “I wish I could talk to a doctor about my concerns. I have been going to the same health center for the last 5 years and every time I go I see a different person.” She added, “I rarely have more than 5-10 minutes with the person that I am seeing and I rarely get the opportunity to ask questions.”

She thanked me for the information and said that she would be getting the COVID vaccine in the future. She said it is so hard to know where to get information now and the very different things that she heard confused her. She told me that she thought her generation got most of its information from short sound bites or TikTok and Instagram videos.

Why did she trust me? I still think that the medical profession is respected. We are all pressured to do more with less time. Conversations where we can listen and then respond go a long way. We can always listen and learn what information people need and will appreciate. I was also struck by how alone this person felt in our health care system. She did not have a relationship with any one person whom she could trust and reach out to with questions. Relationships with our patients go a long way to establishing trust.
 

Pearl

It takes time to listen to and answer our patients’ questions. We need to do that to fight the waves of misinformation our patients face.

Dr. Paauw is professor of medicine in the division of general internal medicine at the University of Washington, Seattle, and he serves as third-year medical student clerkship director at the University of Washington. He is a member of the editorial advisory board of Internal Medicine News. Dr. Paauw has no conflicts to disclose. Contact him at imnews@mdedge.com.

Recently, I had an interesting conversation while getting my hair cut. It gave me a great deal of insight into some of the problems we have right now with how medical information is shared and some of the disconnect our patients may feel.

The young woman who was cutting my hair asked me what I did for an occupation. I said that I was a physician. She said, “Can I please ask you an important question?” She asked me what my thoughts were about the COVID vaccine. She prefaced it with “I am so confused on whether I should get the vaccine. I have seen a number of TikTok videos that talk about nano particles in the COVID vaccine that can be very dangerous.”

I discussed with her how the COVID vaccine actually works and shared with her the remarkable success of the vaccine. I asked her what side effects she was worried about from the vaccine and what her fears were. She said that she had heard that a lot of people had died from the vaccine. I told her that severe reactions from the vaccine were very uncommon.

Paauw_Doug_SEATTLE_2019_web.jpg
Dr. Douglas S. Paauw

She then made a very telling comment: “I wish I could talk to a doctor about my concerns. I have been going to the same health center for the last 5 years and every time I go I see a different person.” She added, “I rarely have more than 5-10 minutes with the person that I am seeing and I rarely get the opportunity to ask questions.”

She thanked me for the information and said that she would be getting the COVID vaccine in the future. She said it is so hard to know where to get information now and the very different things that she heard confused her. She told me that she thought her generation got most of its information from short sound bites or TikTok and Instagram videos.

Why did she trust me? I still think that the medical profession is respected. We are all pressured to do more with less time. Conversations where we can listen and then respond go a long way. We can always listen and learn what information people need and will appreciate. I was also struck by how alone this person felt in our health care system. She did not have a relationship with any one person whom she could trust and reach out to with questions. Relationships with our patients go a long way to establishing trust.
 

Pearl

It takes time to listen to and answer our patients’ questions. We need to do that to fight the waves of misinformation our patients face.

Dr. Paauw is professor of medicine in the division of general internal medicine at the University of Washington, Seattle, and he serves as third-year medical student clerkship director at the University of Washington. He is a member of the editorial advisory board of Internal Medicine News. Dr. Paauw has no conflicts to disclose. Contact him at imnews@mdedge.com.

Publications
Publications
Topics
Article Type
Sections
Teambase XML
<?xml version="1.0" encoding="UTF-8"?>
<!--$RCSfile: InCopy_agile.xsl,v $ $Revision: 1.35 $-->
<!--$RCSfile: drupal.xsl,v $ $Revision: 1.7 $-->
<root generator="drupal.xsl" gversion="1.7"> <header> <fileName>168406</fileName> <TBEID>0C0508A2.SIG</TBEID> <TBUniqueIdentifier>MD_0C0508A2</TBUniqueIdentifier> <newsOrJournal>News</newsOrJournal> <publisherName>Frontline Medical Communications</publisherName> <storyname/> <articleType>353</articleType> <TBLocation>QC Done-All Pubs</TBLocation> <QCDate>20240613T172019</QCDate> <firstPublished>20240614T090308</firstPublished> <LastPublished>20240614T090308</LastPublished> <pubStatus qcode="stat:"/> <embargoDate/> <killDate/> <CMSDate>20240614T090308</CMSDate> <articleSource/> <facebookInfo/> <meetingNumber/> <byline/> <bylineText/> <bylineFull/> <bylineTitleText/> <USOrGlobal/> <wireDocType/> <newsDocType/> <journalDocType/> <linkLabel/> <pageRange/> <citation/> <quizID/> <indexIssueDate/> <itemClass qcode="ninat:text"/> <provider qcode="provider:imng"> <name>IMNG Medical Media</name> <rightsInfo> <copyrightHolder> <name>Frontline Medical News</name> </copyrightHolder> <copyrightNotice>Copyright (c) 2015 Frontline Medical News, a Frontline Medical Communications Inc. company. All rights reserved. This material may not be published, broadcast, copied, or otherwise reproduced or distributed without the prior written permission of Frontline Medical Communications Inc.</copyrightNotice> </rightsInfo> </provider> <abstract/> <metaDescription>BY DOUGLAS S. PAAUW, MD</metaDescription> <articlePDF/> <teaserImage>248215</teaserImage> <teaser>Fight misinformation by taking time to answer your patients’ questions.</teaser> <title>Listen to earn your patients’ trust</title> <deck/> <disclaimer/> <AuthorList/> <articleURL/> <doi/> <pubMedID/> <publishXMLStatus/> <publishXMLVersion>1</publishXMLVersion> <useEISSN>0</useEISSN> <urgency/> <pubPubdateYear/> <pubPubdateMonth/> <pubPubdateDay/> <pubVolume/> <pubNumber/> <wireChannels/> <primaryCMSID/> <CMSIDs/> <keywords/> <seeAlsos/> <publications_g> <publicationData> <publicationCode>fp</publicationCode> <pubIssueName/> <pubArticleType/> <pubTopics/> <pubCategories/> <pubSections/> </publicationData> <publicationData> <publicationCode>im</publicationCode> <pubIssueName/> <pubArticleType/> <pubTopics/> <pubCategories/> <pubSections/> </publicationData> </publications_g> <publications> <term>15</term> <term canonical="true">21</term> </publications> <sections> <term>52</term> <term>41022</term> <term canonical="true">39786</term> </sections> <topics> <term canonical="true">38029</term> </topics> <links> <link> <itemClass qcode="ninat:picture"/> <altRep contenttype="image/jpeg">images/2400c6a0.jpg</altRep> <description role="drol:caption">Dr. Douglas S. Paauw</description> <description role="drol:credit"/> </link> </links> </header> <itemSet> <newsItem> <itemMeta> <itemRole>Main</itemRole> <itemClass>text</itemClass> <title>Listen to earn your patients’ trust</title> <deck/> </itemMeta> <itemContent> <p>BY DOUGLAS S. PAAUW, MD</p> <p>Recently, I had an interesting conversation while getting my hair cut. It gave me a great deal of insight into some of the problems we have right now with how medical information is shared and some of the disconnect our patients may feel.</p> <p>The young woman who was cutting my hair asked me what I did for an occupation. I said that I was a physician. She said, “Can I please ask you an important question?” She asked me what my thoughts were about the COVID vaccine. She prefaced it with “I am so confused on whether I should get the vaccine. I have seen a number of TikTok videos that talk about nano particles in the COVID vaccine that can be very dangerous.” <br/><br/>I discussed with her how the COVID vaccine actually works and shared with her the remarkable success of the vaccine. I asked her what side effects she was worried about from the vaccine and what her fears were. She said that she had heard that a lot of people had died from the vaccine. I told her that severe reactions from the vaccine were very uncommon. <br/><br/>[[{"fid":"248215","view_mode":"medstat_image_flush_right","fields":{"format":"medstat_image_flush_right","field_file_image_alt_text[und][0][value]":"Dr. Douglas S. Paauw, University of Washington, Seattle","field_file_image_credit[und][0][value]":"","field_file_image_caption[und][0][value]":"Dr. Douglas S. Paauw"},"type":"media","attributes":{"class":"media-element file-medstat_image_flush_right"}}]]She then made a very telling comment: “I wish I could talk to a doctor about my concerns. I have been going to the same health center for the last 5 years and every time I go I see a different person.” She added, “I rarely have more than 5-10 minutes with the person that I am seeing and I rarely get the opportunity to ask questions.”<br/><br/>She thanked me for the information and said that she would be getting the COVID vaccine in the future. She said it is so hard to know where to get information now and the very different things that she heard confused her. She told me that she thought her generation got most of its information from short sound bites or TikTok and Instagram videos.<br/><br/>Why did she trust me? I still think that the medical profession is respected. We are all pressured to do more with less time. Conversations where we can listen and then respond go a long way. We can always listen and learn what information people need and will appreciate. I was also struck by how alone this person felt in our health care system. She did not have a relationship with any one person whom she could trust and reach out to with questions. Relationships with our patients go a long way to establishing trust. <br/><br/></p> <h2>Pearl</h2> <p>It takes time to listen to and answer our patients’ questions. We need to do that to fight the waves of misinformation our patients face.<span class="end"/></p> <p> <em>Dr. Paauw is professor of medicine in the division of general internal medicine at the University of Washington, Seattle, and he serves as third-year medical student clerkship director at the University of Washington. He is a member of the editorial advisory board of Internal Medicine News. Dr. Paauw has no conflicts to disclose. Contact him at <span class="Hyperlink"><a href="mailto:imnews%40mdedge.com?subject=">imnews@mdedge.com</a></span>.</em> </p> </itemContent> </newsItem> <newsItem> <itemMeta> <itemRole>teaser</itemRole> <itemClass>text</itemClass> <title/> <deck/> </itemMeta> <itemContent> </itemContent> </newsItem> </itemSet></root>
Disallow All Ads
Content Gating
No Gating (article Unlocked/Free)
Alternative CME
Disqus Comments
Default
Use ProPublica
Hide sidebar & use full width
render the right sidebar.
Conference Recap Checkbox
Not Conference Recap
Clinical Edge
Display the Slideshow in this Article
Medscape Article
Display survey writer
Reuters content
Disable Inline Native ads
WebMD Article

Myth of the Month: Is Contrast-Induced Acute Kidney Injury Real?

Article Type
Changed
Mon, 05/13/2024 - 09:24

A 59-year-old man presents with abdominal pain. He has a history of small bowel obstruction and diverticulitis. His medical history includes chronic kidney disease (CKD; baseline creatinine, 1.8 mg/dL), hypertension, type 2 diabetes, and depression. He had a colectomy 6 years ago for colon cancer.

He takes the following medications: Semaglutide (1 mg weekly), amlodipine (5 mg once daily), and escitalopram (10 mg once daily). On physical exam his blood pressure is 130/80 mm Hg, his pulse is 90, and his temperature is 37.2 degrees C. He has normal bowel sounds but guarding in the right lower quadrant.

Paauw_Doug_SEATTLE_2019_web.jpg
Dr. Douglas S. Paauw


His hemoglobin is 14 g/dL, his blood sodium is 136 mEq/L, his blood potassium is 4.0 mmol/L, his BUN is 26 mg/dL, and his creatinine is 1.9 mg/dL. His kidney, ureter, bladder x-ray is unremarkable.
 

What imaging would you recommend?

A) CT without contrast

B) CT with contrast

C) MRI

D) Abdominal ultrasound

This patient has several potential causes for his abdominal pain that imaging may clarify. I think a contrast CT scan would be the most likely to provide helpful information. It is likely that if it were ordered, there may be hesitation by the radiologist to perform the scan with contrast because of the patient’s CKD.

Concern for contrast-induced kidney injury has limited diagnostic testing for many years. How strong is the evidence for contrast-induced kidney injury, and should we avoid testing that requires contrast in patients with CKD? McDonald and colleagues performed a meta-analysis with 13 studies meeting inclusion criteria, involving 25,950 patients.1 They found no increased risk of acute kidney injury (AKI) in patients who received contrast medium compared with those who did not receive contrast; relative risk of AKI for those receiving contrast was 0.79 (confidence interval: 0.62-1.02). Importantly, there was no difference in AKI in patients with diabetes or CKD.

Ehmann et al. looked at renal outcomes in patients who received IV contrast when they presented to an emergency department with AKI.2 They found that in patients with AKI, receiving contrast was not associated with persistent AKI at hospital discharge. Hinson and colleagues looked at patients arriving at the emergency department and needing imaging.3 They did a retrospective, cohort analysis of 17,934 patients who had CT with contrast, CT with no contrast, or no CT. Contrast administration was not associated with increased incidence of AKI (odds ratio, 0.96, CI: 0.85-1.08).

Aycock et al. did a meta-analysis of AKI after CT scanning, including 28 studies involving 107,335 patients.4 They found that compared with noncontrast CT, CT scanning with contrast was not associated with AKI (OR, 0.94, CI: 0.83-1.07). Elias and Aronson looked at the risk of AKI after contrast in patients receiving CT scans compared with those who received ventilation/perfusion scans to evaluate for pulmonary embolism.5 There were 44 AKI events (4.5%) in patients exposed to contrast media and 33 events (3.4%) in patients not exposed to contrast media (risk difference: 1.1%, 95% CI: -0.6% to 2.9%; OR, 1.39, CI: 0.86-2.26; P = .18).

Despite multiple studies showing no increased risk, there is still a concern that contrast can cause AKI.6 Animal models have shown iodinated contrast can have a deleterious effect on mitochondria and membrane function.6 Criticisms of the retrospective nature of many of the studies I have shared, and the lack of randomized, controlled trials are that there may be bias in these studies, as the highest-risk patients are the ones most likely not to receive contrast. In a joint guideline from the American College of Radiology and the National Kidney Foundation, this statement was made: “The risk of acute kidney injury developing in patients with reduced kidney function following exposure to intravenous iodinated contrast media has been overstated.”7 Their recommendation was to give contrast if needed in patients with glomerular filtration rates (GFRs) greater than 30.



Myth: Contrast-induced renal injury is a concern.

Clinical impact: For CT scanning, it is OK to give contrast when needed. A conservative cutoff for contrast use would be a GFR less than 30.
 

Dr. Paauw is professor of medicine in the Division of General Internal Medicine at the University of Washington, Seattle, and he serves as third-year medical student clerkship director at the University of Washington. Contact Dr. Paauw at dpaauw@uw.edu.

References

1. McDonald JS et al. Radiology. 2013:267:119-128.

2. Ehmann MR et al. Intensive Care Med. 2023:49(2):205-215.

3. Hinson JS et al. Ann Emerg Med. 2017;69(5):577-586.

4. Aycock RD et al. Ann Emerg Med. 2018 Jan;71(1):44-53.

5. Elias A, Aronson D. Thromb Haemost. 2021 Jun;121(6):800-807.

6. Weisbord SD, du Cheryon D. Intensive Care Med. 2018;44(1):107-109.

7. Davenport MS et al. Radiology. 2020;294(3):660-668.

Publications
Topics
Sections

A 59-year-old man presents with abdominal pain. He has a history of small bowel obstruction and diverticulitis. His medical history includes chronic kidney disease (CKD; baseline creatinine, 1.8 mg/dL), hypertension, type 2 diabetes, and depression. He had a colectomy 6 years ago for colon cancer.

He takes the following medications: Semaglutide (1 mg weekly), amlodipine (5 mg once daily), and escitalopram (10 mg once daily). On physical exam his blood pressure is 130/80 mm Hg, his pulse is 90, and his temperature is 37.2 degrees C. He has normal bowel sounds but guarding in the right lower quadrant.

Paauw_Doug_SEATTLE_2019_web.jpg
Dr. Douglas S. Paauw


His hemoglobin is 14 g/dL, his blood sodium is 136 mEq/L, his blood potassium is 4.0 mmol/L, his BUN is 26 mg/dL, and his creatinine is 1.9 mg/dL. His kidney, ureter, bladder x-ray is unremarkable.
 

What imaging would you recommend?

A) CT without contrast

B) CT with contrast

C) MRI

D) Abdominal ultrasound

This patient has several potential causes for his abdominal pain that imaging may clarify. I think a contrast CT scan would be the most likely to provide helpful information. It is likely that if it were ordered, there may be hesitation by the radiologist to perform the scan with contrast because of the patient’s CKD.

Concern for contrast-induced kidney injury has limited diagnostic testing for many years. How strong is the evidence for contrast-induced kidney injury, and should we avoid testing that requires contrast in patients with CKD? McDonald and colleagues performed a meta-analysis with 13 studies meeting inclusion criteria, involving 25,950 patients.1 They found no increased risk of acute kidney injury (AKI) in patients who received contrast medium compared with those who did not receive contrast; relative risk of AKI for those receiving contrast was 0.79 (confidence interval: 0.62-1.02). Importantly, there was no difference in AKI in patients with diabetes or CKD.

Ehmann et al. looked at renal outcomes in patients who received IV contrast when they presented to an emergency department with AKI.2 They found that in patients with AKI, receiving contrast was not associated with persistent AKI at hospital discharge. Hinson and colleagues looked at patients arriving at the emergency department and needing imaging.3 They did a retrospective, cohort analysis of 17,934 patients who had CT with contrast, CT with no contrast, or no CT. Contrast administration was not associated with increased incidence of AKI (odds ratio, 0.96, CI: 0.85-1.08).

Aycock et al. did a meta-analysis of AKI after CT scanning, including 28 studies involving 107,335 patients.4 They found that compared with noncontrast CT, CT scanning with contrast was not associated with AKI (OR, 0.94, CI: 0.83-1.07). Elias and Aronson looked at the risk of AKI after contrast in patients receiving CT scans compared with those who received ventilation/perfusion scans to evaluate for pulmonary embolism.5 There were 44 AKI events (4.5%) in patients exposed to contrast media and 33 events (3.4%) in patients not exposed to contrast media (risk difference: 1.1%, 95% CI: -0.6% to 2.9%; OR, 1.39, CI: 0.86-2.26; P = .18).

Despite multiple studies showing no increased risk, there is still a concern that contrast can cause AKI.6 Animal models have shown iodinated contrast can have a deleterious effect on mitochondria and membrane function.6 Criticisms of the retrospective nature of many of the studies I have shared, and the lack of randomized, controlled trials are that there may be bias in these studies, as the highest-risk patients are the ones most likely not to receive contrast. In a joint guideline from the American College of Radiology and the National Kidney Foundation, this statement was made: “The risk of acute kidney injury developing in patients with reduced kidney function following exposure to intravenous iodinated contrast media has been overstated.”7 Their recommendation was to give contrast if needed in patients with glomerular filtration rates (GFRs) greater than 30.



Myth: Contrast-induced renal injury is a concern.

Clinical impact: For CT scanning, it is OK to give contrast when needed. A conservative cutoff for contrast use would be a GFR less than 30.
 

Dr. Paauw is professor of medicine in the Division of General Internal Medicine at the University of Washington, Seattle, and he serves as third-year medical student clerkship director at the University of Washington. Contact Dr. Paauw at dpaauw@uw.edu.

References

1. McDonald JS et al. Radiology. 2013:267:119-128.

2. Ehmann MR et al. Intensive Care Med. 2023:49(2):205-215.

3. Hinson JS et al. Ann Emerg Med. 2017;69(5):577-586.

4. Aycock RD et al. Ann Emerg Med. 2018 Jan;71(1):44-53.

5. Elias A, Aronson D. Thromb Haemost. 2021 Jun;121(6):800-807.

6. Weisbord SD, du Cheryon D. Intensive Care Med. 2018;44(1):107-109.

7. Davenport MS et al. Radiology. 2020;294(3):660-668.

A 59-year-old man presents with abdominal pain. He has a history of small bowel obstruction and diverticulitis. His medical history includes chronic kidney disease (CKD; baseline creatinine, 1.8 mg/dL), hypertension, type 2 diabetes, and depression. He had a colectomy 6 years ago for colon cancer.

He takes the following medications: Semaglutide (1 mg weekly), amlodipine (5 mg once daily), and escitalopram (10 mg once daily). On physical exam his blood pressure is 130/80 mm Hg, his pulse is 90, and his temperature is 37.2 degrees C. He has normal bowel sounds but guarding in the right lower quadrant.

Paauw_Doug_SEATTLE_2019_web.jpg
Dr. Douglas S. Paauw


His hemoglobin is 14 g/dL, his blood sodium is 136 mEq/L, his blood potassium is 4.0 mmol/L, his BUN is 26 mg/dL, and his creatinine is 1.9 mg/dL. His kidney, ureter, bladder x-ray is unremarkable.
 

What imaging would you recommend?

A) CT without contrast

B) CT with contrast

C) MRI

D) Abdominal ultrasound

This patient has several potential causes for his abdominal pain that imaging may clarify. I think a contrast CT scan would be the most likely to provide helpful information. It is likely that if it were ordered, there may be hesitation by the radiologist to perform the scan with contrast because of the patient’s CKD.

Concern for contrast-induced kidney injury has limited diagnostic testing for many years. How strong is the evidence for contrast-induced kidney injury, and should we avoid testing that requires contrast in patients with CKD? McDonald and colleagues performed a meta-analysis with 13 studies meeting inclusion criteria, involving 25,950 patients.1 They found no increased risk of acute kidney injury (AKI) in patients who received contrast medium compared with those who did not receive contrast; relative risk of AKI for those receiving contrast was 0.79 (confidence interval: 0.62-1.02). Importantly, there was no difference in AKI in patients with diabetes or CKD.

Ehmann et al. looked at renal outcomes in patients who received IV contrast when they presented to an emergency department with AKI.2 They found that in patients with AKI, receiving contrast was not associated with persistent AKI at hospital discharge. Hinson and colleagues looked at patients arriving at the emergency department and needing imaging.3 They did a retrospective, cohort analysis of 17,934 patients who had CT with contrast, CT with no contrast, or no CT. Contrast administration was not associated with increased incidence of AKI (odds ratio, 0.96, CI: 0.85-1.08).

Aycock et al. did a meta-analysis of AKI after CT scanning, including 28 studies involving 107,335 patients.4 They found that compared with noncontrast CT, CT scanning with contrast was not associated with AKI (OR, 0.94, CI: 0.83-1.07). Elias and Aronson looked at the risk of AKI after contrast in patients receiving CT scans compared with those who received ventilation/perfusion scans to evaluate for pulmonary embolism.5 There were 44 AKI events (4.5%) in patients exposed to contrast media and 33 events (3.4%) in patients not exposed to contrast media (risk difference: 1.1%, 95% CI: -0.6% to 2.9%; OR, 1.39, CI: 0.86-2.26; P = .18).

Despite multiple studies showing no increased risk, there is still a concern that contrast can cause AKI.6 Animal models have shown iodinated contrast can have a deleterious effect on mitochondria and membrane function.6 Criticisms of the retrospective nature of many of the studies I have shared, and the lack of randomized, controlled trials are that there may be bias in these studies, as the highest-risk patients are the ones most likely not to receive contrast. In a joint guideline from the American College of Radiology and the National Kidney Foundation, this statement was made: “The risk of acute kidney injury developing in patients with reduced kidney function following exposure to intravenous iodinated contrast media has been overstated.”7 Their recommendation was to give contrast if needed in patients with glomerular filtration rates (GFRs) greater than 30.



Myth: Contrast-induced renal injury is a concern.

Clinical impact: For CT scanning, it is OK to give contrast when needed. A conservative cutoff for contrast use would be a GFR less than 30.
 

Dr. Paauw is professor of medicine in the Division of General Internal Medicine at the University of Washington, Seattle, and he serves as third-year medical student clerkship director at the University of Washington. Contact Dr. Paauw at dpaauw@uw.edu.

References

1. McDonald JS et al. Radiology. 2013:267:119-128.

2. Ehmann MR et al. Intensive Care Med. 2023:49(2):205-215.

3. Hinson JS et al. Ann Emerg Med. 2017;69(5):577-586.

4. Aycock RD et al. Ann Emerg Med. 2018 Jan;71(1):44-53.

5. Elias A, Aronson D. Thromb Haemost. 2021 Jun;121(6):800-807.

6. Weisbord SD, du Cheryon D. Intensive Care Med. 2018;44(1):107-109.

7. Davenport MS et al. Radiology. 2020;294(3):660-668.

Publications
Publications
Topics
Article Type
Sections
Teambase XML
<?xml version="1.0" encoding="UTF-8"?>
<!--$RCSfile: InCopy_agile.xsl,v $ $Revision: 1.35 $-->
<!--$RCSfile: drupal.xsl,v $ $Revision: 1.7 $-->
<root generator="drupal.xsl" gversion="1.7"> <header> <fileName>167947</fileName> <TBEID>0C04FEF7.SIG</TBEID> <TBUniqueIdentifier>MD_0C04FEF7</TBUniqueIdentifier> <newsOrJournal>News</newsOrJournal> <publisherName>Frontline Medical Communications</publisherName> <storyname/> <articleType>353</articleType> <TBLocation>QC Done-All Pubs</TBLocation> <QCDate>20240510T181607</QCDate> <firstPublished>20240513T091952</firstPublished> <LastPublished>20240513T091952</LastPublished> <pubStatus qcode="stat:"/> <embargoDate/> <killDate/> <CMSDate>20240513T091952</CMSDate> <articleSource/> <facebookInfo/> <meetingNumber/> <byline>Douglas Paauw</byline> <bylineText>DOUGLAS S. PAAUW, MD</bylineText> <bylineFull>DOUGLAS S. PAAUW, MD</bylineFull> <bylineTitleText/> <USOrGlobal/> <wireDocType/> <newsDocType/> <journalDocType/> <linkLabel/> <pageRange/> <citation/> <quizID/> <indexIssueDate/> <itemClass qcode="ninat:text"/> <provider qcode="provider:imng"> <name>IMNG Medical Media</name> <rightsInfo> <copyrightHolder> <name>Frontline Medical News</name> </copyrightHolder> <copyrightNotice>Copyright (c) 2015 Frontline Medical News, a Frontline Medical Communications Inc. company. All rights reserved. This material may not be published, broadcast, copied, or otherwise reproduced or distributed without the prior written permission of Frontline Medical Communications Inc.</copyrightNotice> </rightsInfo> </provider> <abstract/> <metaDescription>A 59-year-old man presents with abdominal pain. He has a history of small bowel obstruction and diverticulitis. His medical history includes chronic kidney dise</metaDescription> <articlePDF/> <teaserImage>248215</teaserImage> <teaser>For CT scanning, it is OK to give contrast when needed. A conservative cutoff for contrast use would be a GFR less than 30.</teaser> <title>Myth of the Month: Is Contrast-Induced Acute Kidney Injury Real?</title> <deck/> <disclaimer/> <AuthorList/> <articleURL/> <doi/> <pubMedID/> <publishXMLStatus/> <publishXMLVersion>1</publishXMLVersion> <useEISSN>0</useEISSN> <urgency/> <pubPubdateYear/> <pubPubdateMonth/> <pubPubdateDay/> <pubVolume/> <pubNumber/> <wireChannels/> <primaryCMSID/> <CMSIDs/> <keywords/> <seeAlsos/> <publications_g> <publicationData> <publicationCode>mdemed</publicationCode> <pubIssueName/> <pubArticleType/> <pubTopics/> <pubCategories/> <pubSections/> <journalTitle/> <journalFullTitle/> <copyrightStatement/> </publicationData> <publicationData> <publicationCode>fp</publicationCode> <pubIssueName/> <pubArticleType/> <pubTopics/> <pubCategories/> <pubSections/> </publicationData> <publicationData> <publicationCode>im</publicationCode> <pubIssueName/> <pubArticleType/> <pubTopics/> <pubCategories/> <pubSections/> </publicationData> </publications_g> <publications> <term>58877</term> <term>15</term> <term canonical="true">21</term> </publications> <sections> <term>52</term> <term canonical="true">28225</term> </sections> <topics> <term canonical="true">255</term> <term>230</term> </topics> <links> <link> <itemClass qcode="ninat:picture"/> <altRep contenttype="image/jpeg">images/2400c6a0.jpg</altRep> <description role="drol:caption">Dr. Douglas S. Paauw</description> <description role="drol:credit"/> </link> </links> </header> <itemSet> <newsItem> <itemMeta> <itemRole>Main</itemRole> <itemClass>text</itemClass> <title>Myth of the Month: Is Contrast-Induced Acute Kidney Injury Real?</title> <deck/> </itemMeta> <itemContent> <p>A 59-year-old man presents with abdominal pain. He has a history of small bowel obstruction and diverticulitis. His medical history includes chronic kidney disease (CKD; baseline creatinine, 1.8 mg/dL), hypertension, type 2 diabetes, and depression. He had a colectomy 6 years ago for colon cancer.<br/><br/>He takes the following medications: Semaglutide (1 mg weekly), amlodipine (5 mg once daily), and escitalopram (10 mg once daily). On physical exam his blood pressure is 130/80 mm Hg, his pulse is 90, and his temperature is 37.2 degrees C. He has normal bowel sounds but guarding in the right lower quadrant.[[{"fid":"248215","view_mode":"medstat_image_flush_right","fields":{"format":"medstat_image_flush_right","field_file_image_alt_text[und][0][value]":"Dr. Douglas S. Paauw, University of Washington, Seattle","field_file_image_credit[und][0][value]":"","field_file_image_caption[und][0][value]":"Dr. Douglas S. Paauw"},"type":"media","attributes":{"class":"media-element file-medstat_image_flush_right"}}]] <br/><br/>His hemoglobin is 14 g/dL, his blood sodium is 136 mEq/L, his blood potassium is 4.0 mmol/L, his BUN is 26 mg/dL, and his creatinine is 1.9 mg/dL. His kidney, ureter, bladder x-ray is unremarkable.<br/><br/></p> <p>What imaging would you recommend?<br/><br/>A) CT without contrast<br/><br/>B) CT with contrast<br/><br/>C) MRI<br/><br/>D) Abdominal ultrasound</p> <p>This patient has several potential causes for his abdominal pain that imaging may clarify. I think a contrast CT scan would be the most likely to provide helpful information. It is likely that if it were ordered, there may be hesitation by the radiologist to perform the scan with contrast because of the patient’s CKD. <br/><br/>Concern for contrast-induced kidney injury has limited diagnostic testing for many years. How strong is the evidence for contrast-induced kidney injury, and should we avoid testing that requires contrast in patients with CKD? McDonald and colleagues performed a meta-analysis with 13 studies meeting inclusion criteria, involving 25,950 patients.<sup>1 </sup>They found no increased risk of acute kidney injury (AKI) in patients who received contrast medium compared with those who did not receive contrast; relative risk of AKI for those receiving contrast was 0.79 (confidence interval: 0.62-1.02). Importantly, there was no difference in AKI in patients with diabetes or CKD.<br/><br/>Ehmann et al. looked at renal outcomes in patients who received IV contrast when they presented to an emergency department with AKI.<sup>2 </sup>They found that in patients with AKI, receiving contrast was not associated with persistent AKI at hospital discharge. Hinson and colleagues looked at patients arriving at the emergency department and needing imaging.<sup>3</sup> They did a retrospective, cohort analysis of 17,934 patients who had CT with contrast, CT with no contrast, or no CT. Contrast administration was not associated with increased incidence of AKI (odds ratio, 0.96, CI: 0.85-1.08). <br/><br/>Aycock et al. did a meta-analysis of AKI after CT scanning, including 28 studies involving 107,335 patients.<sup>4</sup> They found that compared with noncontrast CT, CT scanning with contrast was not associated with AKI (OR, 0.94, CI: 0.83-1.07). Elias and Aronson looked at the risk of AKI after contrast in patients receiving CT scans compared with those who received ventilation/perfusion scans to evaluate for pulmonary embolism.<sup>5</sup> There were 44 AKI events (4.5%) in patients exposed to contrast media and 33 events (3.4%) in patients not exposed to contrast media (risk difference: 1.1%, 95% CI: -0.6% to 2.9%; OR, 1.39, CI: 0.86-2.26; <em>P</em> = .18).<br/><br/>Despite multiple studies showing no increased risk, there is still a concern that contrast can cause AKI.<sup>6</sup> Animal models have shown iodinated contrast can have a deleterious effect on mitochondria and membrane function.<sup>6</sup> Criticisms of the retrospective nature of many of the studies I have shared, and the lack of randomized, controlled trials are that there may be bias in these studies, as the highest-risk patients are the ones most likely not to receive contrast. In a joint guideline from the American College of Radiology and the National Kidney Foundation, this statement was made: “The risk of acute kidney injury developing in patients with reduced kidney function following exposure to intravenous iodinated contrast media has been overstated.”<sup>7</sup> Their recommendation was to give contrast if needed in patients with glomerular filtration rates (GFRs) greater than 30.<br/><br/><br/><br/>Myth: Contrast-induced renal injury is a concern.<br/><br/>Clinical impact: For CT scanning, it is OK to give contrast when needed. A conservative cutoff for contrast use would be a GFR less than 30.<br/><br/></p> <p> <em>Dr. Paauw is professor of medicine in the Division of General Internal Medicine at the University of Washington, Seattle, and he serves as third-year medical student clerkship director at the University of Washington. Contact Dr. Paauw at <span class="Hyperlink"><a href="mailto:dpaauw%40uw.edu?subject=">dpaauw@uw.edu</a></span>.</em> </p> <h2>References</h2> <p>1. McDonald JS et al. <span class="Hyperlink"><a href="https://pubs.rsna.org/doi/10.1148/radiol.12121460">Radiology. 2013:267:119-128</a></span>.<br/><br/>2. Ehmann MR et al. <span class="Hyperlink"><a href="https://link.springer.com/article/10.1007/s00134-022-06966-w">Intensive Care Med. 2023:49(2):205-215</a></span>.<br/><br/>3. Hinson JS et al. <span class="Hyperlink"><a href="https://www.annemergmed.com/article/S0196-0644(16)31388-9/abstract">Ann Emerg Med. 2017;69(5):577-586</a></span>.<br/><br/>4. Aycock RD et al. <span class="Hyperlink"><a href="https://www.annemergmed.com/article/S0196-0644(17)30881-8/abstract">Ann Emerg Med. 2018 Jan;71(1):44-53</a></span>.<br/><br/>5. Elias A, Aronson D. <span class="Hyperlink"><a href="https://www.thieme-connect.de/products/ejournals/abstract/10.1055/s-0040-1721387">Thromb Haemost. 2021 Jun;121(6):800-807</a></span>.<br/><br/>6. Weisbord SD, du Cheryon D. <span class="Hyperlink"><a href="https://link.springer.com/article/10.1007/s00134-017-5015-6">Intensive Care Med. 2018;44(1):107-109</a></span>.<br/><br/>7. Davenport MS et al. <span class="Hyperlink"><a href="https://pubs.rsna.org/doi/10.1148/radiol.2019192094">Radiology. 2020;294(3):660-668</a></span>.</p> </itemContent> </newsItem> <newsItem> <itemMeta> <itemRole>teaser</itemRole> <itemClass>text</itemClass> <title/> <deck/> </itemMeta> <itemContent> </itemContent> </newsItem> </itemSet></root>
Disallow All Ads
Content Gating
No Gating (article Unlocked/Free)
Alternative CME
Disqus Comments
Default
Use ProPublica
Hide sidebar & use full width
render the right sidebar.
Conference Recap Checkbox
Not Conference Recap
Clinical Edge
Display the Slideshow in this Article
Medscape Article
Display survey writer
Reuters content
Disable Inline Native ads
WebMD Article

What is the Best Approach to “Sinus Headaches”?

Article Type
Changed
Wed, 03/27/2024 - 10:04

A 27-year-old woman presents requesting antibiotics for a sinus headache. She reports she has had 3-4 episodes a year with pain in her maxillary area and congestion. She has not had fevers with these episodes. She had the onset of this headache 6 hours ago. She has had resolution of the pain within 24 hours in the past with the use of antibiotics and decongestants. What would be the best treatment for her?

A. Amoxicillin

B. Amoxicillin/clavulanate

C. Amoxicillin + fluticasone nasal spray

D. Sumatriptan

The best treatment would be sumatriptan. This is very likely a variant of migraine headache and migraine-directed therapy is the best option. In regard to sinus headache, the International Headache Society (IHS) classification states that chronic sinusitis is not a cause of headache and facial pain unless it relapses into an acute sinusitis.1

Paauw_Doug_SEATTLE_2019_web.jpg
Dr. Douglas S. Paauw

The recurrent nature of the headaches in this patient suggests a primary headache disorder with migraine being the most likely. In a study of 2991 patients with self-diagnosed or physician-diagnosed “sinus headaches,” 88% of the patients met IHS criteria for migraine.2 In this study, most of the patients had symptoms suggesting sinus problems, with the most common symptoms being sinus pressure (84%), sinus pain (82%), and nasal congestion (63%). The likely cause for these symptoms in migraine patients is vasodilation of the nasal mucosa that can be part of the migraine event.

Foroughipour and colleagues found similar results.3 In their study, 58 patients with “sinus headache” were evaluated, with the final diagnosis of migraine in 40 patients (69%), tension-type headache in 16 patients (27%), and chronic sinusitis with recurrent acute episodes in 2 patients (3%). Recurrent antibiotic therapy had been given to 73% of the tension-type headache patients and 66% of the migraine patients.

Obermann et al. looked at how common trigeminal autonomic symptoms were in patients with migraine in a population-based study.4 They found of 841 patients who had migraine, 226 reported accompanying unilateral trigeminal autonomic symptoms (26.9%).

Al-Hashel et al. reported on how patients with frequent migraine are misdiagnosed and how long it takes when they present with sinus symptoms. A total of 130 migraine patients were recruited for the study; of these, 81.5% were misdiagnosed with sinusitis. The mean time delay of migraine diagnosis was almost 8 years.5

In a study by Dr. Elina Kari and Dr. John M. DelGaudio, patients who had a history of “sinus headaches” were treated as though all these headaches were migraines. Fifty-four patients were enrolled, and 38 patients completed the study. All patients had nasal endoscopy and sinus CT scans that were negative. They were then given migraine-directed treatment to use for their headaches. Of the 38 patient who completed the study, 31 patients (82%) had a significant reduction in headache pain with triptan use, and 35 patients (92%) had a significant response to migraine-directed therapy.6 An expert panel consisting of otolaryngologists, neurologists, allergists, and primary care physicians concluded that the majority of sinus headaches can actually be classified as migraines.7

These references aren’t new. This information has been known in the medical literature for more than 2 decades, but I believe that the majority of medical professionals are not aware of it. In my own practice I have found great success treating patients with sinus headache histories with migraine-directed therapy (mostly triptans) when they have return of their headaches.


Pearl: When your patients say they have another sinus headache, think migraine.
 

Dr. Paauw is professor of medicine in the division of general internal medicine at the University of Washington, Seattle, and he serves as third-year medical student clerkship director at the University of Washington. He is a member of the editorial advisory board of Internal Medicine News. Dr. Paauw has no conflicts to disclose. Contact him at imnews@mdedge.com.

References

1. Jones NS. Expert Rev Neurother. 2009;9:439-44.

2. Schreiber CP et al. Arch Intern Med. 2004;164:1769-72.

3. Foroughipour M et al. Eur Arch Otorhinolaryngol. 2011;268:1593-6.

4. Obermann M et al. Cephalalgia. 2007 Jun;27(6):504-9.

5. Al-Hashel JY et al. J Headache Pain. 2013 Dec 12;14(1):97.

6. Kari E and DelGaudi JM. Laryngoscope. 2008;118:2235-9.

7. Levine HL et al. Otolaryngol Head Neck Surg. 2006 Mar;134(3):516-23.

Publications
Topics
Sections

A 27-year-old woman presents requesting antibiotics for a sinus headache. She reports she has had 3-4 episodes a year with pain in her maxillary area and congestion. She has not had fevers with these episodes. She had the onset of this headache 6 hours ago. She has had resolution of the pain within 24 hours in the past with the use of antibiotics and decongestants. What would be the best treatment for her?

A. Amoxicillin

B. Amoxicillin/clavulanate

C. Amoxicillin + fluticasone nasal spray

D. Sumatriptan

The best treatment would be sumatriptan. This is very likely a variant of migraine headache and migraine-directed therapy is the best option. In regard to sinus headache, the International Headache Society (IHS) classification states that chronic sinusitis is not a cause of headache and facial pain unless it relapses into an acute sinusitis.1

Paauw_Doug_SEATTLE_2019_web.jpg
Dr. Douglas S. Paauw

The recurrent nature of the headaches in this patient suggests a primary headache disorder with migraine being the most likely. In a study of 2991 patients with self-diagnosed or physician-diagnosed “sinus headaches,” 88% of the patients met IHS criteria for migraine.2 In this study, most of the patients had symptoms suggesting sinus problems, with the most common symptoms being sinus pressure (84%), sinus pain (82%), and nasal congestion (63%). The likely cause for these symptoms in migraine patients is vasodilation of the nasal mucosa that can be part of the migraine event.

Foroughipour and colleagues found similar results.3 In their study, 58 patients with “sinus headache” were evaluated, with the final diagnosis of migraine in 40 patients (69%), tension-type headache in 16 patients (27%), and chronic sinusitis with recurrent acute episodes in 2 patients (3%). Recurrent antibiotic therapy had been given to 73% of the tension-type headache patients and 66% of the migraine patients.

Obermann et al. looked at how common trigeminal autonomic symptoms were in patients with migraine in a population-based study.4 They found of 841 patients who had migraine, 226 reported accompanying unilateral trigeminal autonomic symptoms (26.9%).

Al-Hashel et al. reported on how patients with frequent migraine are misdiagnosed and how long it takes when they present with sinus symptoms. A total of 130 migraine patients were recruited for the study; of these, 81.5% were misdiagnosed with sinusitis. The mean time delay of migraine diagnosis was almost 8 years.5

In a study by Dr. Elina Kari and Dr. John M. DelGaudio, patients who had a history of “sinus headaches” were treated as though all these headaches were migraines. Fifty-four patients were enrolled, and 38 patients completed the study. All patients had nasal endoscopy and sinus CT scans that were negative. They were then given migraine-directed treatment to use for their headaches. Of the 38 patient who completed the study, 31 patients (82%) had a significant reduction in headache pain with triptan use, and 35 patients (92%) had a significant response to migraine-directed therapy.6 An expert panel consisting of otolaryngologists, neurologists, allergists, and primary care physicians concluded that the majority of sinus headaches can actually be classified as migraines.7

These references aren’t new. This information has been known in the medical literature for more than 2 decades, but I believe that the majority of medical professionals are not aware of it. In my own practice I have found great success treating patients with sinus headache histories with migraine-directed therapy (mostly triptans) when they have return of their headaches.


Pearl: When your patients say they have another sinus headache, think migraine.
 

Dr. Paauw is professor of medicine in the division of general internal medicine at the University of Washington, Seattle, and he serves as third-year medical student clerkship director at the University of Washington. He is a member of the editorial advisory board of Internal Medicine News. Dr. Paauw has no conflicts to disclose. Contact him at imnews@mdedge.com.

References

1. Jones NS. Expert Rev Neurother. 2009;9:439-44.

2. Schreiber CP et al. Arch Intern Med. 2004;164:1769-72.

3. Foroughipour M et al. Eur Arch Otorhinolaryngol. 2011;268:1593-6.

4. Obermann M et al. Cephalalgia. 2007 Jun;27(6):504-9.

5. Al-Hashel JY et al. J Headache Pain. 2013 Dec 12;14(1):97.

6. Kari E and DelGaudi JM. Laryngoscope. 2008;118:2235-9.

7. Levine HL et al. Otolaryngol Head Neck Surg. 2006 Mar;134(3):516-23.

A 27-year-old woman presents requesting antibiotics for a sinus headache. She reports she has had 3-4 episodes a year with pain in her maxillary area and congestion. She has not had fevers with these episodes. She had the onset of this headache 6 hours ago. She has had resolution of the pain within 24 hours in the past with the use of antibiotics and decongestants. What would be the best treatment for her?

A. Amoxicillin

B. Amoxicillin/clavulanate

C. Amoxicillin + fluticasone nasal spray

D. Sumatriptan

The best treatment would be sumatriptan. This is very likely a variant of migraine headache and migraine-directed therapy is the best option. In regard to sinus headache, the International Headache Society (IHS) classification states that chronic sinusitis is not a cause of headache and facial pain unless it relapses into an acute sinusitis.1

Paauw_Doug_SEATTLE_2019_web.jpg
Dr. Douglas S. Paauw

The recurrent nature of the headaches in this patient suggests a primary headache disorder with migraine being the most likely. In a study of 2991 patients with self-diagnosed or physician-diagnosed “sinus headaches,” 88% of the patients met IHS criteria for migraine.2 In this study, most of the patients had symptoms suggesting sinus problems, with the most common symptoms being sinus pressure (84%), sinus pain (82%), and nasal congestion (63%). The likely cause for these symptoms in migraine patients is vasodilation of the nasal mucosa that can be part of the migraine event.

Foroughipour and colleagues found similar results.3 In their study, 58 patients with “sinus headache” were evaluated, with the final diagnosis of migraine in 40 patients (69%), tension-type headache in 16 patients (27%), and chronic sinusitis with recurrent acute episodes in 2 patients (3%). Recurrent antibiotic therapy had been given to 73% of the tension-type headache patients and 66% of the migraine patients.

Obermann et al. looked at how common trigeminal autonomic symptoms were in patients with migraine in a population-based study.4 They found of 841 patients who had migraine, 226 reported accompanying unilateral trigeminal autonomic symptoms (26.9%).

Al-Hashel et al. reported on how patients with frequent migraine are misdiagnosed and how long it takes when they present with sinus symptoms. A total of 130 migraine patients were recruited for the study; of these, 81.5% were misdiagnosed with sinusitis. The mean time delay of migraine diagnosis was almost 8 years.5

In a study by Dr. Elina Kari and Dr. John M. DelGaudio, patients who had a history of “sinus headaches” were treated as though all these headaches were migraines. Fifty-four patients were enrolled, and 38 patients completed the study. All patients had nasal endoscopy and sinus CT scans that were negative. They were then given migraine-directed treatment to use for their headaches. Of the 38 patient who completed the study, 31 patients (82%) had a significant reduction in headache pain with triptan use, and 35 patients (92%) had a significant response to migraine-directed therapy.6 An expert panel consisting of otolaryngologists, neurologists, allergists, and primary care physicians concluded that the majority of sinus headaches can actually be classified as migraines.7

These references aren’t new. This information has been known in the medical literature for more than 2 decades, but I believe that the majority of medical professionals are not aware of it. In my own practice I have found great success treating patients with sinus headache histories with migraine-directed therapy (mostly triptans) when they have return of their headaches.


Pearl: When your patients say they have another sinus headache, think migraine.
 

Dr. Paauw is professor of medicine in the division of general internal medicine at the University of Washington, Seattle, and he serves as third-year medical student clerkship director at the University of Washington. He is a member of the editorial advisory board of Internal Medicine News. Dr. Paauw has no conflicts to disclose. Contact him at imnews@mdedge.com.

References

1. Jones NS. Expert Rev Neurother. 2009;9:439-44.

2. Schreiber CP et al. Arch Intern Med. 2004;164:1769-72.

3. Foroughipour M et al. Eur Arch Otorhinolaryngol. 2011;268:1593-6.

4. Obermann M et al. Cephalalgia. 2007 Jun;27(6):504-9.

5. Al-Hashel JY et al. J Headache Pain. 2013 Dec 12;14(1):97.

6. Kari E and DelGaudi JM. Laryngoscope. 2008;118:2235-9.

7. Levine HL et al. Otolaryngol Head Neck Surg. 2006 Mar;134(3):516-23.

Publications
Publications
Topics
Article Type
Sections
Teambase XML
<?xml version="1.0" encoding="UTF-8"?>
<!--$RCSfile: InCopy_agile.xsl,v $ $Revision: 1.35 $-->
<!--$RCSfile: drupal.xsl,v $ $Revision: 1.7 $-->
<root generator="drupal.xsl" gversion="1.7"> <header> <fileName>167446</fileName> <TBEID>0C04F3AD.SIG</TBEID> <TBUniqueIdentifier>MD_0C04F3AD</TBUniqueIdentifier> <newsOrJournal>News</newsOrJournal> <publisherName>Frontline Medical Communications</publisherName> <storyname/> <articleType>353</articleType> <TBLocation>QC Done-All Pubs</TBLocation> <QCDate>20240327T095021</QCDate> <firstPublished>20240327T100048</firstPublished> <LastPublished>20240327T100048</LastPublished> <pubStatus qcode="stat:"/> <embargoDate/> <killDate/> <CMSDate>20240327T100048</CMSDate> <articleSource/> <facebookInfo/> <meetingNumber/> <byline>Douglas S. Paauw</byline> <bylineText>DOUGLAS S. PAAUW, MD</bylineText> <bylineFull>DOUGLAS S. PAAUW, MD</bylineFull> <bylineTitleText/> <USOrGlobal/> <wireDocType/> <newsDocType/> <journalDocType/> <linkLabel/> <pageRange/> <citation/> <quizID/> <indexIssueDate/> <itemClass qcode="ninat:text"/> <provider qcode="provider:imng"> <name>IMNG Medical Media</name> <rightsInfo> <copyrightHolder> <name>Frontline Medical News</name> </copyrightHolder> <copyrightNotice>Copyright (c) 2015 Frontline Medical News, a Frontline Medical Communications Inc. company. All rights reserved. This material may not be published, broadcast, copied, or otherwise reproduced or distributed without the prior written permission of Frontline Medical Communications Inc.</copyrightNotice> </rightsInfo> </provider> <abstract/> <metaDescription>A 27-year-old woman presents requesting antibiotics for a sinus headache. She reports she has had 3-4 episodes a year with pain in her maxillary area and conges</metaDescription> <articlePDF/> <teaserImage>248215</teaserImage> <teaser>An expert panel consisting of otolaryngologists, neurologists, allergists, and primary care physicians concluded that the majority of sinus headaches can actually be classified as migraines.</teaser> <title>What is the Best Approach to “Sinus Headaches”?</title> <deck/> <disclaimer/> <AuthorList/> <articleURL/> <doi/> <pubMedID/> <publishXMLStatus/> <publishXMLVersion>1</publishXMLVersion> <useEISSN>0</useEISSN> <urgency/> <pubPubdateYear/> <pubPubdateMonth/> <pubPubdateDay/> <pubVolume/> <pubNumber/> <wireChannels/> <primaryCMSID/> <CMSIDs/> <keywords/> <seeAlsos/> <publications_g> <publicationData> <publicationCode>fp</publicationCode> <pubIssueName/> <pubArticleType/> <pubTopics/> <pubCategories/> <pubSections/> </publicationData> <publicationData> <publicationCode>im</publicationCode> <pubIssueName/> <pubArticleType/> <pubTopics/> <pubCategories/> <pubSections/> </publicationData> </publications_g> <publications> <term>15</term> <term canonical="true">21</term> </publications> <sections> <term canonical="true">39786</term> <term>41022</term> </sections> <topics> <term>268</term> <term canonical="true">258</term> </topics> <links> <link> <itemClass qcode="ninat:picture"/> <altRep contenttype="image/jpeg">images/2400c6a0.jpg</altRep> <description role="drol:caption">Dr. Douglas S. Paauw</description> <description role="drol:credit"/> </link> </links> </header> <itemSet> <newsItem> <itemMeta> <itemRole>Main</itemRole> <itemClass>text</itemClass> <title>What is the Best Approach to “Sinus Headaches”?</title> <deck/> </itemMeta> <itemContent> <p>A 27-year-old woman presents requesting antibiotics for a sinus headache. She reports she has had 3-4 episodes a year with pain in her maxillary area and congestion. She has not had fevers with these episodes. She had the onset of this headache 6 hours ago. She has had resolution of the pain within 24 hours in the past with the use of antibiotics and decongestants. What would be the best treatment for her?<br/><br/>A. Amoxicillin<br/><br/>B. Amoxicillin/clavulanate<br/><br/>C. Amoxicillin + fluticasone nasal spray<br/><br/>D. Sumatriptan</p> <p>The best treatment would be sumatriptan. This is very likely a variant of migraine headache and migraine-directed therapy is the best option. In regard to sinus headache, the International Headache Society (IHS) classification states that chronic sinusitis is not a cause of headache and facial pain unless it relapses into an acute sinusitis.<sup>1</sup> </p> <p>[[{"fid":"248215","view_mode":"medstat_image_flush_right","fields":{"format":"medstat_image_flush_right","field_file_image_alt_text[und][0][value]":"Dr. Douglas S. Paauw, University of Washington, Seattle","field_file_image_credit[und][0][value]":"","field_file_image_caption[und][0][value]":"Dr. Douglas S. Paauw"},"type":"media","attributes":{"class":"media-element file-medstat_image_flush_right"}}]]The recurrent nature of the headaches in this patient suggests a primary headache disorder with migraine being the most likely. In a study of 2991 patients with self-diagnosed or physician-diagnosed “sinus headaches,” 88% of the patients met IHS criteria for migraine.<sup>2</sup> In this study, most of the patients had symptoms suggesting sinus problems, with the most common symptoms being sinus pressure (84%), sinus pain (82%), and nasal congestion (63%). The likely cause for these symptoms in migraine patients is vasodilation of the nasal mucosa that can be part of the migraine event. <br/><br/>Foroughipour and colleagues found similar results.<sup>3 </sup>In their study, 58 patients with “sinus headache” were evaluated, with the final diagnosis of migraine in 40 patients (69%), tension-type headache in 16 patients (27%), and chronic sinusitis with recurrent acute episodes in 2 patients (3%). Recurrent antibiotic therapy had been given to 73% of the tension-type headache patients and 66% of the migraine patients.<br/><br/>Obermann et al. looked at how common trigeminal autonomic symptoms were in patients with migraine in a population-based study.<sup>4</sup> They found of 841 patients who had migraine, 226 reported accompanying unilateral trigeminal autonomic symptoms (26.9%).<br/><br/>Al-Hashel et al. reported on how patients with frequent migraine are misdiagnosed and how long it takes when they present with sinus symptoms. A total of 130 migraine patients were recruited for the study; of these, 81.5% were misdiagnosed with sinusitis. The mean time delay of migraine diagnosis was almost 8 years.<sup>5</sup><br/><br/>In a study by Dr. Elina Kari and Dr. John M. DelGaudio, patients who had a history of “sinus headaches” were treated as though all these headaches were migraines. Fifty-four patients were enrolled, and 38 patients completed the study. All patients had nasal endoscopy and sinus CT scans that were negative. They were then given migraine-directed treatment to use for their headaches. Of the 38 patient who completed the study, 31 patients (82%) had a significant reduction in headache pain with triptan use, and 35 patients (92%) had a significant response to migraine-directed therapy.<sup>6 </sup>An expert panel consisting of otolaryngologists, neurologists, allergists, and primary care physicians concluded that the majority of sinus headaches can actually be classified as migraines.<sup>7</sup><br/><br/>These references aren’t new. This information has been known in the medical literature for more than 2 decades, but I believe that the majority of medical professionals are not aware of it. In my own practice I have found great success treating patients with sinus headache histories with migraine-directed therapy (mostly triptans) when they have return of their headaches.<br/><br/><br/><br/><strong>Pearl: When your patients say they have another sinus headache, think migraine.<br/><br/></strong></p> <p> <em>Dr. Paauw is professor of medicine in the division of general internal medicine at the University of Washington, Seattle, and he serves as third-year medical student clerkship director at the University of Washington. He is a member of the editorial advisory board of Internal Medicine News. Dr. Paauw has no conflicts to disclose. Contact him at <span class="Hyperlink"><a href="mailto:imnews%40mdedge.com?subject=">imnews@mdedge.com</a></span>.</em> </p> <h2>References</h2> <p>1. Jones NS. <span class="Hyperlink"><a href="https://www.tandfonline.com/doi/full/10.1586/ern.09.8">Expert Rev Neurother. 2009;9:439-44</a></span>.<br/><br/>2. Schreiber CP et al. <span class="Hyperlink"><a href="https://jamanetwork.com/journals/jamainternalmedicine/fullarticle/217302">Arch Intern Med. 2004;164:1769-72</a></span>.<br/><br/>3. Foroughipour M et al. <span class="Hyperlink"><a href="https://link.springer.com/article/10.1007/s00405-011-1643-6">Eur Arch Otorhinolaryngol. 2011;268:1593-6</a></span>.<br/><br/>4. Obermann M et al. <span class="Hyperlink"><a href="https://journals.sagepub.com/doi/10.1111/j.1468-2982.2007.01316.x">Cephalalgia. 2007 Jun;27(6):504-9</a></span>.<br/><br/>5. Al-Hashel JY et al. <span class="Hyperlink"><a href="https://thejournalofheadacheandpain.biomedcentral.com/articles/10.1186/1129-2377-14-97">J Headache Pain. 2013 Dec 12;14(1):97</a></span>.<br/><br/>6. Kari E and DelGaudi JM. <span class="Hyperlink"><a href="https://onlinelibrary.wiley.com/doi/10.1097/MLG.0b013e318182f81d">Laryngoscope. 2008;118:2235-9</a></span>.<br/><br/>7. Levine HL et al. <span class="Hyperlink"><a href="https://aao-hnsfjournals.onlinelibrary.wiley.com/doi/10.1016/j.otohns.2005.11.024">Otolaryngol Head Neck Surg. 2006 Mar;134(3):516-23</a></span>.</p> </itemContent> </newsItem> <newsItem> <itemMeta> <itemRole>teaser</itemRole> <itemClass>text</itemClass> <title/> <deck/> </itemMeta> <itemContent> </itemContent> </newsItem> </itemSet></root>
Disallow All Ads
Content Gating
No Gating (article Unlocked/Free)
Alternative CME
Disqus Comments
Default
Use ProPublica
Hide sidebar & use full width
render the right sidebar.
Conference Recap Checkbox
Not Conference Recap
Clinical Edge
Display the Slideshow in this Article
Medscape Article
Display survey writer
Reuters content
Disable Inline Native ads
WebMD Article

Unexpectedly Helpful Effects of Drugs Used For Other Reasons

Article Type
Changed
Tue, 03/12/2024 - 17:23

A 73-year-old man with hypertension is evaluated for right great toe pain. A tap of the toe reveals uric acid crystals. He has a history of hypertension and hyperlipidemia. His current medications are hydrochlorothiazide, amlodipine, and atorvastatin.

Which blood pressure medication would you recommend to replace his hydrochlorothiazide?

A. Furosemide

B. Chlorthalidone

C. Lisinopril

D. Losartan

E. Irbesartan

Losartan

Diuretics should be avoided if possible in a patient with gout, as they increase uric acid levels. Of the other three options, losartan offers the added benefit of lowering uric acid levels. Losartan has uricosuric effects — a property that is unique to losartan of the angiotensin receptor blockers (ARBs) that have been studied.1,2 The uric acid lowering appears to be a probenecid-like effect.

Paauw_Doug_SEATTLE_2019_web.jpg
Dr. Douglas S. Paauw

Losartan has also been evaluated to see whether using it in combination with a thiazide diuretic can reduce the rise in uric acid that occurs with thiazides. Matsumura and colleagues looked at data from the COMFORT trial, focusing on the effect of combining losartan with hydrochlorothiazide on uric acid levels.3 They looked at a group of 118 patients on an ARB other than losartan plus a diuretic, who were then randomly assigned to losartan 50 mg/hydrochlorothiazide 12.5 mg or continuation of another ARB plus a diuretic. Blood pressure control was the same between groups, but the patients who received the losartan combination had lower uric acid levels (P = .01).

Ferreira and colleagues looked at the difference in uric acid lowering between high-dose (150 mg/day) vs low-dose losartan (50 mg/day).4 Compared with low-dose, high-dose losartan reduced serum uric acid by 0.27 (0.34 to 0.21) mg/dL, P < .001.
 

SGLT2 inhibitors

SGLT2 inhibitors also lower uric acid. Suijik and colleagues conducted an analysis of two randomized trials of SGLT2 inhibitors (empagliflozin and dapagliflozin), and concluded that SGLT2 inhibitors induce uric acid excretion, which is strongly linked to urinary glucose excretion.5

Metformin

Metformin is used as a firstline drug for the treatment of diabetes. It also has evidence for decreasing colonic polyps. Cho and colleagues looked at over 12,000 patients with diabetes over a 12-year period; 3775 underwent colonoscopies.6 They compared frequency of polyps in patients who were using metformin with those who were not treated with metformin. The polyp detection rate was lower in the metformin group than in the no metformin group (39.4% vs. 62.4%, P < .01).

Higurashi and colleagues performed a double-blind, placebo-controlled trial of metformin in nondiabetic patients for the prevention of colon polyps.7 The dose of metformin used in this study was very low (250 mg/day). There were significantly fewer adenomas in the metformin group (22 of 71 patients) than in the placebo group (32 of 62) (relative risk, 0.60; 95% confidence interval, 0.39-0.92, P = .016).
 

Thiazide diuretics

Thiazide diuretics have long been used to help prevent kidney stones in addition to treating hypertension. They decrease urinary calcium excretion, which may reduce kidney stones. Could this reduction in calcium excretion be good for bones?

Xiao and colleagues did a meta-analysis of 11 prospective studies involving 2,193,160 participants.8 Thiazide diuretic users had a significant 14% reduction in the risk of all fractures (RR, 0.86; 95% CI, 0.80-0.93; P = .009) and an 18% reduction in the risk of hip fracture (RR, 0.82; 95% CI, 0.80-0.93; P = .009). Kruse and colleagues found that long duration and continuity of thiazide exposure seemed to be important to obtain this protective effect on fracture risk.9

Pearls:

  • Losartan, but not other ARBs, lowers uric acid levels and may be helpful in managing hypertension in gout patients; higher doses lower uric acid more.
  • Metformin use appears to decrease colon polyp formation.
  • Thiazide diuretics may reduce fracture risk while patients are taking them.

Dr. Paauw is professor of medicine in the division of general internal medicine at the University of Washington, Seattle, and he serves as third-year medical student clerkship director at the University of Washington. He is a member of the editorial advisory board of Internal Medicine News. Dr. Paauw has no conflicts to disclose. Contact him at imnews@mdedge.com.

References

1. Würzner G et al. Comparative effects of losartan and irbesartan on serum uric acid in hypertensive patients with hyperuricaemia and gout. J Hypertens. 2001 Oct;19(10):1855-60.

2. Puig JG et al. Effect of eprosartan and losartan on uric acid metabolism in patients with essential hypertension. J Hypertens. 1999 Jul;17(7):1033-9.

3. Matsumura K et al. Effect of losartan on serum uric acid in hypertension treated with a diuretic: The COMFORT study. Clin Exp Hypertens. 2015;37(3):192-6.

4. Ferreira JP et al. High- versus low-dose losartan and uric acid: An analysis from HEAAL. J Cardiol. 2023 Jul;82(1):57-61.

5. Suijk DLS et al. SGLT2 inhibition and uric acid excretion in patients with type 2 diabetes and normal kidney function. Soc Nephrol. 2022 May;17(5):663-71.

6. Youn Hee Cho et al. Does metformin affect the incidence of colonic polyps and adenomas in patients with type 2 diabetes mellitus? Intestinal Res. 2014 Apr;12(2):139-45.

7. Higurashi T et al. Metformin for chemoprevention of metachronous colorectal adenoma or polyps in post-polypectomy patients without diabetes: A multicentre double-blind, placebo-controlled, randomised phase 3 trial. Lancet Oncol. 2016;17:475-83.

8. Xiao X et al. Thiazide diuretic usage and risk of fracture: a meta-analysis of cohort studies. Osteoporos Int. 2018 Jul;29(7):1515-24.

9. Kruse C et al. Continuous and long-term treatment is more important than dosage for the protective effect of thiazide use on bone metabolism and fracture risk. J Intern Med. 2016 Jan;279(1):110-22.

Publications
Topics
Sections

A 73-year-old man with hypertension is evaluated for right great toe pain. A tap of the toe reveals uric acid crystals. He has a history of hypertension and hyperlipidemia. His current medications are hydrochlorothiazide, amlodipine, and atorvastatin.

Which blood pressure medication would you recommend to replace his hydrochlorothiazide?

A. Furosemide

B. Chlorthalidone

C. Lisinopril

D. Losartan

E. Irbesartan

Losartan

Diuretics should be avoided if possible in a patient with gout, as they increase uric acid levels. Of the other three options, losartan offers the added benefit of lowering uric acid levels. Losartan has uricosuric effects — a property that is unique to losartan of the angiotensin receptor blockers (ARBs) that have been studied.1,2 The uric acid lowering appears to be a probenecid-like effect.

Paauw_Doug_SEATTLE_2019_web.jpg
Dr. Douglas S. Paauw

Losartan has also been evaluated to see whether using it in combination with a thiazide diuretic can reduce the rise in uric acid that occurs with thiazides. Matsumura and colleagues looked at data from the COMFORT trial, focusing on the effect of combining losartan with hydrochlorothiazide on uric acid levels.3 They looked at a group of 118 patients on an ARB other than losartan plus a diuretic, who were then randomly assigned to losartan 50 mg/hydrochlorothiazide 12.5 mg or continuation of another ARB plus a diuretic. Blood pressure control was the same between groups, but the patients who received the losartan combination had lower uric acid levels (P = .01).

Ferreira and colleagues looked at the difference in uric acid lowering between high-dose (150 mg/day) vs low-dose losartan (50 mg/day).4 Compared with low-dose, high-dose losartan reduced serum uric acid by 0.27 (0.34 to 0.21) mg/dL, P < .001.
 

SGLT2 inhibitors

SGLT2 inhibitors also lower uric acid. Suijik and colleagues conducted an analysis of two randomized trials of SGLT2 inhibitors (empagliflozin and dapagliflozin), and concluded that SGLT2 inhibitors induce uric acid excretion, which is strongly linked to urinary glucose excretion.5

Metformin

Metformin is used as a firstline drug for the treatment of diabetes. It also has evidence for decreasing colonic polyps. Cho and colleagues looked at over 12,000 patients with diabetes over a 12-year period; 3775 underwent colonoscopies.6 They compared frequency of polyps in patients who were using metformin with those who were not treated with metformin. The polyp detection rate was lower in the metformin group than in the no metformin group (39.4% vs. 62.4%, P < .01).

Higurashi and colleagues performed a double-blind, placebo-controlled trial of metformin in nondiabetic patients for the prevention of colon polyps.7 The dose of metformin used in this study was very low (250 mg/day). There were significantly fewer adenomas in the metformin group (22 of 71 patients) than in the placebo group (32 of 62) (relative risk, 0.60; 95% confidence interval, 0.39-0.92, P = .016).
 

Thiazide diuretics

Thiazide diuretics have long been used to help prevent kidney stones in addition to treating hypertension. They decrease urinary calcium excretion, which may reduce kidney stones. Could this reduction in calcium excretion be good for bones?

Xiao and colleagues did a meta-analysis of 11 prospective studies involving 2,193,160 participants.8 Thiazide diuretic users had a significant 14% reduction in the risk of all fractures (RR, 0.86; 95% CI, 0.80-0.93; P = .009) and an 18% reduction in the risk of hip fracture (RR, 0.82; 95% CI, 0.80-0.93; P = .009). Kruse and colleagues found that long duration and continuity of thiazide exposure seemed to be important to obtain this protective effect on fracture risk.9

Pearls:

  • Losartan, but not other ARBs, lowers uric acid levels and may be helpful in managing hypertension in gout patients; higher doses lower uric acid more.
  • Metformin use appears to decrease colon polyp formation.
  • Thiazide diuretics may reduce fracture risk while patients are taking them.

Dr. Paauw is professor of medicine in the division of general internal medicine at the University of Washington, Seattle, and he serves as third-year medical student clerkship director at the University of Washington. He is a member of the editorial advisory board of Internal Medicine News. Dr. Paauw has no conflicts to disclose. Contact him at imnews@mdedge.com.

References

1. Würzner G et al. Comparative effects of losartan and irbesartan on serum uric acid in hypertensive patients with hyperuricaemia and gout. J Hypertens. 2001 Oct;19(10):1855-60.

2. Puig JG et al. Effect of eprosartan and losartan on uric acid metabolism in patients with essential hypertension. J Hypertens. 1999 Jul;17(7):1033-9.

3. Matsumura K et al. Effect of losartan on serum uric acid in hypertension treated with a diuretic: The COMFORT study. Clin Exp Hypertens. 2015;37(3):192-6.

4. Ferreira JP et al. High- versus low-dose losartan and uric acid: An analysis from HEAAL. J Cardiol. 2023 Jul;82(1):57-61.

5. Suijk DLS et al. SGLT2 inhibition and uric acid excretion in patients with type 2 diabetes and normal kidney function. Soc Nephrol. 2022 May;17(5):663-71.

6. Youn Hee Cho et al. Does metformin affect the incidence of colonic polyps and adenomas in patients with type 2 diabetes mellitus? Intestinal Res. 2014 Apr;12(2):139-45.

7. Higurashi T et al. Metformin for chemoprevention of metachronous colorectal adenoma or polyps in post-polypectomy patients without diabetes: A multicentre double-blind, placebo-controlled, randomised phase 3 trial. Lancet Oncol. 2016;17:475-83.

8. Xiao X et al. Thiazide diuretic usage and risk of fracture: a meta-analysis of cohort studies. Osteoporos Int. 2018 Jul;29(7):1515-24.

9. Kruse C et al. Continuous and long-term treatment is more important than dosage for the protective effect of thiazide use on bone metabolism and fracture risk. J Intern Med. 2016 Jan;279(1):110-22.

A 73-year-old man with hypertension is evaluated for right great toe pain. A tap of the toe reveals uric acid crystals. He has a history of hypertension and hyperlipidemia. His current medications are hydrochlorothiazide, amlodipine, and atorvastatin.

Which blood pressure medication would you recommend to replace his hydrochlorothiazide?

A. Furosemide

B. Chlorthalidone

C. Lisinopril

D. Losartan

E. Irbesartan

Losartan

Diuretics should be avoided if possible in a patient with gout, as they increase uric acid levels. Of the other three options, losartan offers the added benefit of lowering uric acid levels. Losartan has uricosuric effects — a property that is unique to losartan of the angiotensin receptor blockers (ARBs) that have been studied.1,2 The uric acid lowering appears to be a probenecid-like effect.

Paauw_Doug_SEATTLE_2019_web.jpg
Dr. Douglas S. Paauw

Losartan has also been evaluated to see whether using it in combination with a thiazide diuretic can reduce the rise in uric acid that occurs with thiazides. Matsumura and colleagues looked at data from the COMFORT trial, focusing on the effect of combining losartan with hydrochlorothiazide on uric acid levels.3 They looked at a group of 118 patients on an ARB other than losartan plus a diuretic, who were then randomly assigned to losartan 50 mg/hydrochlorothiazide 12.5 mg or continuation of another ARB plus a diuretic. Blood pressure control was the same between groups, but the patients who received the losartan combination had lower uric acid levels (P = .01).

Ferreira and colleagues looked at the difference in uric acid lowering between high-dose (150 mg/day) vs low-dose losartan (50 mg/day).4 Compared with low-dose, high-dose losartan reduced serum uric acid by 0.27 (0.34 to 0.21) mg/dL, P < .001.
 

SGLT2 inhibitors

SGLT2 inhibitors also lower uric acid. Suijik and colleagues conducted an analysis of two randomized trials of SGLT2 inhibitors (empagliflozin and dapagliflozin), and concluded that SGLT2 inhibitors induce uric acid excretion, which is strongly linked to urinary glucose excretion.5

Metformin

Metformin is used as a firstline drug for the treatment of diabetes. It also has evidence for decreasing colonic polyps. Cho and colleagues looked at over 12,000 patients with diabetes over a 12-year period; 3775 underwent colonoscopies.6 They compared frequency of polyps in patients who were using metformin with those who were not treated with metformin. The polyp detection rate was lower in the metformin group than in the no metformin group (39.4% vs. 62.4%, P < .01).

Higurashi and colleagues performed a double-blind, placebo-controlled trial of metformin in nondiabetic patients for the prevention of colon polyps.7 The dose of metformin used in this study was very low (250 mg/day). There were significantly fewer adenomas in the metformin group (22 of 71 patients) than in the placebo group (32 of 62) (relative risk, 0.60; 95% confidence interval, 0.39-0.92, P = .016).
 

Thiazide diuretics

Thiazide diuretics have long been used to help prevent kidney stones in addition to treating hypertension. They decrease urinary calcium excretion, which may reduce kidney stones. Could this reduction in calcium excretion be good for bones?

Xiao and colleagues did a meta-analysis of 11 prospective studies involving 2,193,160 participants.8 Thiazide diuretic users had a significant 14% reduction in the risk of all fractures (RR, 0.86; 95% CI, 0.80-0.93; P = .009) and an 18% reduction in the risk of hip fracture (RR, 0.82; 95% CI, 0.80-0.93; P = .009). Kruse and colleagues found that long duration and continuity of thiazide exposure seemed to be important to obtain this protective effect on fracture risk.9

Pearls:

  • Losartan, but not other ARBs, lowers uric acid levels and may be helpful in managing hypertension in gout patients; higher doses lower uric acid more.
  • Metformin use appears to decrease colon polyp formation.
  • Thiazide diuretics may reduce fracture risk while patients are taking them.

Dr. Paauw is professor of medicine in the division of general internal medicine at the University of Washington, Seattle, and he serves as third-year medical student clerkship director at the University of Washington. He is a member of the editorial advisory board of Internal Medicine News. Dr. Paauw has no conflicts to disclose. Contact him at imnews@mdedge.com.

References

1. Würzner G et al. Comparative effects of losartan and irbesartan on serum uric acid in hypertensive patients with hyperuricaemia and gout. J Hypertens. 2001 Oct;19(10):1855-60.

2. Puig JG et al. Effect of eprosartan and losartan on uric acid metabolism in patients with essential hypertension. J Hypertens. 1999 Jul;17(7):1033-9.

3. Matsumura K et al. Effect of losartan on serum uric acid in hypertension treated with a diuretic: The COMFORT study. Clin Exp Hypertens. 2015;37(3):192-6.

4. Ferreira JP et al. High- versus low-dose losartan and uric acid: An analysis from HEAAL. J Cardiol. 2023 Jul;82(1):57-61.

5. Suijk DLS et al. SGLT2 inhibition and uric acid excretion in patients with type 2 diabetes and normal kidney function. Soc Nephrol. 2022 May;17(5):663-71.

6. Youn Hee Cho et al. Does metformin affect the incidence of colonic polyps and adenomas in patients with type 2 diabetes mellitus? Intestinal Res. 2014 Apr;12(2):139-45.

7. Higurashi T et al. Metformin for chemoprevention of metachronous colorectal adenoma or polyps in post-polypectomy patients without diabetes: A multicentre double-blind, placebo-controlled, randomised phase 3 trial. Lancet Oncol. 2016;17:475-83.

8. Xiao X et al. Thiazide diuretic usage and risk of fracture: a meta-analysis of cohort studies. Osteoporos Int. 2018 Jul;29(7):1515-24.

9. Kruse C et al. Continuous and long-term treatment is more important than dosage for the protective effect of thiazide use on bone metabolism and fracture risk. J Intern Med. 2016 Jan;279(1):110-22.

Publications
Publications
Topics
Article Type
Sections
Teambase XML
<?xml version="1.0" encoding="UTF-8"?>
<!--$RCSfile: InCopy_agile.xsl,v $ $Revision: 1.35 $-->
<!--$RCSfile: drupal.xsl,v $ $Revision: 1.7 $-->
<root generator="drupal.xsl" gversion="1.7"> <header> <fileName>167093</fileName> <TBEID>0C04EC6D.SIG</TBEID> <TBUniqueIdentifier>MD_0C04EC6D</TBUniqueIdentifier> <newsOrJournal>News</newsOrJournal> <publisherName>Frontline Medical Communications</publisherName> <storyname/> <articleType>353</articleType> <TBLocation>QC Done-All Pubs</TBLocation> <QCDate>20240301T102900</QCDate> <firstPublished>20240301T110105</firstPublished> <LastPublished>20240301T110105</LastPublished> <pubStatus qcode="stat:"/> <embargoDate/> <killDate/> <CMSDate>20240301T110105</CMSDate> <articleSource/> <facebookInfo/> <meetingNumber/> <byline>Douglas S. Paauw</byline> <bylineText>DOUGLAS S. PAAUW, MD</bylineText> <bylineFull>DOUGLAS S. PAAUW, MD</bylineFull> <bylineTitleText/> <USOrGlobal/> <wireDocType/> <newsDocType/> <journalDocType/> <linkLabel/> <pageRange/> <citation/> <quizID/> <indexIssueDate/> <itemClass qcode="ninat:text"/> <provider qcode="provider:imng"> <name>IMNG Medical Media</name> <rightsInfo> <copyrightHolder> <name>Frontline Medical News</name> </copyrightHolder> <copyrightNotice>Copyright (c) 2015 Frontline Medical News, a Frontline Medical Communications Inc. company. All rights reserved. This material may not be published, broadcast, copied, or otherwise reproduced or distributed without the prior written permission of Frontline Medical Communications Inc.</copyrightNotice> </rightsInfo> </provider> <abstract/> <metaDescription>A 73-year-old man with hypertension is evaluated for right great toe pain. A tap of the toe reveals uric acid crystals. He has a history of hypertension and hyp</metaDescription> <articlePDF/> <teaserImage>248215</teaserImage> <teaser>Instead of negative interactions, some drugs have beneficial effects in areas other than their indicated use. </teaser> <title>Unexpectedly Helpful Effects of Drugs Used For Other Reasons</title> <deck/> <disclaimer/> <AuthorList/> <articleURL/> <doi/> <pubMedID/> <publishXMLStatus/> <publishXMLVersion>1</publishXMLVersion> <useEISSN>0</useEISSN> <urgency/> <pubPubdateYear/> <pubPubdateMonth/> <pubPubdateDay/> <pubVolume/> <pubNumber/> <wireChannels/> <primaryCMSID/> <CMSIDs/> <keywords/> <seeAlsos/> <publications_g> <publicationData> <publicationCode>fp</publicationCode> <pubIssueName/> <pubArticleType/> <pubTopics/> <pubCategories/> <pubSections/> </publicationData> <publicationData> <publicationCode>im</publicationCode> <pubIssueName/> <pubArticleType/> <pubTopics/> <pubCategories/> <pubSections/> </publicationData> </publications_g> <publications> <term>15</term> <term canonical="true">21</term> </publications> <sections> <term>52</term> <term canonical="true">39786</term> </sections> <topics> <term>263</term> <term>194</term> <term>252</term> <term canonical="true">290</term> </topics> <links> <link> <itemClass qcode="ninat:picture"/> <altRep contenttype="image/jpeg">images/2400c6a0.jpg</altRep> <description role="drol:caption">Dr. Douglas S. Paauw</description> <description role="drol:credit"/> </link> </links> </header> <itemSet> <newsItem> <itemMeta> <itemRole>Main</itemRole> <itemClass>text</itemClass> <title>Unexpectedly Helpful Effects of Drugs Used For Other Reasons</title> <deck/> </itemMeta> <itemContent> <p>A 73-year-old man with hypertension is evaluated for right great toe pain. A tap of the toe reveals uric acid crystals. He has a history of hypertension and hyperlipidemia. His current medications are hydrochlorothiazide, amlodipine, and atorvastatin. </p> <p>Which blood pressure medication would you recommend to replace his hydrochlorothiazide?</p> <p><strong>A.</strong> Furosemide<br/><br/><strong>B.</strong> Chlorthalidone<br/><br/><strong>C.</strong> Lisinopril<br/><br/><strong>D.</strong> Losartan<br/><br/><strong>E.</strong> Irbesartan</p> <h2>Losartan</h2> <p>Diuretics should be avoided if possible in a patient with gout, as they increase uric acid levels. Of the other three options, losartan offers the added benefit of lowering uric acid levels. Losartan has uricosuric effects — a property that is unique to losartan of the angiotensin receptor blockers (ARBs) that have been studied.<sup>1,2</sup> The uric acid lowering appears to be a probenecid-like effect. </p> <p>[[{"fid":"248215","view_mode":"medstat_image_flush_right","fields":{"format":"medstat_image_flush_right","field_file_image_alt_text[und][0][value]":"Dr. Douglas S. Paauw, University of Washington, Seattle","field_file_image_credit[und][0][value]":"","field_file_image_caption[und][0][value]":"Dr. Douglas S. Paauw"},"type":"media","attributes":{"class":"media-element file-medstat_image_flush_right"}}]]Losartan has also been evaluated to see whether using it in combination with a thiazide diuretic can reduce the rise in uric acid that occurs with thiazides. Matsumura and colleagues looked at data from the COMFORT trial, focusing on the effect of combining losartan with hydrochlorothiazide on uric acid levels.<sup>3</sup> They looked at a group of 118 patients on an ARB other than losartan plus a diuretic, who were then randomly assigned to losartan 50 mg/hydrochlorothiazide 12.5 mg or continuation of another ARB plus a diuretic. Blood pressure control was the same between groups, but the patients who received the losartan combination had lower uric acid levels (<em>P</em> = .01). <br/><br/>Ferreira and colleagues looked at the difference in uric acid lowering between high-dose (150 mg/day) vs low-dose losartan (50 mg/day).<sup>4</sup> Compared with low-dose, high-dose losartan reduced serum uric acid by 0.27 (0.34 to 0.21) mg/dL, <em>P</em> &lt; .001.<br/><br/></p> <h2>SGLT2 inhibitors </h2> <p>SGLT2 inhibitors also lower uric acid. Suijik and colleagues conducted an analysis of two randomized trials of SGLT2 inhibitors (empagliflozin and dapagliflozin), and concluded that SGLT2 inhibitors induce uric acid excretion, which is strongly linked to urinary glucose excretion.<sup>5</sup></p> <h2>Metformin</h2> <p>Metformin is used as a firstline drug for the treatment of diabetes. It also has evidence for decreasing colonic polyps. Cho and colleagues looked at over 12,000 patients with diabetes over a 12-year period; 3775 underwent colonoscopies.<sup>6</sup> They compared frequency of polyps in patients who were using metformin with those who were not treated with metformin. The polyp detection rate was lower in the metformin group than in the no metformin group (39.4% vs. 62.4%, <em>P</em> &lt; .01). </p> <p>Higurashi and colleagues performed a double-blind, placebo-controlled trial of metformin in nondiabetic patients for the prevention of colon polyps.<sup>7</sup> The dose of metformin used in this study was very low (250 mg/day). There were significantly fewer adenomas in the metformin group (22 of 71 patients) than in the placebo group (32 of 62) (relative risk, 0.60; 95% confidence interval, 0.39-0.92, <em>P</em> = .016).<br/><br/></p> <h2>Thiazide diuretics</h2> <p>Thiazide diuretics have long been used to help prevent kidney stones in addition to treating hypertension. They decrease urinary calcium excretion, which may reduce kidney stones. Could this reduction in calcium excretion be good for bones? </p> <p>Xiao and colleagues did a meta-analysis of 11 prospective studies involving 2,193,160 participants.<sup>8</sup> Thiazide diuretic users had a significant 14% reduction in the risk of all fractures (RR, 0.86; 95% CI, 0.80-0.93; <em>P</em> = .009) and an 18% reduction in the risk of hip fracture (RR, 0.82; 95% CI, 0.80-0.93; <em>P</em> = .009). Kruse and colleagues found that long duration and continuity of thiazide exposure seemed to be important to obtain this protective effect on fracture risk.<sup>9</sup></p> <p> <strong>Pearls:</strong> </p> <ul class="body"> <li>Losartan, but not other ARBs, lowers uric acid levels and may be helpful in managing hypertension in gout patients; higher doses lower uric acid more.</li> <li>Metformin use appears to decrease colon polyp formation.</li> <li>Thiazide diuretics may reduce fracture risk while patients are taking them.</li> <li/> </ul> <p> <em>Dr. Paauw is professor of medicine in the division of general internal medicine at the University of Washington, Seattle, and he serves as third-year medical student clerkship director at the University of Washington. He is a member of the editorial advisory board of Internal Medicine News. Dr. Paauw has no conflicts to disclose. Contact him at <span class="Hyperlink"><a href="mailto:imnews%40mdedge.com?subject=">imnews@mdedge.com</a></span>.</em> </p> <h2>References</h2> <p>1. Würzner G et al. Comparative effects of losartan and irbesartan on serum uric acid in hypertensive patients with hyperuricaemia and gout. <span class="Hyperlink"><a href="https://journals.lww.com/jhypertension/abstract/2001/10000/comparative_effects_of_losartan_and_irbesartan_on.21.aspx">J Hypertens. 2001 Oct;19(10):1855-60</a></span>.<br/><br/>2. Puig JG et al. Effect of eprosartan and losartan on uric acid metabolism in patients with essential hypertension. <span class="Hyperlink"><a href="https://journals.lww.com/jhypertension/abstract/1999/17070/effect_of_eprosartan_and_losartan_on_uric_acid.21.aspx">J Hypertens. 1999 Jul;17(7):1033-9</a></span>.<br/><br/>3. Matsumura K et al. Effect of losartan on serum uric acid in hypertension treated with a diuretic: The COMFORT study. <span class="Hyperlink"><a href="https://www.tandfonline.com/doi/full/10.3109/10641963.2014.933968">Clin Exp Hypertens. 2015;37(3):192-6</a></span>.<br/><br/>4. Ferreira JP et al. High- versus low-dose losartan and uric acid: An analysis from HEAAL. <span class="Hyperlink"><a href="https://www.journal-of-cardiology.com/article/S0914-5087(23)00068-0/abstract">J Cardiol. 2023 Jul;82(1):57-61</a></span>.<br/><br/>5. Suijk DLS et al. SGLT2 inhibition and uric acid excretion in patients with type 2 diabetes and normal kidney function. <span class="Hyperlink"><a href="https://journals.lww.com/cjasn/fulltext/2022/05000/sglt2_inhibition_and_uric_acid_excretion_in.8.aspx">Soc Nephrol. 2022 May;17(5):663-71</a></span>.<br/><br/>6. Youn Hee Cho et al. Does metformin affect the incidence of colonic polyps and adenomas in patients with type 2 diabetes mellitus? <span class="Hyperlink"><a href="https://www.irjournal.org/journal/view.php?doi=10.5217/ir.2014.12.2.139">Intestinal Res. 2014 Apr;12(2):139-45</a></span>.<br/><br/>7. Higurashi T et al. Metformin for chemoprevention of metachronous colorectal adenoma or polyps in post-polypectomy patients without diabetes: A multicentre double-blind, placebo-controlled, randomised phase 3 trial. <span class="Hyperlink"><a href="https://www.thelancet.com/journals/lanonc/article/PIIS1470-2045(15)00565-3/abstract">Lancet Oncol. 2016;17:475-83</a></span>. <br/><br/>8. Xiao X et al. Thiazide diuretic usage and risk of fracture: a meta-analysis of cohort studies. <span class="Hyperlink"><a href="https://link.springer.com/article/10.1007/s00198-018-4486-9">Osteoporos Int. 2018 Jul;29(7):1515-24</a></span>.<br/><br/>9. Kruse C et al. Continuous and long-term treatment is more important than dosage for the protective effect of thiazide use on bone metabolism and fracture risk. <span class="Hyperlink"><a href="https://onlinelibrary.wiley.com/doi/10.1111/joim.12397">J Intern Med. 2016 Jan;279(1):110-22</a></span>.</p> </itemContent> </newsItem> <newsItem> <itemMeta> <itemRole>teaser</itemRole> <itemClass>text</itemClass> <title/> <deck/> </itemMeta> <itemContent> </itemContent> </newsItem> </itemSet></root>
Disallow All Ads
Content Gating
No Gating (article Unlocked/Free)
Alternative CME
Disqus Comments
Default
Use ProPublica
Hide sidebar & use full width
render the right sidebar.
Conference Recap Checkbox
Not Conference Recap
Clinical Edge
Display the Slideshow in this Article
Medscape Article
Display survey writer
Reuters content
Disable Inline Native ads
WebMD Article

Guidelines Aren’t For Everybody

Article Type
Changed
Tue, 02/06/2024 - 11:47

An 88-year-old man comes for clinic follow up. He has a medical history of type 2 diabetes, hypertension, heart failure with reduced ejection fraction, and chronic kidney disease. He recently had laboratory tests done: BUN, 32 mg/dL; creatinine, 2.3 mg/dL; potassium, 4.5 mmol/L; bicarbonate, 22 Eq/L; and A1c, 8.2%.

He checks his blood glucose daily (alternating between fasting blood glucose and before dinner) and his fasting blood glucose levels are around 130 mg/dL. His highest glucose reading was 240 mg/dL. He does not have polyuria or visual changes. Current medications: atorvastatin, irbesartan, empagliflozin, and amlodipine. On physical exam his blood pressure is 130/70 mm Hg, pulse is 80, and his BMI 20.

What medication adjustments would you recommend?

A. Begin insulin glargine at bedtime

B. Begin mealtime insulin aspart

C. Begin semaglutide

D. Begin metformin

E. No changes

I think the correct approach here would be no changes. Most physicians know guideline recommendations for A1c of less than 7% are used for patients with diabetes with few comorbid conditions, normal cognition, and functional status. Many of our elderly patients do not meet these criteria and the goal of intense medical treatment of diabetes is different in those patients. The American Diabetes Association has issued a thoughtful paper on treatment of diabetes in elderly people, stressing that patients should have very individualized goals, and that there is no one-size-fits all A1c goal.1

Paauw_Doug_SEATTLE_2019_web.jpg
Dr. Douglas S. Paauw

In this patient I would avoid adding insulin, given hypoglycemia risk. A GLP-1 agonist might appear attractive given his multiple cardiovascular risk factors, but his low BMI is a major concern for frailty that may well be worsened with reduced nutrient intake. Diabetes is the chronic condition that probably has the most guidance for management in elderly patients.

I recently saw a 92-year-old man with heart failure with reduced ejection fraction and atrial fibrillation who had been losing weight and becoming weaker. He had suffered several falls in the previous 2 weeks. His medication list included amiodarone, apixaban, sacubitril/valsartan, carvedilol, empagliflozin, spironolactone, and furosemide. He was extremely frail and had stopped eating. He was receiving all guideline-directed therapies, yet he was miserable and dying. Falls in this population are potentially as fatal as decompensated heart disease.

I stopped his amiodarone, furosemide, and spironolactone, and reduced his doses of sacubitril/valsartan and carvedilol. His appetite returned and his will to live returned. Heart failure guidelines do not include robust studies of very elderly patients because few studies exist in this population. Frailty assessment is crucial in decision making in your elderly patients.2,3 and frequent check-ins to make sure that they are not suffering from the effects of polypharmacy are crucial. Our goal in our very elderly patients is quality life-years. Polypharmacy has the potential to decrease the quality of life, as well as potentially shorten life.

The very elderly are at risk of the negative consequences of polypharmacy, especially if they have several diseases like diabetes, congestive heart failure, and hypertension that may require multiple medications. Gutierrez-Valencia and colleagues performed a systematic review of 25 articles on frailty and polypharmacy.4 Their findings demonstrated a significant association between an increased number of medications and frailty. They postulated that polypharmacy could actually be a contributor to frailty. There just isn’t enough evidence for the benefit of guidelines in the very aged and the risks of polypharmacy are real. We should use the lowest possible doses of medications in this population, frequently reassess goals, and monitor closely for side effects.


Pearl: Always consider the risks of polypharmacy when considering therapies for your elderly patients.
 

Dr. Paauw is professor of medicine in the division of general internal medicine at the University of Washington, Seattle, and he serves as third-year medical student clerkship director at the University of Washington. Contact Dr. Paauw at dpaauw@uw.edu.

References

1. Older Adults: Standards of Medical Care in Diabetes — 2021. Diabetes Care 2021;44(Suppl 1):S168–S179.

2. Gaur A et al. Cardiogeriatrics: The current state of the art. Heart. 2024 Jan 11:heartjnl-2022-322117.

3. Denfeld QE et al. Assessing and managing frailty in advanced heart failure: An International Society for Heart and Lung Transplantation consensus statement. J Heart Lung Transplant. 2023 Nov 29:S1053-2498(23)02028-4.

4. Gutiérrez-Valencia M et al. The relationship between frailty and polypharmacy in older people: A systematic review. Br J Clin Pharmacol. 2018 Jul;84(7):1432-44.

Publications
Topics
Sections

An 88-year-old man comes for clinic follow up. He has a medical history of type 2 diabetes, hypertension, heart failure with reduced ejection fraction, and chronic kidney disease. He recently had laboratory tests done: BUN, 32 mg/dL; creatinine, 2.3 mg/dL; potassium, 4.5 mmol/L; bicarbonate, 22 Eq/L; and A1c, 8.2%.

He checks his blood glucose daily (alternating between fasting blood glucose and before dinner) and his fasting blood glucose levels are around 130 mg/dL. His highest glucose reading was 240 mg/dL. He does not have polyuria or visual changes. Current medications: atorvastatin, irbesartan, empagliflozin, and amlodipine. On physical exam his blood pressure is 130/70 mm Hg, pulse is 80, and his BMI 20.

What medication adjustments would you recommend?

A. Begin insulin glargine at bedtime

B. Begin mealtime insulin aspart

C. Begin semaglutide

D. Begin metformin

E. No changes

I think the correct approach here would be no changes. Most physicians know guideline recommendations for A1c of less than 7% are used for patients with diabetes with few comorbid conditions, normal cognition, and functional status. Many of our elderly patients do not meet these criteria and the goal of intense medical treatment of diabetes is different in those patients. The American Diabetes Association has issued a thoughtful paper on treatment of diabetes in elderly people, stressing that patients should have very individualized goals, and that there is no one-size-fits all A1c goal.1

Paauw_Doug_SEATTLE_2019_web.jpg
Dr. Douglas S. Paauw

In this patient I would avoid adding insulin, given hypoglycemia risk. A GLP-1 agonist might appear attractive given his multiple cardiovascular risk factors, but his low BMI is a major concern for frailty that may well be worsened with reduced nutrient intake. Diabetes is the chronic condition that probably has the most guidance for management in elderly patients.

I recently saw a 92-year-old man with heart failure with reduced ejection fraction and atrial fibrillation who had been losing weight and becoming weaker. He had suffered several falls in the previous 2 weeks. His medication list included amiodarone, apixaban, sacubitril/valsartan, carvedilol, empagliflozin, spironolactone, and furosemide. He was extremely frail and had stopped eating. He was receiving all guideline-directed therapies, yet he was miserable and dying. Falls in this population are potentially as fatal as decompensated heart disease.

I stopped his amiodarone, furosemide, and spironolactone, and reduced his doses of sacubitril/valsartan and carvedilol. His appetite returned and his will to live returned. Heart failure guidelines do not include robust studies of very elderly patients because few studies exist in this population. Frailty assessment is crucial in decision making in your elderly patients.2,3 and frequent check-ins to make sure that they are not suffering from the effects of polypharmacy are crucial. Our goal in our very elderly patients is quality life-years. Polypharmacy has the potential to decrease the quality of life, as well as potentially shorten life.

The very elderly are at risk of the negative consequences of polypharmacy, especially if they have several diseases like diabetes, congestive heart failure, and hypertension that may require multiple medications. Gutierrez-Valencia and colleagues performed a systematic review of 25 articles on frailty and polypharmacy.4 Their findings demonstrated a significant association between an increased number of medications and frailty. They postulated that polypharmacy could actually be a contributor to frailty. There just isn’t enough evidence for the benefit of guidelines in the very aged and the risks of polypharmacy are real. We should use the lowest possible doses of medications in this population, frequently reassess goals, and monitor closely for side effects.


Pearl: Always consider the risks of polypharmacy when considering therapies for your elderly patients.
 

Dr. Paauw is professor of medicine in the division of general internal medicine at the University of Washington, Seattle, and he serves as third-year medical student clerkship director at the University of Washington. Contact Dr. Paauw at dpaauw@uw.edu.

References

1. Older Adults: Standards of Medical Care in Diabetes — 2021. Diabetes Care 2021;44(Suppl 1):S168–S179.

2. Gaur A et al. Cardiogeriatrics: The current state of the art. Heart. 2024 Jan 11:heartjnl-2022-322117.

3. Denfeld QE et al. Assessing and managing frailty in advanced heart failure: An International Society for Heart and Lung Transplantation consensus statement. J Heart Lung Transplant. 2023 Nov 29:S1053-2498(23)02028-4.

4. Gutiérrez-Valencia M et al. The relationship between frailty and polypharmacy in older people: A systematic review. Br J Clin Pharmacol. 2018 Jul;84(7):1432-44.

An 88-year-old man comes for clinic follow up. He has a medical history of type 2 diabetes, hypertension, heart failure with reduced ejection fraction, and chronic kidney disease. He recently had laboratory tests done: BUN, 32 mg/dL; creatinine, 2.3 mg/dL; potassium, 4.5 mmol/L; bicarbonate, 22 Eq/L; and A1c, 8.2%.

He checks his blood glucose daily (alternating between fasting blood glucose and before dinner) and his fasting blood glucose levels are around 130 mg/dL. His highest glucose reading was 240 mg/dL. He does not have polyuria or visual changes. Current medications: atorvastatin, irbesartan, empagliflozin, and amlodipine. On physical exam his blood pressure is 130/70 mm Hg, pulse is 80, and his BMI 20.

What medication adjustments would you recommend?

A. Begin insulin glargine at bedtime

B. Begin mealtime insulin aspart

C. Begin semaglutide

D. Begin metformin

E. No changes

I think the correct approach here would be no changes. Most physicians know guideline recommendations for A1c of less than 7% are used for patients with diabetes with few comorbid conditions, normal cognition, and functional status. Many of our elderly patients do not meet these criteria and the goal of intense medical treatment of diabetes is different in those patients. The American Diabetes Association has issued a thoughtful paper on treatment of diabetes in elderly people, stressing that patients should have very individualized goals, and that there is no one-size-fits all A1c goal.1

Paauw_Doug_SEATTLE_2019_web.jpg
Dr. Douglas S. Paauw

In this patient I would avoid adding insulin, given hypoglycemia risk. A GLP-1 agonist might appear attractive given his multiple cardiovascular risk factors, but his low BMI is a major concern for frailty that may well be worsened with reduced nutrient intake. Diabetes is the chronic condition that probably has the most guidance for management in elderly patients.

I recently saw a 92-year-old man with heart failure with reduced ejection fraction and atrial fibrillation who had been losing weight and becoming weaker. He had suffered several falls in the previous 2 weeks. His medication list included amiodarone, apixaban, sacubitril/valsartan, carvedilol, empagliflozin, spironolactone, and furosemide. He was extremely frail and had stopped eating. He was receiving all guideline-directed therapies, yet he was miserable and dying. Falls in this population are potentially as fatal as decompensated heart disease.

I stopped his amiodarone, furosemide, and spironolactone, and reduced his doses of sacubitril/valsartan and carvedilol. His appetite returned and his will to live returned. Heart failure guidelines do not include robust studies of very elderly patients because few studies exist in this population. Frailty assessment is crucial in decision making in your elderly patients.2,3 and frequent check-ins to make sure that they are not suffering from the effects of polypharmacy are crucial. Our goal in our very elderly patients is quality life-years. Polypharmacy has the potential to decrease the quality of life, as well as potentially shorten life.

The very elderly are at risk of the negative consequences of polypharmacy, especially if they have several diseases like diabetes, congestive heart failure, and hypertension that may require multiple medications. Gutierrez-Valencia and colleagues performed a systematic review of 25 articles on frailty and polypharmacy.4 Their findings demonstrated a significant association between an increased number of medications and frailty. They postulated that polypharmacy could actually be a contributor to frailty. There just isn’t enough evidence for the benefit of guidelines in the very aged and the risks of polypharmacy are real. We should use the lowest possible doses of medications in this population, frequently reassess goals, and monitor closely for side effects.


Pearl: Always consider the risks of polypharmacy when considering therapies for your elderly patients.
 

Dr. Paauw is professor of medicine in the division of general internal medicine at the University of Washington, Seattle, and he serves as third-year medical student clerkship director at the University of Washington. Contact Dr. Paauw at dpaauw@uw.edu.

References

1. Older Adults: Standards of Medical Care in Diabetes — 2021. Diabetes Care 2021;44(Suppl 1):S168–S179.

2. Gaur A et al. Cardiogeriatrics: The current state of the art. Heart. 2024 Jan 11:heartjnl-2022-322117.

3. Denfeld QE et al. Assessing and managing frailty in advanced heart failure: An International Society for Heart and Lung Transplantation consensus statement. J Heart Lung Transplant. 2023 Nov 29:S1053-2498(23)02028-4.

4. Gutiérrez-Valencia M et al. The relationship between frailty and polypharmacy in older people: A systematic review. Br J Clin Pharmacol. 2018 Jul;84(7):1432-44.

Publications
Publications
Topics
Article Type
Sections
Teambase XML
<?xml version="1.0" encoding="UTF-8"?>
<!--$RCSfile: InCopy_agile.xsl,v $ $Revision: 1.35 $-->
<!--$RCSfile: drupal.xsl,v $ $Revision: 1.7 $-->
<root generator="drupal.xsl" gversion="1.7"> <header> <fileName>166806</fileName> <TBEID>0C04E5FA.SIG</TBEID> <TBUniqueIdentifier>MD_0C04E5FA</TBUniqueIdentifier> <newsOrJournal>News</newsOrJournal> <publisherName>Frontline Medical Communications</publisherName> <storyname/> <articleType>353</articleType> <TBLocation>QC Done-All Pubs</TBLocation> <QCDate>20240202T142308</QCDate> <firstPublished>20240202T151639</firstPublished> <LastPublished>20240202T151639</LastPublished> <pubStatus qcode="stat:"/> <embargoDate/> <killDate/> <CMSDate>20240202T151639</CMSDate> <articleSource/> <facebookInfo/> <meetingNumber/> <byline>Douglas Paauw</byline> <bylineText>DOUGLAS S. PAAUW, MD</bylineText> <bylineFull>DOUGLAS S. PAAUW, MD</bylineFull> <bylineTitleText/> <USOrGlobal/> <wireDocType/> <newsDocType/> <journalDocType/> <linkLabel/> <pageRange/> <citation/> <quizID/> <indexIssueDate/> <itemClass qcode="ninat:text"/> <provider qcode="provider:imng"> <name>IMNG Medical Media</name> <rightsInfo> <copyrightHolder> <name>Frontline Medical News</name> </copyrightHolder> <copyrightNotice>Copyright (c) 2015 Frontline Medical News, a Frontline Medical Communications Inc. company. All rights reserved. This material may not be published, broadcast, copied, or otherwise reproduced or distributed without the prior written permission of Frontline Medical Communications Inc.</copyrightNotice> </rightsInfo> </provider> <abstract/> <metaDescription>An 88-year-old man comes for clinic follow up. He has a medical history of type 2 diabetes, hypertension, heart failure with reduced ejection fraction, and chro</metaDescription> <articlePDF/> <teaserImage>248215</teaserImage> <teaser>Always consider the risks of polypharmacy when considering therapies for your elderly patients.</teaser> <title>Guidelines Aren’t For Everybody</title> <deck/> <disclaimer/> <AuthorList/> <articleURL/> <doi/> <pubMedID/> <publishXMLStatus/> <publishXMLVersion>1</publishXMLVersion> <useEISSN>0</useEISSN> <urgency/> <pubPubdateYear/> <pubPubdateMonth/> <pubPubdateDay/> <pubVolume/> <pubNumber/> <wireChannels/> <primaryCMSID/> <CMSIDs/> <keywords/> <seeAlsos/> <publications_g> <publicationData> <publicationCode>card</publicationCode> <pubIssueName/> <pubArticleType/> <pubTopics/> <pubCategories/> <pubSections/> </publicationData> <publicationData> <publicationCode>endo</publicationCode> <pubIssueName/> <pubArticleType/> <pubTopics/> <pubCategories/> <pubSections/> </publicationData> <publicationData> <publicationCode>fp</publicationCode> <pubIssueName/> <pubArticleType/> <pubTopics/> <pubCategories/> <pubSections/> </publicationData> <publicationData> <publicationCode>im</publicationCode> <pubIssueName/> <pubArticleType/> <pubTopics/> <pubCategories/> <pubSections/> </publicationData> </publications_g> <publications> <term>5</term> <term>34</term> <term>15</term> <term canonical="true">21</term> </publications> <sections> <term>52</term> <term canonical="true">39786</term> </sections> <topics> <term>224</term> <term>185</term> <term canonical="true">205</term> <term>66772</term> <term>194</term> <term>215</term> </topics> <links> <link> <itemClass qcode="ninat:picture"/> <altRep contenttype="image/jpeg">images/2400c6a0.jpg</altRep> <description role="drol:caption">Dr. Douglas S. Paauw</description> <description role="drol:credit"/> </link> </links> </header> <itemSet> <newsItem> <itemMeta> <itemRole>Main</itemRole> <itemClass>text</itemClass> <title>Guidelines Aren’t For Everybody</title> <deck/> </itemMeta> <itemContent> <p>An 88-year-old man comes for clinic follow up. He has a medical history of type 2 diabetes, hypertension, heart failure with reduced ejection fraction, and chronic kidney disease. He recently had laboratory tests done: BUN, 32 mg/dL; creatinine, 2.3 mg/dL; potassium, 4.5 mmol/L; bicarbonate, 22 Eq/L; and A1c, 8.2%. </p> <p>He checks his blood glucose daily (alternating between fasting blood glucose and before dinner) and his fasting blood glucose levels are around 130 mg/dL. His highest glucose reading was 240 mg/dL. He does not have polyuria or visual changes. Current medications: atorvastatin, irbesartan, empagliflozin, and amlodipine. On physical exam his blood pressure is 130/70 mm Hg, pulse is 80, and his BMI 20.<br/><br/>What medication adjustments would you recommend?<br/><br/><strong>A.</strong> Begin insulin glargine at bedtime<br/><br/><strong>B.</strong> Begin mealtime insulin aspart<br/><br/><strong>C.</strong> Begin semaglutide<br/><br/><strong>D.</strong> Begin metformin<br/><br/><strong>E.</strong> No changes<br/><br/>I think the correct approach here would be no changes. Most physicians know guideline recommendations for A1c of less than 7% are used for patients with diabetes with few comorbid conditions, normal cognition, and functional status. Many of our elderly patients do not meet these criteria and the goal of intense medical treatment of diabetes is different in those patients. The American Diabetes Association has issued a thoughtful paper on treatment of diabetes in elderly people, stressing that patients should have very individualized goals, and that there is no one-size-fits all A1c goal.<sup>1</sup> <br/><br/>[[{"fid":"248215","view_mode":"medstat_image_flush_right","fields":{"format":"medstat_image_flush_right","field_file_image_alt_text[und][0][value]":"Dr. Douglas S. Paauw, University of Washington, Seattle","field_file_image_credit[und][0][value]":"","field_file_image_caption[und][0][value]":"Dr. Douglas S. Paauw"},"type":"media","attributes":{"class":"media-element file-medstat_image_flush_right"}}]]In this patient I would avoid adding insulin, given hypoglycemia risk. A GLP-1 agonist might appear attractive given his multiple cardiovascular risk factors, but his low BMI is a major concern for frailty that may well be worsened with reduced nutrient intake. Diabetes is the chronic condition that probably has the most guidance for management in elderly patients.<br/><br/>I recently saw a 92-year-old man with heart failure with reduced ejection fraction and atrial fibrillation who had been losing weight and becoming weaker. He had suffered several falls in the previous 2 weeks. His medication list included amiodarone, apixaban, sacubitril/valsartan, carvedilol, empagliflozin, spironolactone, and furosemide. He was extremely frail and had stopped eating. He was receiving all guideline-directed therapies, yet he was miserable and dying. Falls in this population are potentially as fatal as decompensated heart disease.<br/><br/>I stopped his amiodarone, furosemide, and spironolactone, and reduced his doses of sacubitril/valsartan and carvedilol. His appetite returned and his will to live returned. Heart failure guidelines do not include robust studies of very elderly patients because few studies exist in this population. Frailty assessment is crucial in decision making in your elderly patients.<sup>2,3</sup> and frequent check-ins to make sure that they are not suffering from the effects of polypharmacy are crucial. Our goal in our very elderly patients is quality life-years. Polypharmacy has the potential to decrease the quality of life, as well as potentially shorten life.<br/><br/>The very elderly are at risk of the negative consequences of polypharmacy, especially if they have several diseases like diabetes, congestive heart failure, and hypertension that may require multiple medications. Gutierrez-Valencia and colleagues performed a systematic review of 25 articles on frailty and polypharmacy.<sup>4</sup> Their findings demonstrated a significant association between an increased number of medications and frailty. They postulated that polypharmacy could actually be a contributor to frailty. There just isn’t enough evidence for the benefit of guidelines in the very aged and the risks of polypharmacy are real. We should use the lowest possible doses of medications in this population, frequently reassess goals, and monitor closely for side effects.<br/><br/><br/><br/><strong>Pearl: </strong>Always consider the risks of polypharmacy when considering therapies for your elderly patients.<br/><br/></p> <p> <em>Dr. Paauw is professor of medicine in the division of general internal medicine at the University of Washington, Seattle, and he serves as third-year medical student clerkship director at the University of Washington. Contact Dr. Paauw at <span class="Hyperlink"><a href="mailto:dpaauw%40uw.edu?subject=">dpaauw@uw.edu</a></span>.</em> </p> <h2>References</h2> <p>1. Older Adults: Standards of Medical Care in Diabetes — 2021. <span class="Hyperlink"><a href="https://diabetesjournals.org/care/article/44/Supplement_1/S168/30583/12-Older-Adults-Standards-of-Medical-Care-in">Diabetes Care 2021;44(Suppl 1):S168–S179</a></span>.<br/><br/>2. Gaur A et al. Cardiogeriatrics: The current state of the art. <span class="Hyperlink"><a href="https://heart.bmj.com/content/early/2024/01/11/heartjnl-2022-322117">Heart. 2024 Jan 11:heartjnl-2022-322117</a></span>.<br/><br/>3. Denfeld QE et al. Assessing and managing frailty in advanced heart failure: An International Society for Heart and Lung Transplantation consensus statement. <span class="Hyperlink"><a href="https://www.jhltonline.org/article/S1053-2498(23)02028-4/fulltext">J Heart Lung Transplant. 2023 Nov 29:S1053-2498(23)02028-4</a></span>.<br/><br/>4. Gutiérrez-Valencia M et al. The relationship between frailty and polypharmacy in older people: A systematic review. <span class="Hyperlink"><a href="https://bpspubs.onlinelibrary.wiley.com/doi/10.1111/bcp.13590">Br J Clin Pharmacol. 2018 Jul;84(7):1432-44</a></span>.</p> </itemContent> </newsItem> <newsItem> <itemMeta> <itemRole>teaser</itemRole> <itemClass>text</itemClass> <title/> <deck/> </itemMeta> <itemContent> </itemContent> </newsItem> </itemSet></root>
Disallow All Ads
Content Gating
No Gating (article Unlocked/Free)
Alternative CME
Disqus Comments
Default
Use ProPublica
Hide sidebar & use full width
render the right sidebar.
Conference Recap Checkbox
Not Conference Recap
Clinical Edge
Display the Slideshow in this Article
Medscape Article
Display survey writer
Reuters content
Disable Inline Native ads
WebMD Article

Pet Peeves About the State of Primary Care – Part 2

Article Type
Changed
Fri, 01/12/2024 - 13:32

I have received lots of notes from readers about other pet peeves they have about practicing primary care in our current environment and wanted to share some of them. I appreciate all the emails I received on this topic.

  • The rapid increase in the number of hospital administrators in the last 50 years

This has increased health system costs without providing any relief for practicing physicians, and often has led to policies that have been harmful and detrimental. This would be a great place to start cutting back to get true savings without affecting quality of care.

Paauw_Doug_SEATTLE_2019_web.jpg
Dr. Douglas S. Paauw

  • Emergency physicians and specialists who refer my patient elsewhere for a service we provide in our office

It is expensive for patients to go to a specialty provider for a simple procedure that can be easily done in a primary care practice, or to be referred to see a specialist for a problem that does not need specialty care. This creates further problems accessing specialists.

  • Online reviews of practices, including reviews from people who have never been patients

I am concerned about the accuracy and intent of online reviews. If a patient is upset because they did not receive an antibiotic or narcotic, they can vent their frustration in a review, when what the medical professional was actually doing was good medicine. More concerning to me is that some organizations use these reviews to determine compensation, promotion, and support. These reviews are not evidence based or accurately collected.

  • Offices and organizations being dropped by insurance carriers

Insurance companies are running amok. They make their own rules, which can devastate practices and patients. They can change fees paid unilaterally, and drop practices without explanation or valid reasons. Patients suffer terribly because they now cannot see their long-time physicians or they have to pay much more to see them as they are suddenly “out of network.”

  • The lack of appreciation by organizations as well as the general public of the enormous cost savings primary care professionals contribute to the healthcare system

There are many studies showing that patients who see a primary care physician save the system money and have better health outcomes. US adults who regularly see a primary care physician have 33% lower healthcare costs and 19% lower odds of dying prematurely than those who see only a specialist.1

In one study, for every $1 invested in primary care, there was $13 in savings in healthcare costs.2 I had a patient a few years ago complain about the “enormous” bill she received for a visit where I had done an annual exam, cryotherapy for three actinic keratoses, and a steroid injection for her ailing knee. The cost savings was well over $700 (the new patient cost for two specialty visits). There is no doubt that patients who have stable primary care save money themselves and for the whole medical system.
 

 

 

  • The stress of being witness to a dysfunctional system

It is really hard to see the hurt and difficulty our patients go through on a daily basis while trying to navigate a broken system. We bear witness to them and listen to all the stories when things have gone wrong. This also takes its toll on us, as we are part of the system, and our patients’ frustrations sometimes boil over. We are also the ones who care for the whole patient, so every bad experience with a specialty clinic is shared with us.

Many thanks extended to those who wrote to share their ideas (Drs. Sylvia Androne, Bhawna Bahethi, Pierre Ghassibi, Richard Katz, Louis Kasunic, Rebecca Keenan, David Kosnosky, Gregory Miller, and James Wilkens).

Dr. Paauw is professor of medicine in the Division of General Internal Medicine at the University of Washington, Seattle, and he serves as third-year medical student clerkship director at the University of Washington, Seattle. Contact Dr. Paauw at dpaauw@uw.edu.
 

References

1. Forbes.com. Why Primary Care Matters, and What We Can Do To Increase It. 2023 Nov 27.

2. Washingtonpost.com. A Health Care Solution We Can’t Afford to Ignore: Primary Care.

Publications
Topics
Sections

I have received lots of notes from readers about other pet peeves they have about practicing primary care in our current environment and wanted to share some of them. I appreciate all the emails I received on this topic.

  • The rapid increase in the number of hospital administrators in the last 50 years

This has increased health system costs without providing any relief for practicing physicians, and often has led to policies that have been harmful and detrimental. This would be a great place to start cutting back to get true savings without affecting quality of care.

Paauw_Doug_SEATTLE_2019_web.jpg
Dr. Douglas S. Paauw

  • Emergency physicians and specialists who refer my patient elsewhere for a service we provide in our office

It is expensive for patients to go to a specialty provider for a simple procedure that can be easily done in a primary care practice, or to be referred to see a specialist for a problem that does not need specialty care. This creates further problems accessing specialists.

  • Online reviews of practices, including reviews from people who have never been patients

I am concerned about the accuracy and intent of online reviews. If a patient is upset because they did not receive an antibiotic or narcotic, they can vent their frustration in a review, when what the medical professional was actually doing was good medicine. More concerning to me is that some organizations use these reviews to determine compensation, promotion, and support. These reviews are not evidence based or accurately collected.

  • Offices and organizations being dropped by insurance carriers

Insurance companies are running amok. They make their own rules, which can devastate practices and patients. They can change fees paid unilaterally, and drop practices without explanation or valid reasons. Patients suffer terribly because they now cannot see their long-time physicians or they have to pay much more to see them as they are suddenly “out of network.”

  • The lack of appreciation by organizations as well as the general public of the enormous cost savings primary care professionals contribute to the healthcare system

There are many studies showing that patients who see a primary care physician save the system money and have better health outcomes. US adults who regularly see a primary care physician have 33% lower healthcare costs and 19% lower odds of dying prematurely than those who see only a specialist.1

In one study, for every $1 invested in primary care, there was $13 in savings in healthcare costs.2 I had a patient a few years ago complain about the “enormous” bill she received for a visit where I had done an annual exam, cryotherapy for three actinic keratoses, and a steroid injection for her ailing knee. The cost savings was well over $700 (the new patient cost for two specialty visits). There is no doubt that patients who have stable primary care save money themselves and for the whole medical system.
 

 

 

  • The stress of being witness to a dysfunctional system

It is really hard to see the hurt and difficulty our patients go through on a daily basis while trying to navigate a broken system. We bear witness to them and listen to all the stories when things have gone wrong. This also takes its toll on us, as we are part of the system, and our patients’ frustrations sometimes boil over. We are also the ones who care for the whole patient, so every bad experience with a specialty clinic is shared with us.

Many thanks extended to those who wrote to share their ideas (Drs. Sylvia Androne, Bhawna Bahethi, Pierre Ghassibi, Richard Katz, Louis Kasunic, Rebecca Keenan, David Kosnosky, Gregory Miller, and James Wilkens).

Dr. Paauw is professor of medicine in the Division of General Internal Medicine at the University of Washington, Seattle, and he serves as third-year medical student clerkship director at the University of Washington, Seattle. Contact Dr. Paauw at dpaauw@uw.edu.
 

References

1. Forbes.com. Why Primary Care Matters, and What We Can Do To Increase It. 2023 Nov 27.

2. Washingtonpost.com. A Health Care Solution We Can’t Afford to Ignore: Primary Care.

I have received lots of notes from readers about other pet peeves they have about practicing primary care in our current environment and wanted to share some of them. I appreciate all the emails I received on this topic.

  • The rapid increase in the number of hospital administrators in the last 50 years

This has increased health system costs without providing any relief for practicing physicians, and often has led to policies that have been harmful and detrimental. This would be a great place to start cutting back to get true savings without affecting quality of care.

Paauw_Doug_SEATTLE_2019_web.jpg
Dr. Douglas S. Paauw

  • Emergency physicians and specialists who refer my patient elsewhere for a service we provide in our office

It is expensive for patients to go to a specialty provider for a simple procedure that can be easily done in a primary care practice, or to be referred to see a specialist for a problem that does not need specialty care. This creates further problems accessing specialists.

  • Online reviews of practices, including reviews from people who have never been patients

I am concerned about the accuracy and intent of online reviews. If a patient is upset because they did not receive an antibiotic or narcotic, they can vent their frustration in a review, when what the medical professional was actually doing was good medicine. More concerning to me is that some organizations use these reviews to determine compensation, promotion, and support. These reviews are not evidence based or accurately collected.

  • Offices and organizations being dropped by insurance carriers

Insurance companies are running amok. They make their own rules, which can devastate practices and patients. They can change fees paid unilaterally, and drop practices without explanation or valid reasons. Patients suffer terribly because they now cannot see their long-time physicians or they have to pay much more to see them as they are suddenly “out of network.”

  • The lack of appreciation by organizations as well as the general public of the enormous cost savings primary care professionals contribute to the healthcare system

There are many studies showing that patients who see a primary care physician save the system money and have better health outcomes. US adults who regularly see a primary care physician have 33% lower healthcare costs and 19% lower odds of dying prematurely than those who see only a specialist.1

In one study, for every $1 invested in primary care, there was $13 in savings in healthcare costs.2 I had a patient a few years ago complain about the “enormous” bill she received for a visit where I had done an annual exam, cryotherapy for three actinic keratoses, and a steroid injection for her ailing knee. The cost savings was well over $700 (the new patient cost for two specialty visits). There is no doubt that patients who have stable primary care save money themselves and for the whole medical system.
 

 

 

  • The stress of being witness to a dysfunctional system

It is really hard to see the hurt and difficulty our patients go through on a daily basis while trying to navigate a broken system. We bear witness to them and listen to all the stories when things have gone wrong. This also takes its toll on us, as we are part of the system, and our patients’ frustrations sometimes boil over. We are also the ones who care for the whole patient, so every bad experience with a specialty clinic is shared with us.

Many thanks extended to those who wrote to share their ideas (Drs. Sylvia Androne, Bhawna Bahethi, Pierre Ghassibi, Richard Katz, Louis Kasunic, Rebecca Keenan, David Kosnosky, Gregory Miller, and James Wilkens).

Dr. Paauw is professor of medicine in the Division of General Internal Medicine at the University of Washington, Seattle, and he serves as third-year medical student clerkship director at the University of Washington, Seattle. Contact Dr. Paauw at dpaauw@uw.edu.
 

References

1. Forbes.com. Why Primary Care Matters, and What We Can Do To Increase It. 2023 Nov 27.

2. Washingtonpost.com. A Health Care Solution We Can’t Afford to Ignore: Primary Care.

Publications
Publications
Topics
Article Type
Sections
Teambase XML
<?xml version="1.0" encoding="UTF-8"?>
<!--$RCSfile: InCopy_agile.xsl,v $ $Revision: 1.35 $-->
<!--$RCSfile: drupal.xsl,v $ $Revision: 1.7 $-->
<root generator="drupal.xsl" gversion="1.7"> <header> <fileName>166361</fileName> <TBEID>0C04DC73.SIG</TBEID> <TBUniqueIdentifier>MD_0C04DC73</TBUniqueIdentifier> <newsOrJournal>News</newsOrJournal> <publisherName>Frontline Medical Communications</publisherName> <storyname/> <articleType>353</articleType> <TBLocation>QC Done-All Pubs</TBLocation> <QCDate>20240112T132052</QCDate> <firstPublished>20240112T132837</firstPublished> <LastPublished>20240112T132837</LastPublished> <pubStatus qcode="stat:"/> <embargoDate/> <killDate/> <CMSDate>20240112T132837</CMSDate> <articleSource/> <facebookInfo/> <meetingNumber/> <byline>Douglas S. Paauw</byline> <bylineText/> <bylineFull/> <bylineTitleText/> <USOrGlobal/> <wireDocType/> <newsDocType/> <journalDocType/> <linkLabel/> <pageRange/> <citation/> <quizID/> <indexIssueDate/> <itemClass qcode="ninat:text"/> <provider qcode="provider:imng"> <name>IMNG Medical Media</name> <rightsInfo> <copyrightHolder> <name>Frontline Medical News</name> </copyrightHolder> <copyrightNotice>Copyright (c) 2015 Frontline Medical News, a Frontline Medical Communications Inc. company. All rights reserved. This material may not be published, broadcast, copied, or otherwise reproduced or distributed without the prior written permission of Frontline Medical Communications Inc.</copyrightNotice> </rightsInfo> </provider> <abstract/> <metaDescription>BY DOUGLAS S. PAAUW, MD</metaDescription> <articlePDF/> <teaserImage>248215</teaserImage> <teaser>Physicians note the increase in the number of hospital administrators, specialists who refer patients away from primary care, insurance carriers dropping practices, and more.</teaser> <title>Pet Peeves About the State of Primary Care – Part 2</title> <deck/> <disclaimer/> <AuthorList/> <articleURL/> <doi/> <pubMedID/> <publishXMLStatus/> <publishXMLVersion>1</publishXMLVersion> <useEISSN>0</useEISSN> <urgency/> <pubPubdateYear/> <pubPubdateMonth/> <pubPubdateDay/> <pubVolume/> <pubNumber/> <wireChannels/> <primaryCMSID/> <CMSIDs/> <keywords/> <seeAlsos/> <publications_g> <publicationData> <publicationCode>fp</publicationCode> <pubIssueName/> <pubArticleType/> <pubTopics/> <pubCategories/> <pubSections/> </publicationData> <publicationData> <publicationCode>im</publicationCode> <pubIssueName/> <pubArticleType/> <pubTopics/> <pubCategories/> <pubSections/> </publicationData> </publications_g> <publications> <term>15</term> <term canonical="true">21</term> </publications> <sections> <term canonical="true">39786</term> </sections> <topics> <term canonical="true">38029</term> </topics> <links> <link> <itemClass qcode="ninat:picture"/> <altRep contenttype="image/jpeg">images/2400c6a0.jpg</altRep> <description role="drol:caption">Dr. Douglas S. Paauw</description> <description role="drol:credit"/> </link> </links> </header> <itemSet> <newsItem> <itemMeta> <itemRole>Main</itemRole> <itemClass>text</itemClass> <title>Pet Peeves About the State of Primary Care – Part 2</title> <deck/> </itemMeta> <itemContent> <p>BY DOUGLAS S. PAAUW, MD<br/><br/></p> <p>I have received lots of notes from readers about other pet peeves they have about practicing primary care in our current environment and wanted to share some of them. I appreciate all the emails I received on this topic.</p> <ul class="body"> <li><strong>The rapid increase in the number of hospital administrators in the last 50 years</strong> </li> </ul> <p>This has increased health system costs without providing any relief for practicing physicians, and often has led to policies that have been harmful and detrimental. This would be a great place to start cutting back to get true savings without affecting quality of care.[[{"fid":"248215","view_mode":"medstat_image_flush_right","fields":{"format":"medstat_image_flush_right","field_file_image_alt_text[und][0][value]":"Dr. Douglas S. Paauw, University of Washington, Seattle","field_file_image_credit[und][0][value]":"","field_file_image_caption[und][0][value]":"Dr. Douglas S. Paauw"},"type":"media","attributes":{"class":"media-element file-medstat_image_flush_right"}}]]</p> <ul class="body"> <li> <strong>Emergency physicians and specialists who refer my patient elsewhere for a service we provide in our office</strong> </li> </ul> <p>It is expensive for patients to go to a specialty provider for a simple procedure that can be easily done in a primary care practice, or to be referred to see a specialist for a problem that does not need specialty care. This creates further problems accessing specialists.</p> <ul class="body"> <li> <strong>Online reviews of practices, including reviews from people who have never been patients</strong> </li> </ul> <p>I am concerned about the accuracy and intent of online reviews. If a patient is upset because they did not receive an antibiotic or narcotic, they can vent their frustration in a review, when what the medical professional was actually doing was good medicine. More concerning to me is that some organizations use these reviews to determine compensation, promotion, and support. These reviews are not evidence based or accurately collected.</p> <ul class="body"> <li> <strong>Offices and organizations being dropped by insurance carriers</strong> </li> </ul> <p>Insurance companies are running amok. They make their own rules, which can devastate practices and patients. They can change fees paid unilaterally, and drop practices without explanation or valid reasons. Patients suffer terribly because they now cannot see their long-time physicians or they have to pay much more to see them as they are suddenly “out of network.”</p> <ul class="body"> <li> <strong>The lack of appreciation by organizations as well as the general public of the enormous cost savings primary care professionals contribute to the healthcare system</strong> </li> </ul> <p>There are many studies showing that patients who see a primary care physician save the system money and have better health outcomes. US adults who regularly see a primary care physician have 33% lower healthcare costs and 19% lower odds of dying prematurely than those who see only a specialist.<sup>1</sup> </p> <p>In one study, for every $1 invested in primary care, there was $13 in savings in healthcare costs.<sup>2</sup> I had a patient a few years ago complain about the “enormous” bill she received for a visit where I had done an annual exam, cryotherapy for three actinic keratoses, and a steroid injection for her ailing knee. The cost savings was well over $700 (the new patient cost for two specialty visits). There is no doubt that patients who have stable primary care save money themselves and for the whole medical system.<br/><br/></p> <ul class="body"> <li> <strong>The stress of being witness to a dysfunctional system</strong> </li> </ul> <p>It is really hard to see the hurt and difficulty our patients go through on a daily basis while trying to navigate a broken system. We bear witness to them and listen to all the stories when things have gone wrong. This also takes its toll on us, as we are part of the system, and our patients’ frustrations sometimes boil over. We are also the ones who care for the whole patient, so every bad experience with a specialty clinic is shared with us.</p> <p><br/><br/>Many thanks extended to those who wrote to share their ideas (Drs. Sylvia Androne, Bhawna Bahethi, Pierre Ghassibi, Richard Katz, Louis Kasunic, Rebecca Keenan, David Kosnosky, Gregory Miller, and James Wilkens).<span class="end"/></p> <p> <em>Dr. Paauw is professor of medicine in the Division of General Internal Medicine at the University of Washington, Seattle, and he serves as third-year medical student clerkship director at the University of Washington, Seattle. Contact Dr. Paauw at <span class="Hyperlink"><a href="mailto:dpaauw%40uw.edu?subject=">dpaauw@uw.edu</a></span>.<br/><br/></em> </p> <h2>References</h2> <p>1. <span class="Hyperlink"><a href="https://www.forbes.com/sites/forbesbooksauthors/2023/11/27/why-primary-care-matters-and-what-we-can-do-to-increase-it/?sh=29a192931b51">Forbes.com</a></span>. Why Primary Care Matters, and What We Can Do To Increase It. 2023 Nov 27.<br/><br/>2. <span class="Hyperlink"><a href="https://www.washingtonpost.com/sf/brand-connect/health-is-primary/a-healthcare-solution-we-cant-afford-to-ignore/">Washingtonpost.com</a></span>. A Health Care Solution We Can’t Afford to Ignore: Primary Care. </p> </itemContent> </newsItem> <newsItem> <itemMeta> <itemRole>teaser</itemRole> <itemClass>text</itemClass> <title/> <deck/> </itemMeta> <itemContent> </itemContent> </newsItem> </itemSet></root>
Disallow All Ads
Content Gating
No Gating (article Unlocked/Free)
Alternative CME
Disqus Comments
Default
Use ProPublica
Hide sidebar & use full width
render the right sidebar.
Conference Recap Checkbox
Not Conference Recap
Clinical Edge
Display the Slideshow in this Article
Medscape Article
Display survey writer
Reuters content
Disable Inline Native ads
WebMD Article

Diagnosing patients with sarcoidosis

Article Type
Changed
Mon, 11/20/2023 - 06:36

A 40-year-old women is evaluated for liver abnormalities. She had elevated transaminases and alkaline phosphatase. A liver ultrasound showed multiple lesions. She underwent liver biopsy, which showed granulomas. What test results, if abnormal, would be most suggestive of sarcoidosis?

A. Erythrocyte sedimentation rate

B. C-reactive protein

C. Lymphocyte count

D. Antinuclear antibodies



The correct answer here is lymphocyte count. Sarcoidosis is in just about every differential diagnosis, as it can involve every organ system. I will share with you a few pearls I have learned over 30 years of taking care of patients with sarcoidosis. Lymphocyte counts drop with active sarcoidosis. Sarcoidosis should always be part of the differential when you see lymphopenia. El Jammal et al. studied 90 patients referred for possible granulomatous hepatitis.1 Seventy-three patients had a final diagnosis of granulomatous hepatitis, and 38 of those patients had sarcoidosis. Lymphopenia had a high specificity (85.7%) for the diagnosis of sarcoidosis, with a specificity of 100% in the patients under 50 years old.

Morell and colleagues looked at whether low lymphocyte counts and low lymphocyte percentage were markers of active sarcoidosis.2 Forty patients with biopsy-proven sarcoidosis were prospectively evaluated every 6 months. A low lymphocyte count and a low lymphocyte percentage (< 20%) were detected more frequently in patients with active sarcoidosis than in the patients with asymptomatic sarcoidosis (P < .02 and P < .0001).

Paauw_Doug_SEATTLE_2019_web.jpg
Dr. Douglas S. Paauw

Jones et al. looked at lymphopenia as a marker of sarcoidosis in patients presenting with uveitis.3 The study was a retrospective case-control study (112 patients with sarcoidosis-associated uveitis and 398 controls with other forms of uveitis). The mean lymphocyte count for patients with sarcoidosis was 1.43 vs. 2.04 for other causes of uveitis (P ≤ .0001).

Patients with sarcoidosis are at risk of hypercalciuria, hypercalcemia, and kidney stones. These are common in patients with sarcoidosis, with up to 50% of such patients having hypercalciuria. This is because in sarcoidosis patients 25(OH) vitamin D is converted in granulomas by activated macrophages to 1,25(OH)2 vitamin D, which is the active form of vitamin D.

Several studies have looked at the diagnostic utility of 1,25(OH)2 vitamin D levels in patients with suspected sarcoidosis. Rohmer and colleagues looked at whether 1,25(OH)2 vitamin D levels could help with the diagnosis of sarcoidosis as the cause of uveitis.4 They found that the level of 25(OH) vitamin D in sarcoidosis patients with uveitis was lower than in patients with uveitis without sarcoidosis, 34 vs. 43 nmol/mL (P < .02), whereas the 1,25(OH)2 vitamin D level was higher in patients with sarcoidosis than in those with uveitis without sarcoidosis, 132 vs. 108 pmol/L (P = .02). They looked at the 1,25(OH)2D/25(OH)D ratio; a ratio > 3.5 was strongly associated with an abnormal chest CT-scan (OR = 5.7, P = .003) and granulomas on bronchial biopsy (OR = 14.7, P = .007).

Kavathia et al. looked at whether elevated 1,25(OH)2 vitamin D levels predicted chronicity of sarcoidosis.5 A total of 59 sarcoidosis patients were recruited for the study. Higher serum 1,25(OH)2 vitamin D levels were associated with patients requiring repeated systemic immunosuppressive therapy or > 1 year of therapy. Increasing quartiles of serum 1,25(OH)2 vitamin D level were associated with increased odds of patients having chronic sarcoidosis (OR = 1.82; 95% CI, 1.11-2.99, P = .019).

Because of the higher activated vitamin D levels in sarcoidosis patients, they are at risk for problems with vitamin D supplementation. I have seen two patients develop large numbers of kidney stones after receiving high-dose vitamin D. Sodhi and Aldrich reported on a cohort of 196 sarcoidosis patients who had received vitamin D and compared them with 196 control patients with sarcoidosis who were not receiving vitamin D.6 Hypercalcemia was more frequent in the group that received vitamin D (42.3%) than in the group that did not (18.3%, P < .0001). In this study, only a minority (23%) of patients receiving vitamin D had their 1,25(OH)2 vitamin D level checked.


Pearl: Lymphocyte count and 1,25(OH)2 vitamin D levels can be helpful tests in assessing sarcoidosis activity. Patients with sarcoidosis who receive vitamin D should have their 1.25(OH)2 vitamin D levels monitored.

Dr. Paauw is professor of medicine in the division of general internal medicine at the University of Washington, Seattle, and he serves as third-year medical student clerkship director at the University of Washington. Contact Dr. Paauw at dpaauw@uw.edu.

References

1. El Jammal et al. Sarcoidosis Vasc Diffuse Lung Dis. 2023 Sep 13;40(3):e2023031.

2. Morell F et al. Chest. 2002 Apr;121(4):1239-44.

3. Jones NP et al. Br J Ophthalmol. 2016 Oct;100(10):1393-6.

4. Rohmer J et al. Ocul Immunol Inflamm. 2020 Apr 2;28(3):341-7.

5. Kavathia D et al. Respir Med. 2010 Apr;104(4):564–70.

6. Sodhi A and Aldrich T. Am J Med Sci. 2016 Sep;352(3):252-7.

Publications
Topics
Sections

A 40-year-old women is evaluated for liver abnormalities. She had elevated transaminases and alkaline phosphatase. A liver ultrasound showed multiple lesions. She underwent liver biopsy, which showed granulomas. What test results, if abnormal, would be most suggestive of sarcoidosis?

A. Erythrocyte sedimentation rate

B. C-reactive protein

C. Lymphocyte count

D. Antinuclear antibodies



The correct answer here is lymphocyte count. Sarcoidosis is in just about every differential diagnosis, as it can involve every organ system. I will share with you a few pearls I have learned over 30 years of taking care of patients with sarcoidosis. Lymphocyte counts drop with active sarcoidosis. Sarcoidosis should always be part of the differential when you see lymphopenia. El Jammal et al. studied 90 patients referred for possible granulomatous hepatitis.1 Seventy-three patients had a final diagnosis of granulomatous hepatitis, and 38 of those patients had sarcoidosis. Lymphopenia had a high specificity (85.7%) for the diagnosis of sarcoidosis, with a specificity of 100% in the patients under 50 years old.

Morell and colleagues looked at whether low lymphocyte counts and low lymphocyte percentage were markers of active sarcoidosis.2 Forty patients with biopsy-proven sarcoidosis were prospectively evaluated every 6 months. A low lymphocyte count and a low lymphocyte percentage (< 20%) were detected more frequently in patients with active sarcoidosis than in the patients with asymptomatic sarcoidosis (P < .02 and P < .0001).

Paauw_Doug_SEATTLE_2019_web.jpg
Dr. Douglas S. Paauw

Jones et al. looked at lymphopenia as a marker of sarcoidosis in patients presenting with uveitis.3 The study was a retrospective case-control study (112 patients with sarcoidosis-associated uveitis and 398 controls with other forms of uveitis). The mean lymphocyte count for patients with sarcoidosis was 1.43 vs. 2.04 for other causes of uveitis (P ≤ .0001).

Patients with sarcoidosis are at risk of hypercalciuria, hypercalcemia, and kidney stones. These are common in patients with sarcoidosis, with up to 50% of such patients having hypercalciuria. This is because in sarcoidosis patients 25(OH) vitamin D is converted in granulomas by activated macrophages to 1,25(OH)2 vitamin D, which is the active form of vitamin D.

Several studies have looked at the diagnostic utility of 1,25(OH)2 vitamin D levels in patients with suspected sarcoidosis. Rohmer and colleagues looked at whether 1,25(OH)2 vitamin D levels could help with the diagnosis of sarcoidosis as the cause of uveitis.4 They found that the level of 25(OH) vitamin D in sarcoidosis patients with uveitis was lower than in patients with uveitis without sarcoidosis, 34 vs. 43 nmol/mL (P < .02), whereas the 1,25(OH)2 vitamin D level was higher in patients with sarcoidosis than in those with uveitis without sarcoidosis, 132 vs. 108 pmol/L (P = .02). They looked at the 1,25(OH)2D/25(OH)D ratio; a ratio > 3.5 was strongly associated with an abnormal chest CT-scan (OR = 5.7, P = .003) and granulomas on bronchial biopsy (OR = 14.7, P = .007).

Kavathia et al. looked at whether elevated 1,25(OH)2 vitamin D levels predicted chronicity of sarcoidosis.5 A total of 59 sarcoidosis patients were recruited for the study. Higher serum 1,25(OH)2 vitamin D levels were associated with patients requiring repeated systemic immunosuppressive therapy or > 1 year of therapy. Increasing quartiles of serum 1,25(OH)2 vitamin D level were associated with increased odds of patients having chronic sarcoidosis (OR = 1.82; 95% CI, 1.11-2.99, P = .019).

Because of the higher activated vitamin D levels in sarcoidosis patients, they are at risk for problems with vitamin D supplementation. I have seen two patients develop large numbers of kidney stones after receiving high-dose vitamin D. Sodhi and Aldrich reported on a cohort of 196 sarcoidosis patients who had received vitamin D and compared them with 196 control patients with sarcoidosis who were not receiving vitamin D.6 Hypercalcemia was more frequent in the group that received vitamin D (42.3%) than in the group that did not (18.3%, P < .0001). In this study, only a minority (23%) of patients receiving vitamin D had their 1,25(OH)2 vitamin D level checked.


Pearl: Lymphocyte count and 1,25(OH)2 vitamin D levels can be helpful tests in assessing sarcoidosis activity. Patients with sarcoidosis who receive vitamin D should have their 1.25(OH)2 vitamin D levels monitored.

Dr. Paauw is professor of medicine in the division of general internal medicine at the University of Washington, Seattle, and he serves as third-year medical student clerkship director at the University of Washington. Contact Dr. Paauw at dpaauw@uw.edu.

References

1. El Jammal et al. Sarcoidosis Vasc Diffuse Lung Dis. 2023 Sep 13;40(3):e2023031.

2. Morell F et al. Chest. 2002 Apr;121(4):1239-44.

3. Jones NP et al. Br J Ophthalmol. 2016 Oct;100(10):1393-6.

4. Rohmer J et al. Ocul Immunol Inflamm. 2020 Apr 2;28(3):341-7.

5. Kavathia D et al. Respir Med. 2010 Apr;104(4):564–70.

6. Sodhi A and Aldrich T. Am J Med Sci. 2016 Sep;352(3):252-7.

A 40-year-old women is evaluated for liver abnormalities. She had elevated transaminases and alkaline phosphatase. A liver ultrasound showed multiple lesions. She underwent liver biopsy, which showed granulomas. What test results, if abnormal, would be most suggestive of sarcoidosis?

A. Erythrocyte sedimentation rate

B. C-reactive protein

C. Lymphocyte count

D. Antinuclear antibodies



The correct answer here is lymphocyte count. Sarcoidosis is in just about every differential diagnosis, as it can involve every organ system. I will share with you a few pearls I have learned over 30 years of taking care of patients with sarcoidosis. Lymphocyte counts drop with active sarcoidosis. Sarcoidosis should always be part of the differential when you see lymphopenia. El Jammal et al. studied 90 patients referred for possible granulomatous hepatitis.1 Seventy-three patients had a final diagnosis of granulomatous hepatitis, and 38 of those patients had sarcoidosis. Lymphopenia had a high specificity (85.7%) for the diagnosis of sarcoidosis, with a specificity of 100% in the patients under 50 years old.

Morell and colleagues looked at whether low lymphocyte counts and low lymphocyte percentage were markers of active sarcoidosis.2 Forty patients with biopsy-proven sarcoidosis were prospectively evaluated every 6 months. A low lymphocyte count and a low lymphocyte percentage (< 20%) were detected more frequently in patients with active sarcoidosis than in the patients with asymptomatic sarcoidosis (P < .02 and P < .0001).

Dr. Douglas S. Paauw

Jones et al. looked at lymphopenia as a marker of sarcoidosis in patients presenting with uveitis.3 The study was a retrospective case-control study (112 patients with sarcoidosis-associated uveitis and 398 controls with other forms of uveitis). The mean lymphocyte count for patients with sarcoidosis was 1.43 vs. 2.04 for other causes of uveitis (P ≤ .0001).

Patients with sarcoidosis are at risk of hypercalciuria, hypercalcemia, and kidney stones. These are common in patients with sarcoidosis, with up to 50% of such patients having hypercalciuria. This is because in sarcoidosis patients 25(OH) vitamin D is converted in granulomas by activated macrophages to 1,25(OH)2 vitamin D, which is the active form of vitamin D.

Several studies have looked at the diagnostic utility of 1,25(OH)2 vitamin D levels in patients with suspected sarcoidosis. Rohmer and colleagues looked at whether 1,25(OH)2 vitamin D levels could help with the diagnosis of sarcoidosis as the cause of uveitis.4 They found that the level of 25(OH) vitamin D in sarcoidosis patients with uveitis was lower than in patients with uveitis without sarcoidosis, 34 vs. 43 nmol/mL (P < .02), whereas the 1,25(OH)2 vitamin D level was higher in patients with sarcoidosis than in those with uveitis without sarcoidosis, 132 vs. 108 pmol/L (P = .02). They looked at the 1,25(OH)2D/25(OH)D ratio; a ratio > 3.5 was strongly associated with an abnormal chest CT-scan (OR = 5.7, P = .003) and granulomas on bronchial biopsy (OR = 14.7, P = .007).

Kavathia et al. looked at whether elevated 1,25(OH)2 vitamin D levels predicted chronicity of sarcoidosis.5 A total of 59 sarcoidosis patients were recruited for the study. Higher serum 1,25(OH)2 vitamin D levels were associated with patients requiring repeated systemic immunosuppressive therapy or > 1 year of therapy. Increasing quartiles of serum 1,25(OH)2 vitamin D level were associated with increased odds of patients having chronic sarcoidosis (OR = 1.82; 95% CI, 1.11-2.99, P = .019).

Because of the higher activated vitamin D levels in sarcoidosis patients, they are at risk for problems with vitamin D supplementation. I have seen two patients develop large numbers of kidney stones after receiving high-dose vitamin D. Sodhi and Aldrich reported on a cohort of 196 sarcoidosis patients who had received vitamin D and compared them with 196 control patients with sarcoidosis who were not receiving vitamin D.6 Hypercalcemia was more frequent in the group that received vitamin D (42.3%) than in the group that did not (18.3%, P < .0001). In this study, only a minority (23%) of patients receiving vitamin D had their 1,25(OH)2 vitamin D level checked.


Pearl: Lymphocyte count and 1,25(OH)2 vitamin D levels can be helpful tests in assessing sarcoidosis activity. Patients with sarcoidosis who receive vitamin D should have their 1.25(OH)2 vitamin D levels monitored.

Dr. Paauw is professor of medicine in the division of general internal medicine at the University of Washington, Seattle, and he serves as third-year medical student clerkship director at the University of Washington. Contact Dr. Paauw at dpaauw@uw.edu.

References

1. El Jammal et al. Sarcoidosis Vasc Diffuse Lung Dis. 2023 Sep 13;40(3):e2023031.

2. Morell F et al. Chest. 2002 Apr;121(4):1239-44.

3. Jones NP et al. Br J Ophthalmol. 2016 Oct;100(10):1393-6.

4. Rohmer J et al. Ocul Immunol Inflamm. 2020 Apr 2;28(3):341-7.

5. Kavathia D et al. Respir Med. 2010 Apr;104(4):564–70.

6. Sodhi A and Aldrich T. Am J Med Sci. 2016 Sep;352(3):252-7.

Publications
Publications
Topics
Article Type
Sections
Teambase XML
<?xml version="1.0" encoding="UTF-8"?>
<!--$RCSfile: InCopy_agile.xsl,v $ $Revision: 1.35 $-->
<!--$RCSfile: drupal.xsl,v $ $Revision: 1.7 $-->
<root generator="drupal.xsl" gversion="1.7"> <header> <fileName>165989</fileName> <TBEID>0C04D499.SIG</TBEID> <TBUniqueIdentifier>MD_0C04D499</TBUniqueIdentifier> <newsOrJournal>News</newsOrJournal> <publisherName>Frontline Medical Communications</publisherName> <storyname/> <articleType>353</articleType> <TBLocation>QC Done-All Pubs</TBLocation> <QCDate>20231117T133227</QCDate> <firstPublished>20231117T150249</firstPublished> <LastPublished>20231117T150249</LastPublished> <pubStatus qcode="stat:"/> <embargoDate/> <killDate/> <CMSDate>20231117T150249</CMSDate> <articleSource/> <facebookInfo/> <meetingNumber/> <byline>Douglas S Paauw</byline> <bylineText>DOUGLAS S. PAAUW, MD</bylineText> <bylineFull>DOUGLAS S. PAAUW, MD</bylineFull> <bylineTitleText/> <USOrGlobal/> <wireDocType/> <newsDocType/> <journalDocType/> <linkLabel/> <pageRange/> <citation/> <quizID/> <indexIssueDate/> <itemClass qcode="ninat:text"/> <provider qcode="provider:imng"> <name>IMNG Medical Media</name> <rightsInfo> <copyrightHolder> <name>Frontline Medical News</name> </copyrightHolder> <copyrightNotice>Copyright (c) 2015 Frontline Medical News, a Frontline Medical Communications Inc. company. All rights reserved. This material may not be published, broadcast, copied, or otherwise reproduced or distributed without the prior written permission of Frontline Medical Communications Inc.</copyrightNotice> </rightsInfo> </provider> <abstract/> <metaDescription>A 40-year-old women is evaluated for liver abnormalities. She had elevated transaminases and alkaline phosphatase. A liver ultrasound showed multiple lesions. S</metaDescription> <articlePDF/> <teaserImage>248215</teaserImage> <teaser>Sarcoidosis should always be part of the differential when you see lymphopenia.</teaser> <title>Diagnosing patients with sarcoidosis</title> <deck/> <disclaimer/> <AuthorList/> <articleURL/> <doi/> <pubMedID/> <publishXMLStatus/> <publishXMLVersion>1</publishXMLVersion> <useEISSN>0</useEISSN> <urgency/> <pubPubdateYear/> <pubPubdateMonth/> <pubPubdateDay/> <pubVolume/> <pubNumber/> <wireChannels/> <primaryCMSID/> <CMSIDs/> <keywords/> <seeAlsos/> <publications_g> <publicationData> <publicationCode>fp</publicationCode> <pubIssueName/> <pubArticleType/> <pubTopics/> <pubCategories/> <pubSections/> </publicationData> <publicationData> <publicationCode>im</publicationCode> <pubIssueName/> <pubArticleType/> <pubTopics/> <pubCategories/> <pubSections/> </publicationData> <publicationData> <publicationCode>rn</publicationCode> <pubIssueName/> <pubArticleType/> <pubTopics/> <pubCategories/> <pubSections/> </publicationData> </publications_g> <publications> <term>15</term> <term canonical="true">21</term> <term>26</term> </publications> <sections> <term>27970</term> <term canonical="true">39786</term> </sections> <topics> <term>284</term> <term>226</term> <term>203</term> <term>231</term> <term canonical="true">285</term> <term>241</term> </topics> <links> <link> <itemClass qcode="ninat:picture"/> <altRep contenttype="image/jpeg">images/2400c6a0.jpg</altRep> <description role="drol:caption">Dr. Douglas S. Paauw</description> <description role="drol:credit"/> </link> </links> </header> <itemSet> <newsItem> <itemMeta> <itemRole>Main</itemRole> <itemClass>text</itemClass> <title>Diagnosing patients with sarcoidosis</title> <deck/> </itemMeta> <itemContent> <p>A 40-year-old women is evaluated for liver abnormalities. She had elevated transaminases and alkaline phosphatase. A liver ultrasound showed multiple lesions. She underwent liver biopsy, which showed granulomas. What test results, if abnormal, would be most suggestive of sarcoidosis?<br/><br/>A. Erythrocyte sedimentation rate<br/><br/>B. C-reactive protein<br/><br/>C. Lymphocyte count<br/><br/>D. Antinuclear antibodies</p> <p><br/><br/>The correct answer here is lymphocyte count. Sarcoidosis is in just about every differential diagnosis, as it can involve every organ system. I will share with you a few pearls I have learned over 30 years of taking care of patients with sarcoidosis. Lymphocyte counts drop with active sarcoidosis. Sarcoidosis should always be part of the differential when you see lymphopenia. El Jammal et al. studied 90 patients referred for possible granulomatous hepatitis.<sup>1</sup> Seventy-three patients had a final diagnosis of granulomatous hepatitis, and 38 of those patients had sarcoidosis. Lymphopenia had a high specificity (85.7%) for the diagnosis of sarcoidosis, with a specificity of 100% in the patients under 50 years old. <br/><br/>Morell and colleagues looked at whether low lymphocyte counts and low lymphocyte percentage were markers of active sarcoidosis.<sup>2</sup> Forty patients with biopsy-proven sarcoidosis were prospectively evaluated every 6 months. A low lymphocyte count and a low lymphocyte percentage (&lt; 20%) were detected more frequently in patients with active sarcoidosis than in the patients with asymptomatic sarcoidosis (<em>P</em> &lt; .02 and <em>P</em> &lt; .0001). <br/><br/>[[{"fid":"248215","view_mode":"medstat_image_flush_right","fields":{"format":"medstat_image_flush_right","field_file_image_alt_text[und][0][value]":"Dr. Douglas S. Paauw, University of Washington, Seattle","field_file_image_credit[und][0][value]":"","field_file_image_caption[und][0][value]":"Dr. Douglas S. Paauw"},"type":"media","attributes":{"class":"media-element file-medstat_image_flush_right"}}]]Jones et al. looked at lymphopenia as a marker of sarcoidosis in patients presenting with uveitis.<sup>3</sup> The study was a retrospective case-control study (112 patients with sarcoidosis-associated uveitis and 398 controls with other forms of uveitis). The mean lymphocyte count for patients with sarcoidosis was 1.43 vs. 2.04 for other causes of uveitis (<em>P</em> ≤ .0001).<br/><br/>Patients with sarcoidosis are at risk of hypercalciuria, hypercalcemia, and kidney stones. These are common in patients with sarcoidosis, with up to 50% of such patients having hypercalciuria. This is because in sarcoidosis patients 25(OH) vitamin D is converted in granulomas by activated macrophages to 1,25(OH)<sub>2</sub> vitamin D, which is the active form of vitamin D. <br/><br/>Several studies have looked at the diagnostic utility of 1,25(OH)<sub>2</sub> vitamin D levels in patients with suspected sarcoidosis. Rohmer and colleagues looked at whether 1,25(OH)<sub>2</sub> vitamin D levels could help with the diagnosis of sarcoidosis as the cause of uveitis.<sup>4</sup> They found that the level of 25(OH) vitamin D in sarcoidosis patients with uveitis was lower than in patients with uveitis without sarcoidosis, 34 vs. 43 nmol/mL (<em>P</em> &lt; .02), whereas the 1,25(OH)<sub>2</sub> vitamin D level was higher in patients with sarcoidosis than in those with uveitis without sarcoidosis, 132 vs. 108 pmol/L (<em>P</em> = .02). They looked at the 1,25(OH)<sub>2</sub>D/25(OH)D ratio; a ratio &gt; 3.5 was strongly associated with an abnormal chest CT-scan (OR = 5.7, <em>P</em> = .003) and granulomas on bronchial biopsy (OR = 14.7, <em>P</em> = .007). <br/><br/>Kavathia et al. looked at whether elevated 1,25(OH)<sub>2</sub> vitamin D levels predicted chronicity of sarcoidosis.<sup>5</sup> A total of 59 sarcoidosis patients were recruited for the study. Higher serum 1,25(OH)<sub>2</sub> vitamin D levels were associated with patients requiring repeated systemic immunosuppressive therapy or &gt; 1 year of therapy. Increasing quartiles of serum 1,25(OH)<sub>2</sub> vitamin D level were associated with increased odds of patients having chronic sarcoidosis (OR = 1.82; 95% CI, 1.11-2.99, <em>P</em> = .019). <br/><br/>Because of the higher activated vitamin D levels in sarcoidosis patients, they are at risk for problems with vitamin D supplementation. I have seen two patients develop large numbers of kidney stones after receiving high-dose vitamin D. Sodhi and Aldrich reported on a cohort of 196 sarcoidosis patients who had received vitamin D and compared them with 196 control patients with sarcoidosis who were not receiving vitamin D.<sup>6 </sup>Hypercalcemia was more frequent in the group that received vitamin D (42.3%) than in the group that did not (18.3%, <em>P</em> &lt; .0001). In this study, only a minority (23%) of patients receiving vitamin D had their 1,25(OH)<sub>2</sub> vitamin D level checked.<br/><br/><br/><br/><strong>Pearl</strong>: Lymphocyte count and 1,25(OH)<sub>2</sub> vitamin D levels can be helpful tests in assessing sarcoidosis activity. Patients with sarcoidosis who receive vitamin D should have their 1.25(OH)<sub>2</sub> vitamin D levels monitored.</p> <p> <em>Dr. Paauw is professor of medicine in the division of general internal medicine at the University of Washington, Seattle, and he serves as third-year medical student clerkship director at the University of Washington. Contact Dr. Paauw at <span class="Hyperlink"><a href="mailto:dpaauw%40uw.edu?subject=">dpaauw@uw.edu</a></span>.</em> </p> <h2>References</h2> <p>1. El Jammal et al. <span class="Hyperlink"><a href="https://www.mattioli1885journals.com/index.php/sarcoidosis/article/view/14221">Sarcoidosis Vasc Diffuse Lung Dis. 2023 Sep 13;40(3):e2023031</a></span>.<br/><br/>2. Morell F et al. <span class="Hyperlink"><a href="https://journal.chestnet.org/article/S0012-3692(15)34306-3/fulltext">Chest. 2002 Apr;121(4):1239-44</a></span>.<br/><br/>3. Jones NP et al. <span class="Hyperlink"><a href="Br J Ophthalmol. 2016 Oct;100(10):1393-6">Br J Ophthalmol. 2016 Oct;100(10):1393-6</a></span>. <br/><br/>4. Rohmer J et al.<span class="Hyperlink"><a href="https://www.tandfonline.com/doi/abs/10.1080/09273948.2018.1537399"> Ocul Immunol Inflamm. 2020 Apr 2;28(3):341-7</a></span>.<br/><br/>5. Kavathia D et al. <span class="Hyperlink"><a href="https://www.resmedjournal.com/article/S0954-6111(09)00407-7/fulltext">Respir Med. 2010 Apr;104(4):564–70</a></span>.<br/><br/>6. Sodhi A and Aldrich T. <span class="Hyperlink"><a href="https://www.amjmedsci.org/article/S0002-9629(16)30293-2/fulltext">Am J Med Sci. 2016 Sep;352(3):252-7</a></span>.</p> </itemContent> </newsItem> <newsItem> <itemMeta> <itemRole>teaser</itemRole> <itemClass>text</itemClass> <title/> <deck/> </itemMeta> <itemContent> </itemContent> </newsItem> </itemSet></root>
Disallow All Ads
Content Gating
No Gating (article Unlocked/Free)
Alternative CME
Disqus Comments
Default
Use ProPublica
Hide sidebar & use full width
render the right sidebar.
Conference Recap Checkbox
Not Conference Recap
Clinical Edge
Display the Slideshow in this Article
Medscape Article
Display survey writer
Reuters content
Disable Inline Native ads
WebMD Article

My pet peeves about the current state of primary care

Article Type
Changed
Fri, 10/20/2023 - 09:05

 

For this month’s column, I wanted to share some frustrations I have had about the current state of primary care. We all find those things that are going on in medicine that seem crazy and we just have to find a way to adapt to them. It is good to be able to share some of these thoughts with a community as distinguished as you readers. I know some of these are issues that you all struggle with and I wanted to give a voice to them. I wish I had answers to fix them.

Faxes from insurance companies

I find faxes from insurance companies immensely annoying. First, it takes time to go through lots of unwanted faxes but these faxes are extremely inaccurate. Today I received a fax telling me I might want to consider starting a statin in my 64-year-old HIV patient who has hypertension. He has been on a statin for 10 years.

Dr. Douglas S. Paauw

Another fax warned me to not combine ACE inhibitors and angiotensin II receptor blockers (ARBs) in a patient who was switched from an ACE inhibitor in July to an ARB because of a cough. The fax that was sent to me has a documented end date for the ACE inhibitor before the start date of the ARB.

We only have so much time in the day and piles of faxes are not helpful.

Speaking of faxes: Why do physical therapy offices and nursing homes fax the same form every day? Physicians do not always work in clinic every single day and it increases the workload and burden when you have to sort through three copies of the same fax. I once worked in a world where these would be sent by mail, and mailed back a week later, which seemed to work just fine.
 

Misinformation

Our patients have many sources of health information. Much of the information they get comes from family, friends, social media posts, and Internet sites. The accuracy of the information is often questionable, and in some cases, they are victims of intentional misinformation.

It is frustrating and time consuming to counter the bogus, unsubstantiated information patients receive. It is especially difficult when patients have done their own research on proven therapies (such as statins) and do not want to use them because of the many websites they have looked at that make unscientific claims about the dangers of the proposed therapy. I share evidence-based websites with my patients for their research; my favorite is medlineplus.gov.
 

Access crisis

The availability of specialty care is extremely limited now. In my health care system, there is up to a 6-month wait for appointments in neurology, cardiology, and endocrinology. This puts the burden on the primary care professional to manage the patient’s health, even when the patient really needs specialty care. It also increases the calls we receive to interpret the echocardiograms, MRIs, or lab tests ordered by specialists who do not share the interpretation of the results with their patients.

What can be done to improve this situation? Automatic consults in the hospital should be limited. Every patient who has a transient ischemic attack with a negative workup does not need neurology follow-up. The same goes for patients who have chest pain but a negative cardiac workup in the hospital – they do not need follow-up by a cardiologist, nor do those who have stable, well-managed coronary disease. We have to find a way to keep our specialists seeing the patients whom they can help the most and available for consultation in a timely fashion.

Please share your pet peeves with me. I will try to give them voice in the future. Hang in there, you are the glue that keeps this flawed system together.

Dr. Paauw is professor of medicine in the division of general internal medicine at the University of Washington, Seattle, and he serves as third-year medical student clerkship director at the University of Washington. Contact Dr. Paauw at dpaauw@uw.edu.

Publications
Topics
Sections

 

For this month’s column, I wanted to share some frustrations I have had about the current state of primary care. We all find those things that are going on in medicine that seem crazy and we just have to find a way to adapt to them. It is good to be able to share some of these thoughts with a community as distinguished as you readers. I know some of these are issues that you all struggle with and I wanted to give a voice to them. I wish I had answers to fix them.

Faxes from insurance companies

I find faxes from insurance companies immensely annoying. First, it takes time to go through lots of unwanted faxes but these faxes are extremely inaccurate. Today I received a fax telling me I might want to consider starting a statin in my 64-year-old HIV patient who has hypertension. He has been on a statin for 10 years.

Dr. Douglas S. Paauw

Another fax warned me to not combine ACE inhibitors and angiotensin II receptor blockers (ARBs) in a patient who was switched from an ACE inhibitor in July to an ARB because of a cough. The fax that was sent to me has a documented end date for the ACE inhibitor before the start date of the ARB.

We only have so much time in the day and piles of faxes are not helpful.

Speaking of faxes: Why do physical therapy offices and nursing homes fax the same form every day? Physicians do not always work in clinic every single day and it increases the workload and burden when you have to sort through three copies of the same fax. I once worked in a world where these would be sent by mail, and mailed back a week later, which seemed to work just fine.
 

Misinformation

Our patients have many sources of health information. Much of the information they get comes from family, friends, social media posts, and Internet sites. The accuracy of the information is often questionable, and in some cases, they are victims of intentional misinformation.

It is frustrating and time consuming to counter the bogus, unsubstantiated information patients receive. It is especially difficult when patients have done their own research on proven therapies (such as statins) and do not want to use them because of the many websites they have looked at that make unscientific claims about the dangers of the proposed therapy. I share evidence-based websites with my patients for their research; my favorite is medlineplus.gov.
 

Access crisis

The availability of specialty care is extremely limited now. In my health care system, there is up to a 6-month wait for appointments in neurology, cardiology, and endocrinology. This puts the burden on the primary care professional to manage the patient’s health, even when the patient really needs specialty care. It also increases the calls we receive to interpret the echocardiograms, MRIs, or lab tests ordered by specialists who do not share the interpretation of the results with their patients.

What can be done to improve this situation? Automatic consults in the hospital should be limited. Every patient who has a transient ischemic attack with a negative workup does not need neurology follow-up. The same goes for patients who have chest pain but a negative cardiac workup in the hospital – they do not need follow-up by a cardiologist, nor do those who have stable, well-managed coronary disease. We have to find a way to keep our specialists seeing the patients whom they can help the most and available for consultation in a timely fashion.

Please share your pet peeves with me. I will try to give them voice in the future. Hang in there, you are the glue that keeps this flawed system together.

Dr. Paauw is professor of medicine in the division of general internal medicine at the University of Washington, Seattle, and he serves as third-year medical student clerkship director at the University of Washington. Contact Dr. Paauw at dpaauw@uw.edu.

 

For this month’s column, I wanted to share some frustrations I have had about the current state of primary care. We all find those things that are going on in medicine that seem crazy and we just have to find a way to adapt to them. It is good to be able to share some of these thoughts with a community as distinguished as you readers. I know some of these are issues that you all struggle with and I wanted to give a voice to them. I wish I had answers to fix them.

Faxes from insurance companies

I find faxes from insurance companies immensely annoying. First, it takes time to go through lots of unwanted faxes but these faxes are extremely inaccurate. Today I received a fax telling me I might want to consider starting a statin in my 64-year-old HIV patient who has hypertension. He has been on a statin for 10 years.

Dr. Douglas S. Paauw

Another fax warned me to not combine ACE inhibitors and angiotensin II receptor blockers (ARBs) in a patient who was switched from an ACE inhibitor in July to an ARB because of a cough. The fax that was sent to me has a documented end date for the ACE inhibitor before the start date of the ARB.

We only have so much time in the day and piles of faxes are not helpful.

Speaking of faxes: Why do physical therapy offices and nursing homes fax the same form every day? Physicians do not always work in clinic every single day and it increases the workload and burden when you have to sort through three copies of the same fax. I once worked in a world where these would be sent by mail, and mailed back a week later, which seemed to work just fine.
 

Misinformation

Our patients have many sources of health information. Much of the information they get comes from family, friends, social media posts, and Internet sites. The accuracy of the information is often questionable, and in some cases, they are victims of intentional misinformation.

It is frustrating and time consuming to counter the bogus, unsubstantiated information patients receive. It is especially difficult when patients have done their own research on proven therapies (such as statins) and do not want to use them because of the many websites they have looked at that make unscientific claims about the dangers of the proposed therapy. I share evidence-based websites with my patients for their research; my favorite is medlineplus.gov.
 

Access crisis

The availability of specialty care is extremely limited now. In my health care system, there is up to a 6-month wait for appointments in neurology, cardiology, and endocrinology. This puts the burden on the primary care professional to manage the patient’s health, even when the patient really needs specialty care. It also increases the calls we receive to interpret the echocardiograms, MRIs, or lab tests ordered by specialists who do not share the interpretation of the results with their patients.

What can be done to improve this situation? Automatic consults in the hospital should be limited. Every patient who has a transient ischemic attack with a negative workup does not need neurology follow-up. The same goes for patients who have chest pain but a negative cardiac workup in the hospital – they do not need follow-up by a cardiologist, nor do those who have stable, well-managed coronary disease. We have to find a way to keep our specialists seeing the patients whom they can help the most and available for consultation in a timely fashion.

Please share your pet peeves with me. I will try to give them voice in the future. Hang in there, you are the glue that keeps this flawed system together.

Dr. Paauw is professor of medicine in the division of general internal medicine at the University of Washington, Seattle, and he serves as third-year medical student clerkship director at the University of Washington. Contact Dr. Paauw at dpaauw@uw.edu.

Publications
Publications
Topics
Article Type
Sections
Teambase XML
<?xml version="1.0" encoding="UTF-8"?>
<!--$RCSfile: InCopy_agile.xsl,v $ $Revision: 1.35 $-->
<!--$RCSfile: drupal.xsl,v $ $Revision: 1.7 $-->
<root generator="drupal.xsl" gversion="1.7"> <header> <fileName>165569</fileName> <TBEID>0C04CC5B.SIG</TBEID> <TBUniqueIdentifier>MD_0C04CC5B</TBUniqueIdentifier> <newsOrJournal>News</newsOrJournal> <publisherName>Frontline Medical Communications</publisherName> <storyname/> <articleType>353</articleType> <TBLocation>QC Done-All Pubs</TBLocation> <QCDate>20231019T142156</QCDate> <firstPublished>20231020T085852</firstPublished> <LastPublished>20231020T085852</LastPublished> <pubStatus qcode="stat:"/> <embargoDate/> <killDate/> <CMSDate>20231020T085852</CMSDate> <articleSource/> <facebookInfo/> <meetingNumber/> <byline>Douglas S. Paauw</byline> <bylineText>DOUGLAS S. PAAUW, MD</bylineText> <bylineFull>DOUGLAS S. PAAUW, MD</bylineFull> <bylineTitleText/> <USOrGlobal/> <wireDocType/> <newsDocType/> <journalDocType/> <linkLabel/> <pageRange/> <citation/> <quizID/> <indexIssueDate/> <itemClass qcode="ninat:text"/> <provider qcode="provider:imng"> <name>IMNG Medical Media</name> <rightsInfo> <copyrightHolder> <name>Frontline Medical News</name> </copyrightHolder> <copyrightNotice>Copyright (c) 2015 Frontline Medical News, a Frontline Medical Communications Inc. company. All rights reserved. This material may not be published, broadcast, copied, or otherwise reproduced or distributed without the prior written permission of Frontline Medical Communications Inc.</copyrightNotice> </rightsInfo> </provider> <abstract/> <metaDescription>For this month’s column, I wanted to share some frustrations I have had about the current state of primary care. We all find those things that are going on in m</metaDescription> <articlePDF/> <teaserImage>248215</teaserImage> <teaser>Endless faxes, patient misinformation, and lack of access to specialists top my list.</teaser> <title>My pet peeves about the current state of primary care</title> <deck/> <disclaimer/> <AuthorList/> <articleURL/> <doi/> <pubMedID/> <publishXMLStatus/> <publishXMLVersion>1</publishXMLVersion> <useEISSN>0</useEISSN> <urgency/> <pubPubdateYear/> <pubPubdateMonth/> <pubPubdateDay/> <pubVolume/> <pubNumber/> <wireChannels/> <primaryCMSID/> <CMSIDs/> <keywords/> <seeAlsos/> <publications_g> <publicationData> <publicationCode>fp</publicationCode> <pubIssueName/> <pubArticleType/> <pubTopics/> <pubCategories/> <pubSections/> </publicationData> <publicationData> <publicationCode>im</publicationCode> <pubIssueName/> <pubArticleType/> <pubTopics/> <pubCategories/> <pubSections/> </publicationData> </publications_g> <publications> <term canonical="true">15</term> <term>21</term> </publications> <sections> <term>52</term> <term>41022</term> <term canonical="true">39786</term> </sections> <topics> <term canonical="true">38029</term> </topics> <links> <link> <itemClass qcode="ninat:picture"/> <altRep contenttype="image/jpeg">images/2400c6a0.jpg</altRep> <description role="drol:caption">Dr. Douglas S. Paauw</description> <description role="drol:credit"/> </link> </links> </header> <itemSet> <newsItem> <itemMeta> <itemRole>Main</itemRole> <itemClass>text</itemClass> <title>My pet peeves about the current state of primary care</title> <deck/> </itemMeta> <itemContent> <p>For this month’s column, I wanted to share some frustrations I have had about the current state of primary care. We all find those things that are going on in medicine that seem crazy and we just have to find a way to adapt to them. It is good to be able to share some of these thoughts with a community as distinguished as you readers. I know some of these are issues that you all struggle with and I wanted to give a voice to them. I wish I had answers to fix them.</p> <h2>Faxes from insurance companies</h2> <p>I find faxes from insurance companies immensely annoying. First, it takes time to go through lots of unwanted faxes but these faxes are extremely inaccurate. Today I received a fax telling me I might want to consider starting a statin in my 64-year-old HIV patient who has hypertension. He has been on a statin for 10 years.</p> <p>[[{"fid":"248215","view_mode":"medstat_image_flush_right","fields":{"format":"medstat_image_flush_right","field_file_image_alt_text[und][0][value]":"Dr. Douglas S. Paauw, University of Washington, Seattle","field_file_image_credit[und][0][value]":"","field_file_image_caption[und][0][value]":"Dr. Douglas S. Paauw"},"type":"media","attributes":{"class":"media-element file-medstat_image_flush_right"}}]]Another fax warned me to not combine ACE inhibitors and angiotensin II receptor blockers (ARBs) in a patient who was switched from an ACE inhibitor in July to an ARB because of a cough. The fax that was sent to me has a documented end date for the ACE inhibitor before the start date of the ARB.<br/><br/>We only have so much time in the day and piles of faxes are not helpful. <br/><br/>Speaking of faxes: Why do physical therapy offices and nursing homes fax the same form every day? Physicians do not always work in clinic every single day and it increases the workload and burden when you have to sort through three copies of the same fax. I once worked in a world where these would be sent by mail, and mailed back a week later, which seemed to work just fine.<br/><br/></p> <h2>Misinformation</h2> <p>Our patients have many sources of health information. Much of the information they get comes from family, friends, social media posts, and Internet sites. The accuracy of the information is often questionable, and in some cases, they are victims of intentional misinformation. </p> <p>It is frustrating and time consuming to counter the bogus, unsubstantiated information patients receive. It is especially difficult when patients have done their own research on proven therapies (such as statins) and do not want to use them because of the many websites they have looked at that make unscientific claims about the dangers of the proposed therapy. I share evidence-based websites with my patients for their research; my favorite is <span class="Hyperlink"><a href="http://medlineplus.gov">medlineplus.gov</a></span>.<br/><br/></p> <h2>Access crisis</h2> <p>The availability of specialty care is extremely limited now. In my health care system, there is up to a 6-month wait for appointments in neurology, cardiology, and endocrinology. This puts the burden on the primary care professional to manage the patient’s health, even when the patient really needs specialty care. It also increases the calls we receive to interpret the echocardiograms, MRIs, or lab tests ordered by specialists who do not share the interpretation of the results with their patients.</p> <p>What can be done to improve this situation? Automatic consults in the hospital should be limited. Every patient who has a transient ischemic attack with a negative workup does not need neurology follow-up. The same goes for patients who have chest pain but a negative cardiac workup in the hospital – they do not need follow-up by a cardiologist, nor do those who have stable, well-managed coronary disease. We have to find a way to keep our specialists seeing the patients whom they can help the most and available for consultation in a timely fashion.<br/><br/>Please share your pet peeves with me. I will try to give them voice in the future. Hang in there, you are the glue that keeps this flawed system together.</p> <p> <em>Dr. Paauw is professor of medicine in the division of general internal medicine at the University of Washington, Seattle, and he serves as third-year medical student clerkship director at the University of Washington. Contact Dr. Paauw at <span class="Hyperlink"><a href="mailto:dpaauw%40uw.edu?subject=">dpaauw@uw.edu</a></span>. </em> </p> </itemContent> </newsItem> <newsItem> <itemMeta> <itemRole>teaser</itemRole> <itemClass>text</itemClass> <title/> <deck/> </itemMeta> <itemContent> </itemContent> </newsItem> </itemSet></root>
Disallow All Ads
Content Gating
No Gating (article Unlocked/Free)
Alternative CME
Disqus Comments
Default
Use ProPublica
Hide sidebar & use full width
render the right sidebar.
Conference Recap Checkbox
Not Conference Recap
Clinical Edge
Display the Slideshow in this Article
Medscape Article
Display survey writer
Reuters content
Disable Inline Native ads
WebMD Article

More on using expired medications

Article Type
Changed
Wed, 09/20/2023 - 13:26

A patient inquires about whether he or she can use an EpiPen after the expiration date. What should you advise?

A. The EpiPen is unlikely to be effective after the expiration date.

B. The EpiPen may be dangerous to use after the expiration date.

C. The EpiPen is likely to be okay up to 2 years past the expiration date.

Dr. Douglas S. Paauw

I think that choice C is the most accurate and will get to all the evidence shortly. The expiration date is not the date that the drug stops being effective or potentially becomes toxic. It is a date, required by law, that the manufacturer can guarantee greater than 90% original potency of the medication.

Epinephrine is a costly drug and is usually replaced when the Epipen expires. Weir and colleagues studied six epinephrine syringes 30 months past their expiration date.1 Three of the syringes and one control, nonexpired syringe were analyzed using liquid chromatography-mass spectrometry and nuclear magnetic resonance to determine epinephrine content. The contents of the other three syringes of epinephrine were cultured for bacteria and fungus, which yielded no microbial growth. The study showed that the content of epinephrine present in the original sample remained unchanged, compared with the control.

Rachid et al. looked at 35 EpiPens 3-36 months past their expiration dates.2 The percentage of epinephrine found remained 84%-101%, with all EpiPens less than 24 months past expiration having > 90% of the labeled epinephrine dose. Cantrell and colleagues evaluated a combination of 40 EpiPens and Epipen Jrs that were 1-50 months past expiration.3 These pens had not been kept in ideal conditions, as some had been in cars, outdoor cabins, and other environments without temperature control. Sixty-one percent of the Epipens and 56% of the EpiPen Juniors had > 90% of the labeled epinephrine content. I think expired Epipens can be used as a back-up option – that is, they are safe to use if there is not an Epipen available that is not expired.
 

Shelf life extension program

Lyon and colleagues reported data from the Shelf Life Extension Program (SLEP).4 A total of 122 drugs were studied representing 3,005 lots. Based on testing and stability assessment, 88% of the lots were extended at least 1 year beyond their original expiration date for an average extension of 66 months, but the additional stability period was highly variable. Several antibiotics were studied, including ciprofloxacin (mean extension, 55 months), amoxicillin (mean extension, 23 months), and doxycycline (mean extension, 50 months).

What about other drugs not in pill form?

I am frequently asked about the longevity of medication formulations that are not in pill form. For example, I have been asked about using expired eye drops. There are few data on this. Reis at al. studied whether travoprost that was past the expiration date still lowered intraocular pressures.5 Intraocular pressures in glaucoma patients treated with travoprost 6 weeks after the seal was broken were compared with pressures when drops were used immediately after the container seal was broken. There was no significant difference in intraocular pressure between the two treatment groups during the study.

I found one case report of harm from using expired eye medications. Use of expired eye drops was associated with a case of bilateral toxic epithelial keratopathy.6 Eye drops can be contaminated and cause irritation from the breakdown products of preservatives.

Many people use inhalers for many years. This is especially true for albuterol, which is often used for very intermittent symptoms. I found one recent study on the stability of albuterol. Kutty et al. studied expired albuterol inhalers and solutions up to 20 years past expiration.7 Almost all lots of albuterol maintained > 90% of product (73%-103%), many years past their expiration date. Even at 73% retained activity, the dose would likely be effective.
 

Pearl: Expired epinephrine and albuterol appear to retain activity several years past expiration.

Dr. Paauw is professor of medicine in the division of general internal medicine at the University of Washington, Seattle, and he serves as third-year medical student clerkship director at the University of Washington. He has no conflicts of interest. Contact Dr. Paauw at dpaauw@uw.edu.

References

1. Weir WB et al. Prehosp Emerg Care. 2018 Jul-Aug;22(4):414-8.

2. Rachid O et al. Ann Allergy Asthma Immunol. 2015 Apr;114(4):354-6.

3. Cantrell FL et al. Ann Intern Med. 2017 Jun 20;166(12):918-9.

4. Lyon RC et al. J Pharmaceut Sci. 2006;95(7):1549-60.

5. Reis R et al. Clin Ther. 2004 Dec;26(12):2121-7.

6. AlGhadeer H, AlHumaiden A. J Clin Pharm Ther. 2022 Dec;47(12):2379-82.

7. Kutty RG et al. Heliyon. 2022 Aug 5;8(8):e10104.

Publications
Topics
Sections

A patient inquires about whether he or she can use an EpiPen after the expiration date. What should you advise?

A. The EpiPen is unlikely to be effective after the expiration date.

B. The EpiPen may be dangerous to use after the expiration date.

C. The EpiPen is likely to be okay up to 2 years past the expiration date.

Dr. Douglas S. Paauw

I think that choice C is the most accurate and will get to all the evidence shortly. The expiration date is not the date that the drug stops being effective or potentially becomes toxic. It is a date, required by law, that the manufacturer can guarantee greater than 90% original potency of the medication.

Epinephrine is a costly drug and is usually replaced when the Epipen expires. Weir and colleagues studied six epinephrine syringes 30 months past their expiration date.1 Three of the syringes and one control, nonexpired syringe were analyzed using liquid chromatography-mass spectrometry and nuclear magnetic resonance to determine epinephrine content. The contents of the other three syringes of epinephrine were cultured for bacteria and fungus, which yielded no microbial growth. The study showed that the content of epinephrine present in the original sample remained unchanged, compared with the control.

Rachid et al. looked at 35 EpiPens 3-36 months past their expiration dates.2 The percentage of epinephrine found remained 84%-101%, with all EpiPens less than 24 months past expiration having > 90% of the labeled epinephrine dose. Cantrell and colleagues evaluated a combination of 40 EpiPens and Epipen Jrs that were 1-50 months past expiration.3 These pens had not been kept in ideal conditions, as some had been in cars, outdoor cabins, and other environments without temperature control. Sixty-one percent of the Epipens and 56% of the EpiPen Juniors had > 90% of the labeled epinephrine content. I think expired Epipens can be used as a back-up option – that is, they are safe to use if there is not an Epipen available that is not expired.
 

Shelf life extension program

Lyon and colleagues reported data from the Shelf Life Extension Program (SLEP).4 A total of 122 drugs were studied representing 3,005 lots. Based on testing and stability assessment, 88% of the lots were extended at least 1 year beyond their original expiration date for an average extension of 66 months, but the additional stability period was highly variable. Several antibiotics were studied, including ciprofloxacin (mean extension, 55 months), amoxicillin (mean extension, 23 months), and doxycycline (mean extension, 50 months).

What about other drugs not in pill form?

I am frequently asked about the longevity of medication formulations that are not in pill form. For example, I have been asked about using expired eye drops. There are few data on this. Reis at al. studied whether travoprost that was past the expiration date still lowered intraocular pressures.5 Intraocular pressures in glaucoma patients treated with travoprost 6 weeks after the seal was broken were compared with pressures when drops were used immediately after the container seal was broken. There was no significant difference in intraocular pressure between the two treatment groups during the study.

I found one case report of harm from using expired eye medications. Use of expired eye drops was associated with a case of bilateral toxic epithelial keratopathy.6 Eye drops can be contaminated and cause irritation from the breakdown products of preservatives.

Many people use inhalers for many years. This is especially true for albuterol, which is often used for very intermittent symptoms. I found one recent study on the stability of albuterol. Kutty et al. studied expired albuterol inhalers and solutions up to 20 years past expiration.7 Almost all lots of albuterol maintained > 90% of product (73%-103%), many years past their expiration date. Even at 73% retained activity, the dose would likely be effective.
 

Pearl: Expired epinephrine and albuterol appear to retain activity several years past expiration.

Dr. Paauw is professor of medicine in the division of general internal medicine at the University of Washington, Seattle, and he serves as third-year medical student clerkship director at the University of Washington. He has no conflicts of interest. Contact Dr. Paauw at dpaauw@uw.edu.

References

1. Weir WB et al. Prehosp Emerg Care. 2018 Jul-Aug;22(4):414-8.

2. Rachid O et al. Ann Allergy Asthma Immunol. 2015 Apr;114(4):354-6.

3. Cantrell FL et al. Ann Intern Med. 2017 Jun 20;166(12):918-9.

4. Lyon RC et al. J Pharmaceut Sci. 2006;95(7):1549-60.

5. Reis R et al. Clin Ther. 2004 Dec;26(12):2121-7.

6. AlGhadeer H, AlHumaiden A. J Clin Pharm Ther. 2022 Dec;47(12):2379-82.

7. Kutty RG et al. Heliyon. 2022 Aug 5;8(8):e10104.

A patient inquires about whether he or she can use an EpiPen after the expiration date. What should you advise?

A. The EpiPen is unlikely to be effective after the expiration date.

B. The EpiPen may be dangerous to use after the expiration date.

C. The EpiPen is likely to be okay up to 2 years past the expiration date.

Dr. Douglas S. Paauw

I think that choice C is the most accurate and will get to all the evidence shortly. The expiration date is not the date that the drug stops being effective or potentially becomes toxic. It is a date, required by law, that the manufacturer can guarantee greater than 90% original potency of the medication.

Epinephrine is a costly drug and is usually replaced when the Epipen expires. Weir and colleagues studied six epinephrine syringes 30 months past their expiration date.1 Three of the syringes and one control, nonexpired syringe were analyzed using liquid chromatography-mass spectrometry and nuclear magnetic resonance to determine epinephrine content. The contents of the other three syringes of epinephrine were cultured for bacteria and fungus, which yielded no microbial growth. The study showed that the content of epinephrine present in the original sample remained unchanged, compared with the control.

Rachid et al. looked at 35 EpiPens 3-36 months past their expiration dates.2 The percentage of epinephrine found remained 84%-101%, with all EpiPens less than 24 months past expiration having > 90% of the labeled epinephrine dose. Cantrell and colleagues evaluated a combination of 40 EpiPens and Epipen Jrs that were 1-50 months past expiration.3 These pens had not been kept in ideal conditions, as some had been in cars, outdoor cabins, and other environments without temperature control. Sixty-one percent of the Epipens and 56% of the EpiPen Juniors had > 90% of the labeled epinephrine content. I think expired Epipens can be used as a back-up option – that is, they are safe to use if there is not an Epipen available that is not expired.
 

Shelf life extension program

Lyon and colleagues reported data from the Shelf Life Extension Program (SLEP).4 A total of 122 drugs were studied representing 3,005 lots. Based on testing and stability assessment, 88% of the lots were extended at least 1 year beyond their original expiration date for an average extension of 66 months, but the additional stability period was highly variable. Several antibiotics were studied, including ciprofloxacin (mean extension, 55 months), amoxicillin (mean extension, 23 months), and doxycycline (mean extension, 50 months).

What about other drugs not in pill form?

I am frequently asked about the longevity of medication formulations that are not in pill form. For example, I have been asked about using expired eye drops. There are few data on this. Reis at al. studied whether travoprost that was past the expiration date still lowered intraocular pressures.5 Intraocular pressures in glaucoma patients treated with travoprost 6 weeks after the seal was broken were compared with pressures when drops were used immediately after the container seal was broken. There was no significant difference in intraocular pressure between the two treatment groups during the study.

I found one case report of harm from using expired eye medications. Use of expired eye drops was associated with a case of bilateral toxic epithelial keratopathy.6 Eye drops can be contaminated and cause irritation from the breakdown products of preservatives.

Many people use inhalers for many years. This is especially true for albuterol, which is often used for very intermittent symptoms. I found one recent study on the stability of albuterol. Kutty et al. studied expired albuterol inhalers and solutions up to 20 years past expiration.7 Almost all lots of albuterol maintained > 90% of product (73%-103%), many years past their expiration date. Even at 73% retained activity, the dose would likely be effective.
 

Pearl: Expired epinephrine and albuterol appear to retain activity several years past expiration.

Dr. Paauw is professor of medicine in the division of general internal medicine at the University of Washington, Seattle, and he serves as third-year medical student clerkship director at the University of Washington. He has no conflicts of interest. Contact Dr. Paauw at dpaauw@uw.edu.

References

1. Weir WB et al. Prehosp Emerg Care. 2018 Jul-Aug;22(4):414-8.

2. Rachid O et al. Ann Allergy Asthma Immunol. 2015 Apr;114(4):354-6.

3. Cantrell FL et al. Ann Intern Med. 2017 Jun 20;166(12):918-9.

4. Lyon RC et al. J Pharmaceut Sci. 2006;95(7):1549-60.

5. Reis R et al. Clin Ther. 2004 Dec;26(12):2121-7.

6. AlGhadeer H, AlHumaiden A. J Clin Pharm Ther. 2022 Dec;47(12):2379-82.

7. Kutty RG et al. Heliyon. 2022 Aug 5;8(8):e10104.

Publications
Publications
Topics
Article Type
Sections
Teambase XML
<?xml version="1.0" encoding="UTF-8"?>
<!--$RCSfile: InCopy_agile.xsl,v $ $Revision: 1.35 $-->
<!--$RCSfile: drupal.xsl,v $ $Revision: 1.7 $-->
<root generator="drupal.xsl" gversion="1.7"> <header> <fileName>165151</fileName> <TBEID>0C04C38F.SIG</TBEID> <TBUniqueIdentifier>MD_0C04C38F</TBUniqueIdentifier> <newsOrJournal>News</newsOrJournal> <publisherName>Frontline Medical Communications</publisherName> <storyname/> <articleType>353</articleType> <TBLocation>QC Done-All Pubs</TBLocation> <QCDate>20230920T113059</QCDate> <firstPublished>20230920T115442</firstPublished> <LastPublished>20230920T115442</LastPublished> <pubStatus qcode="stat:"/> <embargoDate/> <killDate/> <CMSDate>20230920T115442</CMSDate> <articleSource/> <facebookInfo/> <meetingNumber/> <byline>Douglas S. Pauww</byline> <bylineText>DOUGLAS S. PAAUW, MD</bylineText> <bylineFull>DOUGLAS S. PAAUW, MD</bylineFull> <bylineTitleText/> <USOrGlobal/> <wireDocType/> <newsDocType/> <journalDocType/> <linkLabel/> <pageRange/> <citation/> <quizID/> <indexIssueDate/> <itemClass qcode="ninat:text"/> <provider qcode="provider:imng"> <name>IMNG Medical Media</name> <rightsInfo> <copyrightHolder> <name>Frontline Medical News</name> </copyrightHolder> <copyrightNotice>Copyright (c) 2015 Frontline Medical News, a Frontline Medical Communications Inc. company. All rights reserved. This material may not be published, broadcast, copied, or otherwise reproduced or distributed without the prior written permission of Frontline Medical Communications Inc.</copyrightNotice> </rightsInfo> </provider> <abstract/> <metaDescription>The expiration date is not the date that the drug stops being effective or potentially becomes toxic.</metaDescription> <articlePDF/> <teaserImage>248215</teaserImage> <teaser>The expiration date is a date, required by law, that the manufacturer can guarantee greater than 90% original potency of the medication.</teaser> <title>More on using expired medications</title> <deck/> <disclaimer/> <AuthorList/> <articleURL/> <doi/> <pubMedID/> <publishXMLStatus/> <publishXMLVersion>1</publishXMLVersion> <useEISSN>0</useEISSN> <urgency/> <pubPubdateYear/> <pubPubdateMonth/> <pubPubdateDay/> <pubVolume/> <pubNumber/> <wireChannels/> <primaryCMSID/> <CMSIDs/> <keywords/> <seeAlsos/> <publications_g> <publicationData> <publicationCode>fp</publicationCode> <pubIssueName/> <pubArticleType/> <pubTopics/> <pubCategories/> <pubSections/> </publicationData> <publicationData> <publicationCode>im</publicationCode> <pubIssueName/> <pubArticleType/> <pubTopics/> <pubCategories/> <pubSections/> </publicationData> </publications_g> <publications> <term canonical="true">15</term> <term>21</term> </publications> <sections> <term>52</term> <term canonical="true">39786</term> <term>41022</term> </sections> <topics> <term>280</term> <term canonical="true">38029</term> </topics> <links> <link> <itemClass qcode="ninat:picture"/> <altRep contenttype="image/jpeg">images/2400c6a0.jpg</altRep> <description role="drol:caption">Dr. Douglas S. Paauw</description> <description role="drol:credit"/> </link> </links> </header> <itemSet> <newsItem> <itemMeta> <itemRole>Main</itemRole> <itemClass>text</itemClass> <title>More on using expired medications</title> <deck/> </itemMeta> <itemContent> <p>A patient inquires about whether he or she can use an EpiPen after the expiration date. What should you advise?</p> <p>A. The EpiPen is unlikely to be effective after the expiration date.<br/><br/>B. The EpiPen may be dangerous to use after the expiration date.<br/><br/>C. The EpiPen is likely to be okay up to 2 years past the expiration date.<br/><br/>[[{"fid":"248215","view_mode":"medstat_image_flush_right","fields":{"format":"medstat_image_flush_right","field_file_image_alt_text[und][0][value]":"Dr. Douglas S. Paauw, University of Washington, Seattle","field_file_image_credit[und][0][value]":"","field_file_image_caption[und][0][value]":"Dr. Douglas S. Paauw"},"type":"media","attributes":{"class":"media-element file-medstat_image_flush_right"}}]]I think that choice C is the most accurate and will get to all the evidence shortly. <span class="tag metaDescription">The expiration date is not the date that the drug stops being effective or potentially becomes toxic.</span> It is a date, required by law, that the manufacturer can guarantee greater than 90% original potency of the medication. <br/><br/>Epinephrine is a costly drug and is usually replaced when the Epipen expires. Weir and colleagues studied six epinephrine syringes 30 months past their expiration date.<sup>1</sup> Three of the syringes and one control, nonexpired syringe were analyzed using liquid chromatography-mass spectrometry and nuclear magnetic resonance to determine epinephrine content. The contents of the other three syringes of epinephrine were cultured for bacteria and fungus, which yielded no microbial growth. The study showed that the content of epinephrine present in the original sample remained unchanged, compared with the control. <br/><br/>Rachid et al. looked at 35 EpiPens 3-36 months past their expiration dates.<sup>2</sup> The percentage of epinephrine found remained 84%-101%, with all EpiPens less than 24 months past expiration having &gt; 90% of the labeled epinephrine dose. Cantrell and colleagues evaluated a combination of 40 EpiPens and Epipen Jrs that were 1-50 months past expiration.<sup>3 </sup>These pens had not been kept in ideal conditions, as some had been in cars, outdoor cabins, and other environments without temperature control. Sixty-one percent of the Epipens and 56% of the EpiPen Juniors had &gt; 90% of the labeled epinephrine content. I think expired Epipens can be used as a back-up option – that is, they are safe to use if there is not an Epipen available that is not expired.<br/><br/></p> <h2>Shelf life extension program</h2> <p>Lyon and colleagues reported data from the Shelf Life Extension Program (SLEP).<sup>4</sup> A total of 122 drugs were studied representing 3,005 lots. Based on testing and stability assessment, 88% of the lots were extended at least 1 year beyond their original expiration date for an average extension of 66 months, but the additional stability period was highly variable. Several antibiotics were studied, including ciprofloxacin (mean extension, 55 months), amoxicillin (mean extension, 23 months), and doxycycline (mean extension, 50 months).</p> <h2>What about other drugs not in pill form?</h2> <p>I am frequently asked about the longevity of medication formulations that are not in pill form. For example, I have been asked about using expired eye drops. There are few data on this. Reis at al. studied whether travoprost that was past the expiration date still lowered intraocular pressures.<sup>5</sup> Intraocular pressures in glaucoma patients treated with travoprost 6 weeks after the seal was broken were compared with pressures when drops were used immediately after the container seal was broken. There was no significant difference in intraocular pressure between the two treatment groups during the study. </p> <p>I found one case report of harm from using expired eye medications. Use of expired eye drops was associated with a case of bilateral toxic epithelial keratopathy.<sup>6</sup> Eye drops can be contaminated and cause irritation from the breakdown products of preservatives.<br/><br/>Many people use inhalers for many years. This is especially true for albuterol, which is often used for very intermittent symptoms. I found one recent study on the stability of albuterol. Kutty et al. studied expired albuterol inhalers and solutions up to 20 years past expiration.<sup>7</sup> Almost all lots of albuterol maintained &gt; 90% of product (73%-103%), many years past their expiration date. Even at 73% retained activity, the dose would likely be effective. <br/><br/></p> <p>Pearl: Expired epinephrine and albuterol appear to retain activity several years past expiration.<span class="end"/></p> <p> <em>Dr. Paauw is professor of medicine in the division of general internal medicine at the University of Washington, Seattle, and he serves as third-year medical student clerkship director at the University of Washington. He has no conflicts of interest. Contact Dr. Paauw at dpaauw@uw.edu. </em> </p> <h2>References</h2> <p>1. Weir WB et al. <span class="Hyperlink"><a href="https://www.tandfonline.com/doi/abs/10.1080/10903127.2017.1402109?journalCode=ipec20">Prehosp Emerg Care. 2018 Jul-Aug;22(4):414-8</a></span>.<br/><br/>2. Rachid O et al. <span class="Hyperlink"><a href="https://www.annallergy.org/article/S1081-1206(15)00046-0/fulltext">Ann Allergy Asthma Immunol. 2015 Apr;114(4):354-6</a></span>.<br/><br/>3. Cantrell FL et al. <span class="Hyperlink"><a href="https://www.acpjournals.org/doi/10.7326/L16-0612">Ann Intern Med. 2017 Jun 20;166(12):918-9</a></span>.<br/><br/>4. Lyon RC et al. <span class="Hyperlink"><a href="https://jpharmsci.org/article/S0022-3549(16)32045-7/fulltext">J Pharmaceut Sci. 2006;95(7):1549-60</a></span>.<br/><br/>5. Reis R et al. <span class="Hyperlink"><a href="https://www.clinicaltherapeutics.com/article/S0149-2918(04)00076-1/pdf">Clin Ther. 2004 Dec;26(12):2121-7</a></span>.<br/><br/>6. AlGhadeer H, AlHumaiden A. <span class="Hyperlink"><a href="https://onlinelibrary.wiley.com/doi/10.1111/jcpt.13808">J Clin Pharm Ther. 2022 Dec;47(12):2379-82</a>.</span><br/><br/>7. Kutty RG et al. <span class="Hyperlink"><a href="https://europepmc.org/article/med/36016533">Heliyon. 2022 Aug 5;8(8):e10104</a>.</span></p> </itemContent> </newsItem> <newsItem> <itemMeta> <itemRole>teaser</itemRole> <itemClass>text</itemClass> <title/> <deck/> </itemMeta> <itemContent> </itemContent> </newsItem> </itemSet></root>
Disallow All Ads
Content Gating
No Gating (article Unlocked/Free)
Alternative CME
Disqus Comments
Default
Use ProPublica
Hide sidebar & use full width
render the right sidebar.
Conference Recap Checkbox
Not Conference Recap
Clinical Edge
Display the Slideshow in this Article
Medscape Article
Display survey writer
Reuters content
Disable Inline Native ads
WebMD Article

My favorite iron pearls

Article Type
Changed
Thu, 09/21/2023 - 14:09

A 45-year-old women presents for evaluation of fatigue. She has been tired for the past 6 months. She has had no problems with sleep and no other new symptoms. Her physical exam is unremarkable. Her Patient Health Questionnaire–9 score is 4. Lab results are as follows: hemoglobin, 13 g/dL; hematocrit, 39%; mean corpuscular volume, 90 fL; blood urea nitrogen, 10 mg/dL; Cr, 1.0 mg/dL; AST, 20 IU/L; ALT, 15 IU/L; ferritin, 35 mcg/mL; thyroid-stimulating hormone, 3.5 mIU/L.

What would you recommend?

A. Sertraline

B. Sleep study

C. Iron supplementation

Dr. Douglas S. Paauw


I would treat this patient with iron. Verdon and colleagues conducted a randomized, double-blind placebo-controlled trial of iron treatment in nonanemic women.1 The women who received iron had a much greater reduction in fatigue score, compared with the women who did not (P < .004). Only women with ferritin levels less than 50 mcg/L benefited. Houston and colleagues performed a systematic review of the literature of iron supplementation for fatigue and concluded that iron should be considered for treatment of fatigue in nonanemic women.2 The key number for benefit was a ferritin level less than 50 mcg/L.

Hair thinning is a common concern for many women. Does iron deficiency have a possible role in this problem? A number of studies have correlated low ferritin levels with hair loss.3 There is less clear evidence of iron treatment being effective. Hard studied 140 women with diffuse hair loss, and found 19% had iron deficiency without anemia.4 All patients with iron deficiency were treated with oral iron and in all patients hair loss ceased, and hair regrowth occurred. The target ferritin goal for treatment is greater than 40 mcg/L.5

Iron deficiency is an important trigger for restless leg syndrome (RLS). All patients who present with RLS should have ferritin checked, and appropriate evaluation for the cause of iron deficiency if ferritin levels are low. Allen and colleagues published clinical practice guidelines for iron treatment of RLS.6 The guidelines conclude that ferric carboxymaltose (1,000 mg) is effective for treating moderate to severe RLS in those with serum ferritin less than 300 mcg/L and could be used as first-line therapy for RLS in adults, with oral iron (65 mg) possibly effective in patients with ferritin levels less than 75 mcg/L.

Pearl: Think of iron as therapy for fatigue in nonanemic women with a ferritin level less than 50 mcg/L, consider a trial of iron for thinning hair in women with ferritin levels less than 50 mcg/L, and a trial of iron in those with RLS with ferritin levels less than 75 mcg/L.

Dr. Paauw is professor of medicine in the division of general internal medicine at the University of Washington, Seattle, and he serves as third-year medical student clerkship director at the University of Washington. Contact Dr. Paauw at dpaauw@uw.edu.

References

1. Verdon F et al. BMJ. 2003 May 24;326(7399):1124. .

2. Houston BL et al. BMJ Open. 2018 Apr 5;8(4):e019240.

3. Almohanna HM et al. Dermatol Ther (Heidelb). 2019 Mar;9(1):51-70. .

4. Hard S. Acta Derm Venereol. 1963;43:562-9.

5. Kantor J et al. J Invest Dermatol. 2003 Nov;121(5):985-8. .

6. Allen RP et al. Sleep Med. 2018 Jan;41:27-44. .

Publications
Topics
Sections

A 45-year-old women presents for evaluation of fatigue. She has been tired for the past 6 months. She has had no problems with sleep and no other new symptoms. Her physical exam is unremarkable. Her Patient Health Questionnaire–9 score is 4. Lab results are as follows: hemoglobin, 13 g/dL; hematocrit, 39%; mean corpuscular volume, 90 fL; blood urea nitrogen, 10 mg/dL; Cr, 1.0 mg/dL; AST, 20 IU/L; ALT, 15 IU/L; ferritin, 35 mcg/mL; thyroid-stimulating hormone, 3.5 mIU/L.

What would you recommend?

A. Sertraline

B. Sleep study

C. Iron supplementation

Dr. Douglas S. Paauw


I would treat this patient with iron. Verdon and colleagues conducted a randomized, double-blind placebo-controlled trial of iron treatment in nonanemic women.1 The women who received iron had a much greater reduction in fatigue score, compared with the women who did not (P < .004). Only women with ferritin levels less than 50 mcg/L benefited. Houston and colleagues performed a systematic review of the literature of iron supplementation for fatigue and concluded that iron should be considered for treatment of fatigue in nonanemic women.2 The key number for benefit was a ferritin level less than 50 mcg/L.

Hair thinning is a common concern for many women. Does iron deficiency have a possible role in this problem? A number of studies have correlated low ferritin levels with hair loss.3 There is less clear evidence of iron treatment being effective. Hard studied 140 women with diffuse hair loss, and found 19% had iron deficiency without anemia.4 All patients with iron deficiency were treated with oral iron and in all patients hair loss ceased, and hair regrowth occurred. The target ferritin goal for treatment is greater than 40 mcg/L.5

Iron deficiency is an important trigger for restless leg syndrome (RLS). All patients who present with RLS should have ferritin checked, and appropriate evaluation for the cause of iron deficiency if ferritin levels are low. Allen and colleagues published clinical practice guidelines for iron treatment of RLS.6 The guidelines conclude that ferric carboxymaltose (1,000 mg) is effective for treating moderate to severe RLS in those with serum ferritin less than 300 mcg/L and could be used as first-line therapy for RLS in adults, with oral iron (65 mg) possibly effective in patients with ferritin levels less than 75 mcg/L.

Pearl: Think of iron as therapy for fatigue in nonanemic women with a ferritin level less than 50 mcg/L, consider a trial of iron for thinning hair in women with ferritin levels less than 50 mcg/L, and a trial of iron in those with RLS with ferritin levels less than 75 mcg/L.

Dr. Paauw is professor of medicine in the division of general internal medicine at the University of Washington, Seattle, and he serves as third-year medical student clerkship director at the University of Washington. Contact Dr. Paauw at dpaauw@uw.edu.

References

1. Verdon F et al. BMJ. 2003 May 24;326(7399):1124. .

2. Houston BL et al. BMJ Open. 2018 Apr 5;8(4):e019240.

3. Almohanna HM et al. Dermatol Ther (Heidelb). 2019 Mar;9(1):51-70. .

4. Hard S. Acta Derm Venereol. 1963;43:562-9.

5. Kantor J et al. J Invest Dermatol. 2003 Nov;121(5):985-8. .

6. Allen RP et al. Sleep Med. 2018 Jan;41:27-44. .

A 45-year-old women presents for evaluation of fatigue. She has been tired for the past 6 months. She has had no problems with sleep and no other new symptoms. Her physical exam is unremarkable. Her Patient Health Questionnaire–9 score is 4. Lab results are as follows: hemoglobin, 13 g/dL; hematocrit, 39%; mean corpuscular volume, 90 fL; blood urea nitrogen, 10 mg/dL; Cr, 1.0 mg/dL; AST, 20 IU/L; ALT, 15 IU/L; ferritin, 35 mcg/mL; thyroid-stimulating hormone, 3.5 mIU/L.

What would you recommend?

A. Sertraline

B. Sleep study

C. Iron supplementation

Dr. Douglas S. Paauw


I would treat this patient with iron. Verdon and colleagues conducted a randomized, double-blind placebo-controlled trial of iron treatment in nonanemic women.1 The women who received iron had a much greater reduction in fatigue score, compared with the women who did not (P < .004). Only women with ferritin levels less than 50 mcg/L benefited. Houston and colleagues performed a systematic review of the literature of iron supplementation for fatigue and concluded that iron should be considered for treatment of fatigue in nonanemic women.2 The key number for benefit was a ferritin level less than 50 mcg/L.

Hair thinning is a common concern for many women. Does iron deficiency have a possible role in this problem? A number of studies have correlated low ferritin levels with hair loss.3 There is less clear evidence of iron treatment being effective. Hard studied 140 women with diffuse hair loss, and found 19% had iron deficiency without anemia.4 All patients with iron deficiency were treated with oral iron and in all patients hair loss ceased, and hair regrowth occurred. The target ferritin goal for treatment is greater than 40 mcg/L.5

Iron deficiency is an important trigger for restless leg syndrome (RLS). All patients who present with RLS should have ferritin checked, and appropriate evaluation for the cause of iron deficiency if ferritin levels are low. Allen and colleagues published clinical practice guidelines for iron treatment of RLS.6 The guidelines conclude that ferric carboxymaltose (1,000 mg) is effective for treating moderate to severe RLS in those with serum ferritin less than 300 mcg/L and could be used as first-line therapy for RLS in adults, with oral iron (65 mg) possibly effective in patients with ferritin levels less than 75 mcg/L.

Pearl: Think of iron as therapy for fatigue in nonanemic women with a ferritin level less than 50 mcg/L, consider a trial of iron for thinning hair in women with ferritin levels less than 50 mcg/L, and a trial of iron in those with RLS with ferritin levels less than 75 mcg/L.

Dr. Paauw is professor of medicine in the division of general internal medicine at the University of Washington, Seattle, and he serves as third-year medical student clerkship director at the University of Washington. Contact Dr. Paauw at dpaauw@uw.edu.

References

1. Verdon F et al. BMJ. 2003 May 24;326(7399):1124. .

2. Houston BL et al. BMJ Open. 2018 Apr 5;8(4):e019240.

3. Almohanna HM et al. Dermatol Ther (Heidelb). 2019 Mar;9(1):51-70. .

4. Hard S. Acta Derm Venereol. 1963;43:562-9.

5. Kantor J et al. J Invest Dermatol. 2003 Nov;121(5):985-8. .

6. Allen RP et al. Sleep Med. 2018 Jan;41:27-44. .

Publications
Publications
Topics
Article Type
Sections
Teambase XML
<?xml version="1.0" encoding="UTF-8"?>
<!--$RCSfile: InCopy_agile.xsl,v $ $Revision: 1.35 $-->
<!--$RCSfile: drupal.xsl,v $ $Revision: 1.7 $-->
<root generator="drupal.xsl" gversion="1.7"> <header> <fileName>164626</fileName> <TBEID>0C04B96B.SIG</TBEID> <TBUniqueIdentifier>MD_0C04B96B</TBUniqueIdentifier> <newsOrJournal>News</newsOrJournal> <publisherName>Frontline Medical Communications</publisherName> <storyname/> <articleType>353</articleType> <TBLocation>QC Done-All Pubs</TBLocation> <QCDate>20230811T121532</QCDate> <firstPublished>20230811T133046</firstPublished> <LastPublished>20230811T133046</LastPublished> <pubStatus qcode="stat:"/> <embargoDate/> <killDate/> <CMSDate>20230811T133046</CMSDate> <articleSource/> <facebookInfo/> <meetingNumber/> <byline>Douglas Pauww</byline> <bylineText>DOUGLAS S. PAAUW, MD</bylineText> <bylineFull>DOUGLAS S. PAAUW, MD</bylineFull> <bylineTitleText/> <USOrGlobal/> <wireDocType/> <newsDocType/> <journalDocType/> <linkLabel/> <pageRange/> <citation/> <quizID/> <indexIssueDate/> <itemClass qcode="ninat:text"/> <provider qcode="provider:imng"> <name>IMNG Medical Media</name> <rightsInfo> <copyrightHolder> <name>Frontline Medical News</name> </copyrightHolder> <copyrightNotice>Copyright (c) 2015 Frontline Medical News, a Frontline Medical Communications Inc. company. All rights reserved. This material may not be published, broadcast, copied, or otherwise reproduced or distributed without the prior written permission of Frontline Medical Communications Inc.</copyrightNotice> </rightsInfo> </provider> <abstract/> <metaDescription>A 45-year-old women presents for evaluation of fatigue. She has been tired for the past 6 months. She has had no problems with sleep and no other new symptoms. </metaDescription> <articlePDF/> <teaserImage>248215</teaserImage> <teaser>Iron supplements can treat fatigue, restless legs syndrome, and possibly thinning hair.</teaser> <title>My favorite iron pearls</title> <deck/> <disclaimer/> <AuthorList/> <articleURL/> <doi/> <pubMedID/> <publishXMLStatus/> <publishXMLVersion>1</publishXMLVersion> <useEISSN>0</useEISSN> <urgency/> <pubPubdateYear/> <pubPubdateMonth/> <pubPubdateDay/> <pubVolume/> <pubNumber/> <wireChannels/> <primaryCMSID/> <CMSIDs/> <keywords/> <seeAlsos/> <publications_g> <publicationData> <publicationCode>fp</publicationCode> <pubIssueName/> <pubArticleType/> <pubTopics/> <pubCategories/> <pubSections/> </publicationData> <publicationData> <publicationCode>im</publicationCode> <pubIssueName/> <pubArticleType/> <pubTopics/> <pubCategories/> <pubSections/> </publicationData> </publications_g> <publications> <term>15</term> <term canonical="true">21</term> </publications> <sections> <term canonical="true">39786</term> </sections> <topics> <term>322</term> <term canonical="true">225</term> </topics> <links> <link> <itemClass qcode="ninat:picture"/> <altRep contenttype="image/jpeg">images/2400c6a0.jpg</altRep> <description role="drol:caption">Dr. Douglas S. Paauw</description> <description role="drol:credit"/> </link> </links> </header> <itemSet> <newsItem> <itemMeta> <itemRole>Main</itemRole> <itemClass>text</itemClass> <title>My favorite iron pearls</title> <deck/> </itemMeta> <itemContent> <p>A 45-year-old women presents for evaluation of fatigue. She has been tired for the past 6 months. She has had no problems with sleep and no other new symptoms. Her physical exam is unremarkable. Her Patient Health Questionnaire–9 score is 4. Lab results are as follows: hemoglobin, 13 g/dL; hematocrit, 39%; mean corpuscular volume, 90 fL; blood urea nitrogen, 10 mg/dL; Cr, 1.0 mg/dL; AST, 20 IU/L; ALT, 15 IU/L; ferritin, 35 mcg/mL; thyroid-stimulating hormone, 3.5 mIU/L.<br/><br/>What would you recommend?<br/><br/>A. Sertraline<br/><br/>B. Sleep study<br/><br/>C. Iron supplementation</p> <p>[[{"fid":"248215","view_mode":"medstat_image_flush_right","fields":{"format":"medstat_image_flush_right","field_file_image_alt_text[und][0][value]":"Dr. Douglas S. Paauw, University of Washington, Seattle","field_file_image_credit[und][0][value]":"","field_file_image_caption[und][0][value]":"Dr. Douglas S. Paauw"},"type":"media","attributes":{"class":"media-element file-medstat_image_flush_right"}}]]<br/><br/>I would treat this patient with iron. Verdon and colleagues conducted a randomized, double-blind placebo-controlled trial of iron treatment in nonanemic women.<sup>1</sup> The women who received iron had a much grater reduction in fatigue score, compared with the women who did not (<em>P</em> &lt; .004). Only women with ferritin levels less than 50 mcg/L benefited. Houston and colleagues performed a systematic review of the literature of iron supplementation for fatigue and concluded that iron should be considered for treatment of fatigue in nonanemic women.<sup>2</sup> The key number for benefit was a ferritin level less than 50 mcg/L.<br/><br/>Hair thinning is a common concern for many women. Does iron deficiency have a possible role in this problem? A number of studies have correlated low ferritin levels with hair loss.<sup>3</sup> There is less clear evidence of iron treatment being effective. Hard studied 140 women with diffuse hair loss, and found 19% had iron deficiency without anemia.<sup>4</sup> All patients with iron deficiency were treated with oral iron and in all patients hair loss ceased, and hair regrowth occurred. The target ferritin goal for treatment is greater than 40 mcg/L.<sup>5</sup><br/><br/>Iron deficiency is an important trigger for restless leg syndrome (RLS). All patients who present with RLS should have ferritin checked, and appropriate evaluation for the cause of iron deficiency if ferritin levels are low. Allen and colleagues published clinical practice guidelines for iron treatment of RLS.<sup>6</sup> The guidelines conclude that ferric carboxymaltose (1,000 mg) is effective for treating moderate to severe RLS in those with serum ferritin less than 300 mcg/L and could be used as first-line therapy for RLS in adults, with oral iron (65 mg) possibly effective in patients with ferritin levels less than 75 mcg/L.<br/><br/><strong>Pearl:</strong> Think of iron as therapy for fatigue in nonanemic women with a ferritin level less than 50 mcg/L, consider a trial of iron for thinning hair in women with ferritin levels less than 50 mcg/L, and a trial of iron in those with RLS with ferritin levels less than 75 mcg/L.</p> <p> <em>Dr. Paauw is professor of medicine in the division of general internal medicine at the University of Washington, Seattle, and he serves as third-year medical student clerkship director at the University of Washington. Contact Dr. Paauw at <span class="Hyperlink"><a href="mailto:dpaauw%40uw.edu?subject=">dpaauw@uw.edu</a></span>.</em> </p> <h2>References</h2> <p>1. Verdon F et al. <span class="Hyperlink"><a href="https://www.bmj.com/content/326/7399/1124">BMJ. 2003 May 24;326(7399):1124</a></span>. .<br/><br/>2. Houston BL et al. <span class="Hyperlink"><a href="https://bmjopen.bmj.com/content/8/4/e019240">BMJ Open. 2018 Apr 5;8(4):e019240</a></span>. <br/><br/>3. Almohanna HM et al. <span class="Hyperlink"><a href="https://link.springer.com/article/10.1007/s13555-018-0278-6">Dermatol Ther (Heidelb). 2019 Mar;9(1):51-70</a></span>. .<br/><br/>4. Hard S. <span class="Hyperlink"><a href="https://pubmed.ncbi.nlm.nih.gov/14088648/">Acta Derm Venereol. 1963;43:562-9</a></span>. <br/><br/>5. Kantor J et al. <span class="Hyperlink"><a href="https://www.jidonline.org/article/S0022-202X(15)30494-2/fulltext">J Invest Dermatol. 2003 Nov;121(5):985-8</a></span>. .<br/><br/>6. Allen RP et al. <span class="Hyperlink"><a href="https://www.sciencedirect.com/science/article/pii/S1389945717315599?via%3Dihub">Sleep Med. 2018 Jan;41:27-44</a></span>. . </p> </itemContent> </newsItem> <newsItem> <itemMeta> <itemRole>teaser</itemRole> <itemClass>text</itemClass> <title/> <deck/> </itemMeta> <itemContent> </itemContent> </newsItem> </itemSet></root>
Disallow All Ads
Content Gating
No Gating (article Unlocked/Free)
Alternative CME
Disqus Comments
Default
Use ProPublica
Hide sidebar & use full width
render the right sidebar.
Conference Recap Checkbox
Not Conference Recap
Clinical Edge
Display the Slideshow in this Article
Medscape Article
Display survey writer
Reuters content
Disable Inline Native ads
WebMD Article